SlideShare a Scribd company logo
ĐẠI HOC THÁI NGUYÊN
TRƯ NG ĐẠI HOC KHOA HOC
Tải tài liệu tại sividoc.com
Viết đề tài giá sinh viên – ZALO:0973.287.149-TEAMLUANVAN.COM
NGUYEN TH± ANH
VE H PHƯƠNG TRÌNH PHI TUYEN
VÀ ỨNG DỤNG
LU N VĂN THẠC SĨ TOÁN HOC
Thái Nguyên - 2017
ĐẠI HOC THÁI NGUYÊN
TRƯ NG ĐẠI HOC KHOA HOC
Tải tài liệu tại sividoc.com
Viết đề tài giá sinh viên – ZALO:0973.287.149-TEAMLUANVAN.COM
NGUYEN TH± ANH
VE H PHƯƠNG TRÌNH PHI TUYEN
VÀ ỨNG DỤNG
LU N VĂN THẠC SĨ TOÁN HOC
Chuyên ngành: Phương pháp Toán sơ cap
Mã so: 60 46 01 13
NGƯ I HƯ NG DAN KHOA HOC
PGS.TS. NÔNG QUOC CHINH
Thái Nguyên - 2017
i
Viết đề tài giá sinh viên – ZALO:0973.287.149-TEAMLUANVAN.COM
Mnc lnc
M đau 1
1 M t so kien thfíc bo tr ve h phương trình 3
1.1 H phương trình tuyen tính (xem [3])................................................................... 3
1.2 H phương trình phi tuyen.................................................................................8
2 Nhfing phương pháp thư ng dùng đe giải h phương trình phi tuyen 17
2.1 Phương pháp the...................................................................................................... 17
2.2 Phương pháp đ t ȁn phụ......................................................................................... 25
2.3 Phương pháp sả dụng tính đơn đi u của hàm so(xem [2]).................................. 30
2.4 Phương pháp sả dụng bat đȁng thác..................................................................... 36
2.5 Phoi hợp nhieu phương pháp giải h phương trình............................................ 45
3 M t so fíng dnng của h phương trình phi tuyen 54
3.1 Úng dụng của h phương trình đa thác trong giải các bài toán cực trị và
cháng minh bat đȁng thác. ................................................................................. 54
3.2 M®t vài áng dụng thực te trong khoa hoc và đời song...................................... 56
Ket lu n 60
Tài li u tham khảo 61
1
Viết đề tài giá sinh viên – ZALO:0973.287.149-TEAMLUANVAN.COM
M đau
Giải h phương trình là m®t phan quan trong trong chương trình toán THPT và trong
các chuyên ngành Đại so, Giải tích. Khi đe c p đen vi c giải h phương trình, hoc sinh
trung hoc cơ sở de dàng tìm được các phương pháp giải h phương trình tuyen tính như
giải h bang phương pháp c®ng đại so, phương pháp the của hoc sinh lớp 9, hay sả dụng
định thác của lớp 10. Tuy nhiên, đoi với h phương trình phi tuyen, gan như có rat ít
tài li u nghiên cáu sâu ve lĩnh vực này. Trong các kì thi hoc sinh giỏi các cap, các kì thi
Olympic trong nước và quoc te, các h phương trình phi tuyen thường là loại khó và ít
có định dạng cũng như phương pháp giải cụ the. Vì v y hoc sinh g p rat nhieu khó khăn
trong vi c giải các bài toán ve h phương trình phi tuyen trong chương trình toán phő
thông và trong các kì thi hoc sinh giỏi các cap.
Đe tài lu n văn nghiên cáu m®t so h phương trình phi tuyen thường g p trong chương
trình toán phő thông và các phương pháp giải h phương trình này cũng như áng dụng
của chúng dùng trong ôn luy n hoc sinh giỏi lớp 9 và ôn thi tuyen sinh vào lớp 10 đ c
bi t là dành cho hoc sinh ôn thi chuyên toán.
Ngoài phan mở đau và ket lu n, lu n văn được trình bày gom ba chương:
Chương 1. M t so kien thfíc bo tr ve h phương trình.
Chương này trình bày các kien thác bő trợ, các khái ni m, định nghĩa cơ bản ve h
phương trình, t p nghi m của h phương trình nói chung và h phương trình phi tuyen
nói riêng.
2
Viết đề tài giá sinh viên – ZALO:0973.287.149-TEAMLUANVAN.COM
Chương 2. Nhfing phương pháp thư ng dùng đe giải h phương
trình phi tuyen.
Chương 2, trình bày các dạng toán thường g p của h phương trình phi tuyen và
cách giải chúng được tìm hieu qua các tài li u [1], [2], [3], [4], [5].
Chương 3. M t so fíng dnng của h phương trình phi tuyen.
Lu n văn được hoàn thành tại trường Đại hoc Khoa hoc - Đại hoc Thái Nguyên với
sự hướng dan của PGS.TS. Nông Quoc Chinh. Tác giả xin được bày tỏ lòng biet ơn sâu
sac đoi với sự quan tâm hướng dan của thay, tới các thay cô trong Ban giám hi u, Phòng
đào tạo và Khoa Toán - Tin của trường Đại hoc Khoa hoc. Đong thời tác giả xin được
cảm ơn các anh chị cùng khóa đã chỉ bảo, hướng dan cho tác giả hoc t p và hoàn thành
ke hoạch hoc t p.
Thái Nguyên, ngày 25 tháng 4 năm 2017.
Hoc viên
Nguyen Thị Anh
3
Viết đề tài giá sinh viên – ZALO:0973.287.149-TEAMLUANVAN.COM
Σ
Chương 1
M t so kien thfíc bo tr ve h
phương trình
Chương này tôi xin trình bày các kien thác bő trợ, các khái ni m, định nghĩa cơ bản
ve h phương trình, t p nghi m của h phương trình nói chung và h phương trình phi
tuyen nói riêng.
1.1 H phương trình tuyen tính (xem [3])
1.1.1 H phương trình tuyen tính tong quát
Định nghĩa 1.1. Cho K là m®t trường. H m phương trình tuyen tính n ȁn x1, x2, . . . , xn
h so trên trường K là h có dạng
a11x1 + a12x2 + · · · + a1nxn = b1
a21x1 + a22x2 + · · · + a2nxn = b2
· · ·
am1x1 + am2x2 + · · · + amnxn = bm
hay có the viet gon hơn:
n
(aikxk) = bi, i = 1, . . . , m.
k=1
Trong đó aik, bi là các phan tả thu®c trường K, aik goi là h so của ȁn xk, bi là h so
tự do, i = 1, . . . , m; k = 1, . . . , n.
4
Viết đề tài giá sinh viên – ZALO:0973.287.149-TEAMLUANVAN.COM
Σ
.
. .
Σ
. .
H phương trình (1.1) goi là h phương trình tuyen tính tőng quát.
Đăc bi t neu b1 = . . . = bm = 0 thì h (1.1) có dạng
n
(aikxk) = 0, i = 1, . . . , m.
k=1
được goi là h phương trình tuyen tính thuan nhat.
1.1.2 M t so phương pháp giải h phương trình tuyen tính
1.1.2.1 Quy tac Cramer
Định nghĩa 1.2. M®t h phương trình tuyen tính có so phương trình bang so ȁn và ma
tr n A của h có định thác |A| = 0 được goi là h Cramer.
Định lj 1.1. Định lý Cramer (Cramer’s rule – công thác xác định công thác nghi m của
h Cramer)
H Cramer
a11x1 + a12x2 + · · · + a1nxn = b1
a21x1 + a22x2 + · · · + a2nxn = b2
· · ·
an1x1 + an2x2 + · · · + annxn = bn
Là m®t h xác định. Nghi m duy nhat của h được xác định bởi công thác
∆j
xj =
∆
(6)
Trong đó ∆ = |A| và
a11 · · · b1 · · · a1n
∆j = · · ·
an1 · · · bn · · · ann
Công thác (6) được goi là công thác Cramer.
Chú j: Định thác ∆j là định thác của ma tr n nh n được tà ma tr n A của h (1.2)
bang cách thay các phan tả ở c®t thá j (H so của ȁn xj) bởi các phan tả b1, · · · , bn (Các
h so tự do).
Định lj 1.2. H n phương trình tuyen tính thuan nhat n ȁn so
n
aikxk = 0, i = 1, . . . , n
k=1
.
5
Viết đề tài giá sinh viên – ZALO:0973.287.149-TEAMLUANVAN.COM
x1 + x2 + · · · + xn − nxn = n − 1
k
n n n
x2 + 2x3 + 3x4 + · · · + nx1 = 2
(x1 + x2 + · · · + xn) − nxn = k − (k + 1)
bx1 + ax2 + bx3 + · · · + bx2001 + bx2002 = 2
chỉ có nghi m tam thường Θ = (0, · · · , 0) khi và chỉ khi định thác |A| = 0.
Ví dn 1.1 (Xem [4]). Giải h phương trình:
x1 + 2x2 + 3x3 + · · · + nxn = 1
· · ·
xn + 2x1 + 3x2 + · · · + nxn−1 = n
(Đe thi OLYMPIC Toán Sinh viên năm 1999)
Bài giai.
C®ng tat cả các phương trình của h , suy ra
x1 + x2 + · · · + xn = 1
Tiep theo, lan lượt lay phương trình thá k trà cho phương trình thá k − 1(1 < k < n);
trà phương trình thá n cho phương trình thá nhat. Suy ra
x =
(x1 + x2 + · · · + xn)
+
1
=
2
x = −
n − 2
=
2 − n
n n n
xk =
2
V y h phương trình có nghi m
xn
n
=
2 − n
n
Với k = 1, 2, · · · , n − 1.
Ví dn 1.2 (xem [4]). Cho h phương trình
ax1 + bx2 + bx3 + · · · + bx2001 + bx2002 = 1
· · · · · · · · ·
bx1 + bx2 + bx3 + · · · + bx2001 + ax2002 = 2002
Tìm đieu ki n của a đoi với b đe h đã cho có nghi m duy nhat.
⇔
6
Viết đề tài giá sinh viên – ZALO:0973.287.149-TEAMLUANVAN.COM
.
.
.
.
11 12 · · · 1r · · · 1n 1
.
b a b · · · b b
.
.
1 a b · · · b b
.
· · · · · · · · ·
(Đe thi OLYMPIC toán Sinh viên 2002)
Bài giai.
Kí hi u D là định thác của h phương trình. Ta có
a b b · · · b b
D = · · ·
b b · · · a b
. b b · · · b a .
2002×2002
C®ng tat cả các c®t với c®t đau tiên, ta có
1 b b · · · b b
(a + 2001b) · · ·
1 b · · · a b
. 1 b · · · b a .
2002×2002
(Nhân hàng đau với −1 roi c®ng vào các hàng còn lại)
= (a + 2001b).(a − b)2001
V y h đã cho có nghi m duy nhat ⇔ a /= b và a −2001b.
1.1.2.2 Thu t toán Gaoxơ (Gauss)
Trong đại so tuyen tính, thu t toán Gauss là m®t thu t toán có the được sả dụng đe
tìm nghi m của m®t h phương trình tuyen tính, tìm hạng (hay rank) của m®t ma tr n,
đe tính ma tr n nghịch đảo của m®t ma tr n vuông khả nghịch. thu t toán Gauss được
đ t theo tên của nhà toán hoc Đác là Carl Friedrich Gauss.
• N®i dung của thu t toán Gauss đe giải h phương trình tuyen tính là : Thực hi n liên
tiep các phép bien đői tương đương đe tà h phương trình (1.1) ban đau chúng ta nhân
được m®t h mới tương đương có so phương trình ít hơn, và có dạng bâc thang sau:
aJ xi1 + aJ xi2 + + aJ xir + + aJ xin = bJ
aJ
22xi2 + · · · + a2
J
nxir + · · · + aJ
2nxin = bJ
2
aJ
rrxir + · · · + aJ
rnxin = bJ
r
Trong đó xij(j = 1, · · · , n) là m®t hoán vị nào đó của các ȁn xi, (i = 1, · · · , n); aik /= 0
với k = 1, · · · , r.
7
Viết đề tài giá sinh viên – ZALO:0973.287.149-TEAMLUANVAN.COM
x1 + x2 + x3 + · · · + xn−1 + xn =
x2 + x3 + · · · + xn−1 + xn =
20052 − 2
a + x
= + · · · + x
a
2004
+ x1
(Đe thi OLYMPIC toán Sinh viên 2002)
• Các trường hợp sau có the xảy ra:
Trường hợp 1: Neu trong quá trình bien đői g p phương trình vô nghi m dạng 0x1 +· · ·+
0xn = b, b 0, thì ta dàng lại và ket lu n h vô nghi m.
Trường hợp 2: Neu r = r, khi đó phương trình cuoi cùng của h có dạng aJ
mnxin = bJ
n. H
(1.6) là m®t h Cramer, do đó h có duy nhat m®t nghi m. Có the tìm nghi m đóbang
cách giải tàng phương trình của h (1.6) tà dưới lên và thay the dan giá trị của cácȁn
tìm được tà phương trình dưới vào các phương trình trên.
Trường hợp 3 : Neu r < n thì h phương trình đã cho có vô so nghi m. Đe giải h 1.6
ta có the xem các ȁn xir+1 , · · · , xin là các tham so (lay giá trị tùy ý) và chuyen các so
hàng cháa các ȁn đó sang ve phải; roi dùng phương pháp ở trường hợp 2 đoi với các
ȁn xi1 , · · · , xir xem xir+1 , · · · , xin là tham so. Trong thực te đe bien đői h phương trình
tuyen tính 1.1 ban đau ve dạng b c thang, người ta thường thực hi n các phép bien đői
tương đương trên ma tr n bő sung AJ. Các phép bien đői đó được goi là các phép bien
đői sơ cap.
Ví dn 1.3 (xem [3]). Giải h phương trình
a
· · · · · · · · ·
1 n−1
n
2005n − 1
Bài giai.
C®ng thêm bieu thác x1 + x2 + · · · + xi−1 vào cả hai ve của phương trình thá i của h đã
cho, với i = 2, 3, · · · , n, ta có
x1 + x2
a
+ · · · + xn
a
=
a + x1 + x2 + · · · + xi−1
+ x
2005i − 1
2005i − 2005
+ x2 + · · · + xi−1
⇒
2004
=
2005i − 1
(
V y với i = 2, 3,··· , n
)
2004
xi = (x1 + x2 + · · · + xi) − (x1 + x2 + · · · + xi−1)
a 2005i+1 − 2005 a 2005i − 2005
=
2005i+1
.
a
=
2005i
2004
−
2005i
. 2004
x
1
8
Viết đề tài giá sinh viên – ZALO:0973.287.149-TEAMLUANVAN.COM
g = g
(x;y)
(y;x)
∗ Ph
ương pháp giải:
a
Lay phương trình thá nhat trà đi phương trình thá hai, ta được x1 =
2005
a a
V y xi =
2005i (i = 1, 2, · · · , n − 1); xn =
2004.2005n−1
1.2 H phương trình phi tuyen
1.2.1 Khái ni m
Định nghĩa 1.3 (xem [6]). M®t phương trình đại so không phải là tuyen tính được goi
là phương trình phi tuyen.
Định nghĩa 1.4 (xem [6]). H phương trình phi tuyen là m®t h gom nhieu phương
trình, nhieu ȁn, trong đó có ít nhat m®t phương trình phi tuyen.
1.2.2 M t vài dạng h phương trình phi tuyen
1.2.2.1 H phương trình đại so
1.2.2.1.1 H phương trình đoi xfíng loại I (xem [2])
Định nghĩa 1.5. H phương trình đoi xáng loại I là h phương trình có dạng
f(x;y) = 0
g(x;y) = 0
Với
f(x;y) = f(y;x)
Khi tráo đői vị trí của x và y trong h phương trình thì h không thay đői.
Nh n xét 1.1. Neu h có nghi m (x0; y0) thì (y0; x0) cũng là m®t nghi m của h .
Đ t
x + y = S
xy = P
Đieu ki n S2 − 4P ≥ 0
Ta được h phương trình
F(S;P ) = 0
E(S;P ) = 0
Ta tìm được S; P.
Khi đó x, y là nghi m của h phương trình X2 − SX + P = 0(∗)
Giả sả phương trình có hai nghi m X1; X2
9
Viết đề tài giá sinh viên – ZALO:0973.287.149-TEAMLUANVAN.COM
2 2
x y +
x
y = a
−
∗ −
∆ = S2 − 4P ≥ 0
2x3 = a x = 3
+ Neu ∆ > 0 thì X1 X2 nên h (1.5) có hai nghi m phân bi t (X1; X2), (X2; X1).
+ Neu ∆ = 0 thì X1 = X2 = x0 nên h có nghi m duy nhat (x0; x0).
+ H có ít nhat m®t nghiêm thỏa mãn x ≥ 0 ⇔ h (∗) có ít nhat m®t nghi m (S; P) thỏa
mãn S ≥ 0
P ≥ 0
∗ M t so bieu thfíc đoi xfíng thư ng g p:
x2 + y2 = (x + y)2 − 2xy = S2 − 2P
x3 + y3 = (x + y)(x2 + y2 − xy) = S3 − 3SP
x4 + y4 = (x2 + y2)2 − 2x2y2 = (S2 − 2P)2 − 2P 2
Ví dn 1.4. Tìm a đe h phương trình sau có nghi m duy nhat:
xy + x + y = a + 1
(Đe thi HSG lớp 9 tỉnh Hà N®i năm 2009 - 2010)
Bài giai.
Đ t
S = x + y
P = xy
ta có h mới
S + P = a + 1
SP = a
Theo định lý Viest, S và P là hai nghi m của phương trình X2 − (a + 1)X + a = 0(1).
Hơn nǎa, cũng theo Viest x, y là nghi m của phương trình Y 2 − SY + P = 0(2).
Do đó đe (1) có nghi m duy nhat thì (2) có nghi m duy nhat, tác
∆(2) = 0 ⇔ S2 = 4P ⇔ x = y
(Ho c có the dùng nh n xét, do vai trò của x, y trong moi phương trình như nhau nên h
có nghi m (m; n) thì nó cũng có nghi m (n; m). Như v y đe h có nghi m duy nhat thì
x = y.)
The vào ta được
x2 + 2x = a + 1 x2 + 2x (a + 1) = 0
⇔ qa (∗)
Đe h có nghi m duy nhat thì h ( ) có m®t nghi m x =
−2
= 1
2.1
⇒
q
3
a
2
= −1 ⇔ a = −2.
2
10
Viết đề tài giá sinh viên – ZALO:0973.287.149-TEAMLUANVAN.COM
1 1 7
xyzt = 648
x y 12
z t 18
(Thi HSG TP HCM 1986 - 1987 Vòng 1)
(Thi HSG TP HCM 07/01/1993)
Thả lại thay thỏa mãn.
V y đe h đã cho có nghi m duy nhat thì a = −2.
• Trong m®t vài trường hợp, h phương trình đoi xáng loại I còn có the g p ở h phương
trình phi tuyen 3 ȁn, 4 ȁn,... thỏa mãn đieu ki n: vai trò của các ȁn trong moi phương
trình của h như nhau.
Ví dn 1.5. Giải h phương trình
x + y + z = 1
x2 + y2 + z2 = 1
x3 + y3 + z3 = 1
Ví dn 1.6. Giải h phương trình
x + y + z + t = 22
+ =
1
+
1
=
5
Ví dn 1.7. Giải h phương trình
x + y + z = 6
x2 + y2 + z2 = 18
√
x +
√
y +
√
z = 4
(Thi HSG Toàn Quoc 1996 - 1997 Bảng A)
1.2.2.1.2 H phương trình đoi xfíng loại II (xem [1])
Định nghĩa 1.6. H phương trình đoi xáng loại II là h gom hai phương trình mà khi
ta thay đői vị trí của hai bien thì phương trình trên thành phương trình dưới và phương
trình dưới thành phương trình trên. Khi đó h phương trình có dạng
f(x;y) = 0 (1)
f(y;x) = 0 (2)
11
Viết đề tài giá sinh viên – ZALO:0973.287.149-TEAMLUANVAN.COM
−
x − 3y = 4
y
y
x y
∗ Phương pháp giải: Lay ve trà ve của hai phương trình, ta được:
(y − x)h(x;y) = 0 ⇔ x = y ho c h(x;y) = 0.
Ket hợp với (1), giải phương trình, tìm (x; y)
Ví dn 1.8 (xem [5]). Giải h phương trình:
x
y 3x = 4
x
y
(Đe thi tuyen sinh ĐH Quoc Gia Hà N®i - 1997)
Bài giai.
y x
(x − y) + 3(x − y) = 4( − )
x + y
x − 3y = 4
x
(x − y)(1 +
⇔
y
) = 0 (1)
xy
x − 3y = 4
x
(2)
(1) ⇔ x − y = 0 ho c 1 +
x + y
= 0 ⇔ x = y ho c x =
−y
.
xy
٨ Với x = y, thay vào (2), ta có
x − 3x = 4 ⇔ −2x = 4 ⇔ x = y = −2.
y + 1
٨ Với x =
−y
y + 1
,(Đieu ki n y /= −1) thay vào (2), ta có
−y
y + 1 y + 1
— 3y = 4.y.
−y
−y
y + 1
−y
y + 1
— 3y + 4y + 4 = 0
+ y + 4 = 0
⇒ y2 + 4y + 4 = 0
⇔ (y + 2)2 = 0
⇔ y = −2.(Thỏa mãn)
Thay y = −2 ⇒ x = −2
V y h phương trình đã cho có nghi m (x; y) = (−2; −2).
Ví dn 1.9 (xem [5]). Giải h phương trình:
x2 + y2 + xy = 37
y2 + z2 + xz = 28
y2 + z2 + yz = 19
⇔
⇔
Đieu ki n (x; y) (0; 0) H đã cho tương đương với
12
Viết đề tài giá sinh viên – ZALO:0973.287.149-TEAMLUANVAN.COM
2 2
x + 4
x
y
−2y = −6
2 2
2x − 13xy
+
15y = 0 (2)
(Thi HSG Toàn Quoc 1994 - 1995)
1.2.2.1.3 H phương trình phi tuyen v i ve trái đang cap. (xem
[2])
Dạng tőng quát của h phương trình phi tuyen có ve trái đȁng cap là
ax2 + bxy + cy2 = d
aJx2 + bJxy + cJy2 = dJ
∗ Phương pháp giải
٨ Xét x = 0, thay vào h (1.11), ta được
cy2 = d
cJy2 = dJ
Neu h phương trình này có nghi m thì h (1.11) có nghi m, không thì h vô nghi m.
٨ Xét x /= 0
- Neu m®t trong hai d ho c dJ bang 0. Giả sả d = 0, thì ta chia cả hai ve của phương
trình thá nhat cho x2, tà đó ta thu được phương trình có dạng A(
y
y
)2
x
y
+ B + C = 0.
x
Giải phương trình b c hai, tìm
tìm được y, tà đó thu được x.
, tà đó rút y theo x, lại thay vào phương trình thá hai,
x
- Neu cả d và dJ đeu khác 0 thì ta có the tạo ra m®t phương trình thuan nhat (h so tự
do bang 0), sau đó thực hi n cách giải như trên.
Ví dn 1.10 (xem [2]). Giải h phương trình:
x2 − 2xy + 3y2 = 9 (1)
Ví dn 1.11 (xem [2]). Giải h phương trình:
2x2 − xy + 3y2 = 13
(Đe thi lớp 10 chuyên toán 31.04.1994 TP Hà N®i)
Bài giai.
H đã cho tương đương với h :
x2 + 4xy − 2y2 = −6
25x2 + 46xy − 8y2 = 0 (1)
13
Viết đề tài giá sinh viên – ZALO:0973.287.149-TEAMLUANVAN.COM
∗
xy
=
12
a + b =
5
xz 36
Giải phương trình (1), đ t t =
y
, ta có:
x
8t2 − 46t − 25 = 0 ⇔ t1 =
−1
2
25
ho c t2 =
4
∗ Với t =
−1
, ta có h phương trình
2
x = −2y
x2 + 4xy − 2y2 = −6
25
⇔ (x; y) = (−2; 1), (2; −1)
Với t =
x =
4
y
, ta có h phương trình
4
4 25 −4 −25
25
x2 + 4xy − 2y2 = −6
⇔ (x; y) = (√139
; 139
), (√139
; ).
139
4 25
√
−4 −25
V y h đã cho có 4 nghi m (x; y) = (−2; 1), (2; −1), (
139
; √
139
), (√
139
; √
139
).
1.2.2.1.4 H phương trình đoi xfíng dạng phân thfíc
Ví dn 1.12 (xem [2]). Giải h phương trình:
x + y 5
yz 18
=
y + z 5
Bài giai.
1
Đ t a =
x
;b =
1 1
; c =
y z
x + z 13
Khi đó h đã cho tương đương với h :
12
5
b + c =
18
a + c =
13
36
Đây là m®t h phương trình tuyen tính, giải được nghi m
1 1
(a; b; c) = ( ; ;
4 6
1
9
) ⇒ (x; y; z) = (4; 6; 9)
V y h phương trình có nghi m (x; y; z) = (4; 6; 9).
Ví dn 1.13 (xem [2]). Giải h phương trình:
x2(y + z)2 = (3x2 + x + 1)y2z2
y2(x + z)2 = (4y2 + y + 1)x2z2
z2(x + y)2 = (5z2 + z + 1)x2y2
=
√
√
14
Viết đề tài giá sinh viên – ZALO:0973.287.149-TEAMLUANVAN.COM
yz x x2
(
y + z
)2 = 3 +
1
+
1
xz y y2
(
x + y
)2
= 5 +
1
+
1
Bài giai.
Neu x = 0 ⇒ y = 0; z = 0, do đó (0; 0; 0) là m®t nghi m của h .
Ta tìm các nghi m khác (0; 0; 0).
Chia cả hai ve cho x2y2z2 ta được h tương đương
Đ t a =
1
;b =
x
xy
1 1
; c = .
y z
z z2
Khi đó h đã cho tương đương với
(a + b)2 = c2 + c + 5 (1)
(b + c)2 = a2 + a + 3 (2)
(a + c)2 = b2 + b + 4 (3)
Lay (2) − (3) ⇒ (a − b)(2(a + b + c) + 1) = 1
Lay (1) − (2) ⇒ (b − c)(2(a + b + c) + 1) = 1
⇒ a − b = b − c ⇒ a + c = 2b
Thay vào (2) ⇒ 3b2 − b + 4 = 0 (Vô nghi m). Suy ra h đã cho không ton tại nghi m
khác (0; 0; 0)
∗ Chú ý: Hai kieu đoi xáng loại I và loại II là nhǎng dạng rat cơ bản. Tuy nhiên khi g p
các bài toán giải h phương trình có nhieu hơn hai ȁn, không nên chỉ nhìn hình thác roi
r p khuôn theo lời giải của dạng.
1.2.2.2 H phương trình mũ và Lôgarit (xem [2])
∗ Các phương trình mũ và loogarit:
- Phương trình: ax = b(a > 0; a /= 1), vô nghi m neu b ≤ 0; phương trình có nghi m
x = loga b neu b > 0.
- Phương trình loga x = b(a > 0; a /= 1) ⇔ x = ab.
- Phương trình af(x)
= ag(x)
(a > 0) ⇔ a = 1hoca /= 1, f(x) = g(x).
- Phương trình loga
f(x) = loga
g(x), (a > 0, a 1) ⇔
f(x) > 0 hay g(x) > 0
.
f(x) = g(x)
(
x + z
)2
= 4 +
1
+
1
15
Viết đề tài giá sinh viên – ZALO:0973.287.149-TEAMLUANVAN.COM
2
x2 + 1
x
y2 + 1
y
xy + 1
1
∗ Phương pháp giải:
Vi c giải h phương trình mũ và logarit giong như vi c giải các h phương trình đại so
với các bien đői ve bieu thác mũ và loogarit.
Phương pháp chung : Rút the, c®ng đại so, đ t ȁn phụ, bien đői tích; dánh giá, bat đȁng
thác, dùng đạo hàm;... đe đưa ve giải phương trình mũ và loogarit với 4 hướng thông
dụng là : Đưa ve cùng m®t cơ so; đ t ȁn phụ, loogarit hóa hay mũ hóa; sả dụng tính chat
của hàm so;...
Ví dn 1.14 (xem [2]). Giải h phương trình:
xlog8 y + ylog8 x = 4 (1)
log4 x − log4 y = 1 (2)
Bài giai.
Đieu ki n x, y > 0 ta có:
x
(2) ⇔ log4
y
= 1 ⇔ x = 4y nên (1):
(4y)log8 y + ylog8 4y = 4
⇔ 4log8 y.ylog8 y + ylog8 4.ylog8 y
= 4
2 2
y 3 .ylog8 y + y 3 .ylog8 y
2
y3
+log8 y
= 2
2 1
⇔ (
3
+ log8 y). log8 y =
3
⇔ 3 log8 y + 2 log8 y − 1 = 0
⇔ log8 y = −1 ho c log8 y =
3
1
⇔ y =
8
ho c y = 2
1
Do đó x =
2
ho c x = 8.
1 1
V y h PT có hai nghi m là (x; y) = ( ;
2
); (8; 2).
8
Ví dn 1.15 (xem [2]). Giải h phương trình:
1
+
1
=
2
(1)
Bài giai.
Đieu ki n x, y > 0
log2
2
. log3
3
= 1 (2)
⇔
⇔
16
Viết đề tài giá sinh viên – ZALO:0973.287.149-TEAMLUANVAN.COM
−
1 x 1
1 1 2
Ta có (1)
x2 + 1
+
y2 + 1
=
xy + 1
⇔ (xy + 1)(x2 + y2 + 2) = 2(x2 + 1)(y2 + 1)
⇔ xy(x2 + y2) + 2xy + x2 + y2 + 2 = 2x2y2 + 2(x2 + y2) + 2
⇔ xy(x − y)2 = (x − y)2
⇔ (x − y)2(xy − 1) = 0
- Neu x = y thì x = y = 1 là nghi m của h phương trình.
Xét trường hợp x = y /= 1 thì :
(1)(log2 x − 1)(log 3x + 1) = 1
⇔ log2 x. log3 x = log2 x + log3 x
1
⇔
log2
1
+ = 1
x log3 x
⇔ logx 2 + logx 3 = 1 ⇔ logx 6 = 1 ⇔ x = 6
- Neu xy = 1 thì y =
x
và x =
/ 1, ta có log2
2
. log3
3x
= 1
⇔ (log2 x − 1)(log3 x + 1) = −1
1
⇔ log2 x. log3 x = log3 x − log2 x ⇔
log
2
1
= 1
x log3 x
2
⇔ logx 2 − logx 3 = 1 ⇔ logx
3
= 1 ⇔ x =
3
.
2 2
V y h phương trình có ba nghi m là (x; y) = (1; 1), (6; 6), ( ; ).
3 3
2
17
Viết đề tài giá sinh viên – ZALO:0973.287.149-TEAMLUANVAN.COM
Chương 2
Nhfing phương pháp thư ng dùng đe
giải h phương trình phi tuyen
2.1 Phương pháp the
2.1.1. N i dung phương pháp
Thông thường ta bieu dien 1 ȁn ho c 1 bieu thác thích hợp tà m®t phương trình của
h theo các ȁn khác roi thay vào các phương trình còn lại đe được m®t h mới tương
đương với h đã cho và có so ȁn ít hơn so với h ban đau.
Ví dụ đoi với h hai phương trình hai ȁn ta có the thực hi n các bước sau:
+ Bước 1: Tà m®t phương trình của h đã cho, ta bieu dien m®t ȁn theo ȁn kia roi thay
the vào phương trình còn lại đe được phương trình mới chỉ có m®t ȁn.
+ Bước 2: Dùng phương trình mới vàa có được, thay the cho m®t trong hai phương trình
của h , tạo thành h mới tương đương với h phương trình ban đau.
+ Bước 3: Giải phương trình m®t ȁn vàa có roi suy ra nghi m của h đã cho.
∗ Chú j:
- Phương trình m®t ȁn này phải giải được.
- Trong quá trình giải h bang phương pháp the, ta thay xuat hi n phương trình có các
h so của cả hai ȁn đeu bang 0 thì h đã cho có the vô nghi m ho c vô so nghi m. - Đoi
với h gom nhieu phương trình, nhieu ȁn, vi c sả dụng phương pháp the làm giảm so ȁn
trong phương trình đe được h mới tương đương với h đã cho, ta giải h mới, tìm được
nghi m và thay ngươc trở lại tìm ȁn đã the.
18
Viết đề tài giá sinh viên – ZALO:0973.287.149-TEAMLUANVAN.COM
2
xy − z = 1
(2)
z = 0
2 2
x + 3y − xy + 2x − 5y − 4 =0
(2)
⇔ −
9 9
2.1.2. Bài t p áp dnng phương pháp the
Bài toán 2.1 (Xem [5]). Giải h phương trình:
x + y = 2 (1)
(Thi HSG TPHCM 1979 - 1980 vòng 2)
Bài giai.
Tà (1) ⇒ y = 2 − x, the vào (2), ta có
x(2 − x) − z2 = 1
⇔ 2x − x2 − z2 − 1 = 0
⇔ (x2 − 2x + 1) + z2 = 0
⇔ (x − 1)2 + z2 = 0
⇔
x = 1
Suy ra y = 1.
V y h đã cho có nghi m (x; y; z) = (1; 1; 0).
Bài toán 2.2 (xem [4]). Giải h phương trình:
x + 2y = 4 (1)
( Thi HSG TPHCM 1981 - 1982 vòng 2)
Bài giai.
Tà (1) ⇒ x = 4 − 2y, the vào phương trình (2):
(4 − 2y)2 + 3y2 − (4 − 2y)y + 2(4 − 2y) − 5y = 0
9y2 29y + 20 = 0
20
⇔ (9 − 9y)(y − ) = 0
9
20
⇔ y = 1ho c y =
9
Với y = 1 ⇒ x = 4 − 2.1 = 2
Với y = 2 ⇒ x = 4 − 2.
20
=
−4
V y h phương trình đã cho có hai nghi m (x; y) = (2; 1), (
−4
;
20
).
9 9
19
Viết đề tài giá sinh viên – ZALO:0973.287.149-TEAMLUANVAN.COM
√ √
⇔ ⇔
xy + x + y = x2 − 2y2
2.1.2.1 Phép the đại so
Định nghĩa 2.1. Phép the đại so là phép the sả dụng các bieu thác đại so ho c các ȁn
có trong phương trình đe bieu dien m®t ȁn qua các ȁn còn lại roi thay the vào các phương
trình của h đe được m®t h mới tương đương với h ban đau, với so ȁn ít hơn.
Bài toán 2.3 (xem [1]). Giải h phương trình:
x 2y − y x − 1 = 2x − 2y
(∀x, y ∈ R)
(Đe thi TS Đại hoc - Cao đȁng - Khoi D - 2008)
Bài giai.
Đieu ki n x ≥ 1; y ≥ 0.
xy + x + y = x2 − 2y2 ⇔ (x + y)(x − 2y − 1) = 0
⇔ x = −y ho c x = 2y + 1
٨ Trường hợp 1: x = −y. Vì y ≥ 0 nên x ≤ 0 (Vô lý)
٨ Trường hợp 2: x = 2y + 1, the vào phương trình thá hai, ta có:
(2y + 1)
√
2y − y
√
2y = 2y + 2
⇔ (y + 1)(
√
2y − 2) = 0
⇔ y = −1 (loại) ho c y = 2.
Với y = 2 ⇒ x = 5.
V y h đã cho có nghi m (x; y) = (5; 2).
Bài toán 2.4 (xem [6]). Giải h phương trình:
x + y + z = 0 (1)
2x + 3y + z = 0 (2)
Bài giai.
(x + 1)2 + (y + 2)2 + (z + 3)2 = 14 (3)
Xét
x + y = −z
2x + 3y = −z
coi như h phương trình hai ȁn (x; y)
2x + 2y = 2z
2x + 3y = −z
y = z
x = −2z
The vào phương trình (3), ta có:
20
Viết đề tài giá sinh viên – ZALO:0973.287.149-TEAMLUANVAN.COM
x + y = 4
Suy ra
(1 − 2z)2 + (z + 2)2 + (z + 3)2 = 14
⇔ 6z2 + 6z = 0
⇔ 6z(z + 1) = 0
⇔ z = 0 ho c z = −1.
∗ Với z = 0 ⇒ (x; y; z) = (0; 0; 0)
∗ Với z = −1 ⇒ (x; y; z) = (2; −1; −1).
V y h đã cho có hai nghi m (x; y; z) = (0; 0; 0), (2; −1; −1).
Bài toán 2.5 (xem [6]). Giải h phương trình ȁn x, y, u, v :
x3u + y3v = 14 (1)
x2u + y2v = 5 (2)
xu + yv = 2 (3)
u + v = 1 (4)
(Thi HSG TP HCM ngày 17/12/1994 Vòng 2)
Bài giai.
Tà (4) ⇒ v = 1 − u. The vào (1)
ux3 + y3(1 − u) = 14 ⇒ u(x3 − y3) = 14 − y3
⇒ u(x − y)(x2 + xy + y2) = 14 − y3.(5)
Tương tự
The v = 1 − u vào (2), ta có u(x − y)(x + y) = 5 − y2.(6)
The v = 1 − u vào (3), ta có u(x − y) = 2 − y.(7)
Tà (5) và (7) suy ra (2 − y)(x2 + xy + y2) = 14 − y3.
Tà (6) và (7) suy ra (2 − y)(x + y) = 5 − y2.
Khai trien phương trình cuoi, ta có
2(x + y)2 − xy(x + y) − 2xy − 14 = 0
2(x + y) − xy − 5 = 0
Đ t x + y = a, xy = b suy ra
2a2 − ab − 2b − 14 = 0
2a − b − 5 = 0
xy = 3
⇔
a = 4
b = 3
Suy ra (x; y) = (3; 1) ho c (1; 3).
21
Viết đề tài giá sinh viên – ZALO:0973.287.149-TEAMLUANVAN.COM
y = cos α (α ∈ [0; 2Π])
√
√
√
√
The x; y vào, ta tìm được các nghi m của h :
1
(x; y; u; v) = (3; 1; ;
2
1 1 1
), (1; 3; ; ).
2 2 2
2.1.2.2 Phép the lư ng giác
Định nghĩa 2.2. Phép the lượng giác là phép the sả dụng các bieu thác là các hàm
lượng giác thay the cho các ȁn có trong h phương trình đe được m®t h phương trình
mới tương đương với h đã cho nhưng các ȁn được thay the hoàn toàn bang ȁn mới.
Bài toán 2.6 (xem [6]). Giải h phương trình
x2 + y2 = 1 (1)
√
2(x − y)(1 + 4xy) =
√
3 (2)
Bài giai.
Tà phương trình (1) cho ta gợi ý đ t ȁn phụ đưa ve lượng giác.
Đ t
x = sin α
Khi đó phương trình (2) được viet dưới dạng:
6
(sin α − cos α)(1 + 2 sin 2α) =
2
6
⇔ sin α − cos α + 2 sin 2α sin α − 2 sin 2α cos α =
2
6
⇔ sin α − cos α + cos α − cos 3α − sin 3α − sin α =
2
⇔ sin 3α + cos 3α =
−
√
6
Π
⇔ cos(3α +
4
) =
2
— 3
= cos
5Π
2 6
7Π k2Π
⇔ α = + ho c α =
−13Π
+
k2Π
(k ∈ Z)
36 3 36
7Π 7Π 7Π
3
31Π 55Π 11Π 35Π 59Π
Vì α ∈ [0; 2Π] nên α ∈ {
36
;
36
;
36
;
36
; ; ;
36 36
;
36 36
7Π 7Π
V y h có nghi m (x; y) = (sin α; cos α) với α ∈ {
36
;
36
;
Bài toán 2.7 (xem [5]). Giải h phương trình
7Π 31Π
; ;
36 36
55Π
;
36
11Π
;
36
35Π
;
36
59Π
36
}.
z2 + 2xyz = 1 (1)
3x2y2 + 3xy2 = 1 + x3y4 (2)
z + zy4 + 4y3 = 4y + 6y2z (3)
}
22
Viết đề tài giá sinh viên – ZALO:0973.287.149-TEAMLUANVAN.COM
−
−
−
Bài giai.
Vì z = 0 không là nghi m của h phương trình nên:
1 − z2
(1) ⇔ xy =
2z
Đ t z = tan ϕ(∗) với ϕ ∈
−Π
;
Π
 {0}
Ta có xy =
1 − z2
2z
2
1 tan2 ϕ
=
2 tan ϕ
2
= cot 2ϕ
Thay vào (2) ta được:
3 cot2 2ϕ + 3y cot 2ϕ = 1 + y cot3 2ϕ
3 cot2 2ϕ − 1 1
⇔ y =
cot3 2ϕ 3 cot 2ϕ
=
cot 6ϕ
= tan 6ϕ
Suy ra: x = cot 2ϕ. cot 6ϕ, thay vào (3) ta được:
4 tan 6ϕ 4 tan3 6ϕ
z =
1 − 6 tan2 6ϕ + tan4 6ϕ
= tan 24ϕ(∗∗)
Tà (∗) và (∗∗) ta có:
tan 24ϕ = tan ϕ ⇔ 24ϕ = ϕ + kΠ, k ∈ Z ⇔ ϕ =
Với ϕ ∈
−Π
;
Π
 {0} ta thu được :
kΠ
23
, k ∈
2 2
Π 2Π 11Π
ϕ = ±
23
; ±
23
; . . . ; ±
23
V y h phương trình có các nghi m là :
Π 2Π 11Π
(x; y; z) = (cot 2ϕ. cot 6ϕ; tan 6ϕ; tan ϕ) Với ϕ ∈ ±
23
; ±
23
; . . . ; ±
23
Bài toán 2.8 (xem [5]). Giải h phương trình
2z(x + y) + 1 = x2 − y2 (1)
y2 + z2 = 1 + 2xy + 2xz − 2yz (2)
y(3x2 − 1) = −2x(x2 + 1) (3)
Bài giai.
1
Vì x = ±√
3
không thỏa mãn phương trình (3) nên:
−2(x2 + 1) 3x3 − x − 2x(x2 + 1) x3 − 3x
(3) ⇔ y =
3x2 − 1
⇔ x + y =
3x2 − 1
⇔ x + y =
3x2 − 1
Z
23
Viết đề tài giá sinh viên – ZALO:0973.287.149-TEAMLUANVAN.COM
−
— −
⇔ − − −
−
Π
sin 6ϕ cos2 ϕ
22 11 2 22 11 2
2 2 6 6 22 22 22 22 22
Đ t x = tan ϕ, ϕ ∈ −
Π
;
Π
 {−
Π
;
Π
}
2 2 6 6
⇒ cos ϕ /= 0; cos 3ϕ /= 0
Ta có:
tan ϕ + y =
tan3 ϕ 3 tan ϕ
3 tan2
ϕ − 1
⇔ y = tan 3ϕ − tan ϕ
(1) ⇔ z =
x2 y 1
2(x + y)
(do x = −y không thỏa mãn phương trình (1) ⇒ tan 3ϕ =
/ 0)
z =
(2 tan ϕ − tan 3ϕ) tan 3ϕ − 1
2 tan 3ϕ
2 tan ϕ. tan 3ϕ − tan2 3ϕ − 1
z =
z = tan ϕ −
2 tan 3ϕ
tan 3ϕ + cot 3ϕ
2
z = tan ϕ −
1 sin 3ϕ
( +
2 cos 3ϕ
1
cos 3ϕ
)
sin 3ϕ
z = tan ϕ −
sin 6ϕ
(2) ⇔ x2 + y2 + z2 − 2xy − 2xz + 2yz = 1 + x2
⇔ (y + z − x)2 = 1 + x2
(tan 3ϕ tan ϕ + tan ϕ
1
tan ϕ)2 = 1 + tan2 ϕ
sin 6ϕ
⇔ (
sin 3ϕ
−
1
− tan ϕ)2 =
1
cos 3ϕ 2 sin3 ϕ cos 3ϕ cos2 ϕ
2 sin2 3ϕ 1 2
⇔ (
2. sin 3ϕ. cos 3ϕ
− tan ϕ)
1
=
cos2 ϕ
⇔ (
cos 6ϕ
+ tan ϕ)2 =
1
⇔ (
cos 5ϕ
)2
=
1
sin 6ϕ. cos ϕ cos2 ϕ
⇔ cos 5ϕ = ± sin 6ϕ
⇔ cos 5ϕ = ± cos(
2
− 6ϕ)
Π Π
⇔ cos 5ϕ = cos(
2
− 6ϕ) ho c cos 5ϕ = cos(
2
+ 6ϕ)
Π Π
⇔ 5ϕ = ±(
2
− 6ϕ) + k2Π ho c 5ϕ = ±(
2
+ 6ϕ) + k2Π
Π k2Π Π −Π k2Π −Π
⇔ ϕ = + , ϕ = − 2kΠ ho c ϕ = + , ϕ = − 2kΠ(k ∈ Z)
Với ϕ ∈ −
Π
;
Π
 {−
Π
;
Π
} ⇒ ϕ = ±
Π
; ±
3Π
; ±
5Π
; ±
7Π
; ±
9Π
V y h phương trình đã cho có nghi m:
1
(x; y; z) = (tan ϕ; tan 3ϕ − tan ϕ; tan ϕ −
sin 6ϕ
)
⇔
24
Viết đề tài giá sinh viên – ZALO:0973.287.149-TEAMLUANVAN.COM
1 1 1
1 1 1
tan α tan β + tan β tan γ + tan γ tan α = 1 (3)
Π 3Π 5Π 7Π 9Π
Với ϕ = ±
22
; ±
22
; ±
22
; ±
22
; ±
22
Bài toán 2.9 (xem [1]). Giải h phương trình
xy + yz + xz = 1 (1)
Bài giai.
20(x +
x
) = 11(y +
y
) = 2007(z +
z
) (2)
Đieu ki n : xyz /= 0
Neu (x; y; z) là m®t nghi m của h phương trình thì (−x; −y; −z) cũng là m®t nghi m
của h , và tà (1) suy ra x, y, z cùng dau. Ta chỉ can xét (x; y; z) dương.
Với ∀x, y, zR và khác 0, ta đ t:
x = tan α
y = tan β
z = tan γ
Với 0 < α, β, γ <
2
Tà đó (1) và (2) trở thành:
20(tan α +
tan α
) = 11(tan β +
tan β
) = 2007(tan γ +
tan γ
) (4)
Ta có:
(3) ⇔ tan α(tan β + tan γ) = 1 − tan α tan γ
tan α =
1 − tan β tan γ
tan β + tan γ
Π
= cot(β + γ)
⇔ α + β + γ =
2
⇔ 2α, 2β, 2γ là các góc của m®t tam giác.
(4) ⇔ 20
tan2 α + 1
= 11
tan α
tan2 β + 1
= 2007
tan β
tan2 γ + 1
tan γ
20 11 2007
⇔
sin 2α
=
sin 2β
=
sin 2γ
Áp dụng định lý sin ta tính được ba cạnh của tam giác có ba góc 2α, 2β, 2γ là
a = 20
b = 11
c = 2007 (Không thỏa mãn bat đȁng thác tam giác)
Do đó tam giác không ton tại.
V y h vô nghi m.
Π
⇔
(∀x, y ∈ R)
25
Viết đề tài giá sinh viên – ZALO:0973.287.149-TEAMLUANVAN.COM
ut = 6
2.2 Phương pháp đ t an phn
Đoi với m®t so bài toán phác tạp, bang cách đ t ȁn phụ, sě làm bài toán trở lên de
dàng hơn. (Thường áp dụng đoi với nhǎng h phương trình xuat hi n cụm ȁn nào đó
được l p lại nhieu lan.)
2.2.1. N i dung phương pháp
Điem mau chot của phương pháp này là phải phát hi n ȁn phụ u = f(x,y) và v = g(x,y)
ngay trong tàng phương trình của h ho c sau các phép bien đői.
Thông thường các phép bien đői thường xoay quanh vi c c®ng, trà hai phương trình của
h ho c chia cả hai ve của tàng phương trình cho các so hạng khác không có sȁn trong
các phương trình của h đe tìm ra phan chung mà sau đó ta đ t ȁn phụ đe đưa h phương
trình ban đau ve m®t h phương trình mới tương đương với h đã cho. Tìm ȁn phụ và
thay the ngược trở lại đe tìm được nghi m của phương trình ban đau.
2.2.2. Bài t p áp dnng phương pháp đ t an phn
Bài toán 2.10 (xem [6]). Giải h phương trình:
x2 − y2 + x − y = 5
x3 − x2y − xy2 + y3 = 6
Bài giai.
Bien đői h đã cho ⇔
(x2 − y2) + (x − y) = 5
(x2 − y2)(x − y) = 6
Đ t u = x2 − y2 và t = x − y; ta có h phương trình
u + t = 5
⇒ u và t là hai nghi m của phương trình
X2 − 5X + 6 = 0
⇔ (X − 2)(X − 3) = 0
⇔ X = 2 ho c X = 3.
Xét hai trường hợp:
- Trường hợp 1: u = 2, t = 3
26
Viết đề tài giá sinh viên – ZALO:0973.287.149-TEAMLUANVAN.COM
3 2
x y − x − xy
= 1
⇔
Đ t u = −x2 + xy, v = x3y. H trở th
ành:
x2 − y2 = 2
x − y = 3
x + y =
3
2
x − y = 3
x =
11
6 .
y =
−7
6
- Trường hợp 2:u = 3, t = 2
x2 − y2 = 3
x − y = 2
x + y =
3
2
x − y = 2
x =
7
4 .
y =
−1
4
V y h đã cho có hai nghi m (x; y) = (
11
;
−7
), (
7
; −1
).
6 6 4 4
Bài toán 2.11 (xem [1]). Giải h phương trình:
x4 − x3y2 + x2y2 = 1
(Đe dự bị khoi A - 2007)
Bài giai.
H đã cho tương đương với
(−x2 + xy)2 + x3y2 = 1
(−x2 + xy) − x3y = 1
u2 + v = 1
u + v = 1
u2 − u = 0
u + v = 1
⇔
u(u − 1) = 0
u + v = 1
u = 1
v = 0
ho c
u = 0
.
v = 1
Xét các trường hợp:
- Trường hợp 1: u = 0; v = 1
⇔
−x2 + xy = 0 x = 0; x = y
⇔
x = y
⇔
x = y
x3y = 1 y =
x3
x4 = 1 x = ±1
- Trường hợp 2: u = 1; v = 0
−x2 + xy = 1
x3y = 0
⇒
y = 0
x2 = −1
(Vô nghi m)
V y h đã cho có hai nghi m (x; y) = (1; 1), (−1; −1).
Bài toán 2.12 (xem [1]). Giải h phương trình
x2 + y2 + x + y = 4
x(x + y + 1) + y(y + 1) = 2
1
⇒ ⇒ ⇔
⇒ ⇔ ⇔
⇔
⇒
⇒
27
Viết đề tài giá sinh viên – ZALO:0973.287.149-TEAMLUANVAN.COM
2 2 2
x + y=
S − 2P
P = xy = −2
(Đe thi dự bị - Khoi A - Năm 2005)
Bài giai.
H đã cho tương đương với
x2 + y2 + x + y = 4
x2 + y2 + x + y + xy = 2
x2 + y2 + x + y = 4
xy = −2
Đ t
x + y = S
xy = P
suy ra
S2 = x2 + y2 + 2xy
S2 − 2P + S = 4 P = −2
H đã cho tương đương với
∗Trường hợp 1
S2 − P + S = 2 S = 0 ho c S = −1
S = x + y = 0
P = xy = −2
⇔ x, y là nghi m của phương trình X2 − 0X − 2 = 0
Suy ra (x; y) = (
√
2; −
√
2), (−
√
2;
√
2).
∗ Trường hợp 2
S = x + y = −1
⇔ x, y là nghi m của phương trình Y 2 + Y − 2 = 0
⇔ Y = 1 ho c Y = −2
Suy ra (x; y) = (1; −2) ho c (−2; 1).
V y h đã cho có bon nghi m (x; y) = (
√
2; −
√
2), (−
√
2;
√
2), (1; −2), (−2; 1).
Bài toán 2.13 (xem [1]). Giải h phương trình
x2 − xy + y2 = 3(x − y)
x2 + xy + y2 = 7(x − y)3
(∀x, y ∈ R)
Bài giai.
Đ t
x + y = u
.
xy = v
H đã cho trở thành
u2 − 3u + v = 0
v = 2u2
3u2 − 3u = 0
v = 2u2
Suy ra u = 0 ho c u = 1
∗Trường hợp 1
u = 0
⇔
x − y = 0
⇔
x = 0
v = 0 xy = 0 y = 0
(Đe thi dự bị 1 - Khoi D - Năm 2006)
⇔
⇔
⇔
28
Viết đề tài giá sinh viên – ZALO:0973.287.149-TEAMLUANVAN.COM
2 2
x + 4
x
y
−2y = −6
−1
x =
y
25
√
∗Trường hợp2
u = 1
⇔
x − y = 1
⇔
x = 2
Ho c
x = −1
v = 2 xy = 2 y = 1 y = −2
V y h đã cho có nghi m (x; y) = (0; 0), (2; 1), (−1; −2).
Bài toán 2.14 (xem [6]). Giải h phương trình
2x2 − xy + 3y2 = 13
(Đe thi lớp 10 chuyên toán 31/07/1994)
Bài giai.
H đã cho tương đương với
x2 + 46xy − 8y2 = 0
x2 + 4xy − 2y2 = −6
x2 + 4xy − 2y2 = −6 x2 + 4xy − 2y2 = −6
⇔ t =
x
⇔ x = yt
y
8t2 − 46t − 25 = 0
t =
−1
ho c t =
25
2 4
∗ Trường hợp 1
2
x2 + 4xy − 2y2 = −6
∗Trường hợp 2
⇔ (x; y) = (−2; 1), (2; −1).
x =
4
y
4
⇔ (x; y) = (√
;
25
), (√
−4 ; √
−25
).
x2 + 4xy − 2y2 = −6
139 139
4
√
139
25
139
−4
−25
V y h có bon nghi m ?(x; y) = (−2; 1), (2; −1), (
139
; √
139
), (√
139
; √
139
).
29
Viết đề tài giá sinh viên – ZALO:0973.287.149-TEAMLUANVAN.COM
Bài toán 2.15 (xem [5]). Cho x; y là hai so nguyên dương sao cho
xy + x + y = 71
x2y − xy2 = 880
Tìm giá trị của bieu thác M = x2 + y2
(Thi HSG Đại Hoc IOWA Mĩ 04/1991)
30
Viết đề tài giá sinh viên – ZALO:0973.287.149-TEAMLUANVAN.COM
xy = P
∗
S = 55
S.P = 880
H đã cho tương đương với
xy + (x + y) = 71
Bài giai.
Đ t
x + y = S
Đieu ki n S2 − 4P ≥ 0
xy(x + y) = 880
H đã cho tương đương với
S + P = 71
Suy ra S và P là hai nghi m của phương trình
X2 − 71X + 880 = 0 ⇒ X1 = 55; X2 = 16.
Xét hai trường hợp:
Trường hợp 1
P = 55
S = 16
Ta có S2 − 4P = 162 − 4.55 = 36 > 0(Thỏa mãn)
Suy ra x, y là hai nghi m của phương trình A2 − 16A + 55 = 0
⇔ A1 = 11, A2 = 5 (∃(x; y) thỏa mãn đieu ki n x, y nguyên dương)
Ta có M = x2 + y2 = (x + y)2 − 2xy
M = 162 − 2.55 = 146.
∗ Trường hợp 2
P = 16
Ta có S2 − 4P = 552 − 4.16 = 2961 > 0
Suy ra x, y là hai nghi m của phương trình B2 − 55B + 16 = 0
Ta có ΔB = 2961 > 0 ⇒ ∃(x; y).
Nhưng
√
o là so vô tỉ nên không thỏa mãn đieu ki n x, y nguyên dương của bài toán.
V y M = 146.
Bài toán 2.16 (xem [5]). Giải h phương trình
x2 + y + x3y + xy2 + xy =
−5
4
x4 + y2 + xy(1 + 2x) =
−5
4
Bài giai.
Đ t x2 + y = u và xy = v thì h đã cho tương đương với
u + v + uv =
−5
4
u2 + v =
−5
4
(1)
(1)
31
Viết đề tài giá sinh viên – ZALO:0973.287.149-TEAMLUANVAN.COM
−
4
⇔
r
x(
1
− x2 =
) x3 +
1
x −
y =
2 2
2 2
4
r
16
−
r
⇔
25
Tà (2) suy ra v =
−5
u2, thay vào phương trình (1), ta có
4
5
u −
4
− u2 + u(
5
−
4
− u3) =
−5
4
⇔ 4u3 + 4u2 + u = 0
⇔ u(2u + 1)2 = 0
u = 0 ho c u =
−1
2
Xét hai trường hợp
Trường hợp 1: Neu u = 0 thì v =
−5
4 ta có
x2 + y = 0
xy =
−5
4
y = −x2
−x3 =
−5
x =
r
3 5
y = − 3
Trường hợp 2: Với u =
−1
2 thì v =
−3
, ta có h phương trình
2
x2 + y =
−1
y =
−1
− x2 y =
−1
− x2 x = 1
2 2
− 2
−3 3
−3
V y h đã cho có hai nghi m (x; y) = (1;
−3
), ( 3
2
5
; 3 25
).
16
2.3 Phương pháp sfi dnng tính đơn đi u của hàm
so(xem [2])
Định nghĩa 2.3. Hàm so f(x) xác định trên [a, b] được goi là tăng (tương áng tăng nghiêm
ng t) neu với ∀x1, x2 ∈ [a, b] và x1 < x2 ta có f(x1) ≤ f(x2) (tương áng f(x1) < f(x2)).
Tương tự được goi là giảm (tương áng giảm nghiêm ng t) neu với ∀x1, x2 ∈ [a, b], x1 < x2
ta có f(x1) ≥ f(x2) (tương áng f(x1) > f(x2)).
Nhǎng hàm so tăng ho c giảm trên [a, b] được goi là đơn đi u trong đoạn đó. Với trường
hợp tăng nghiêm ng t ho c giảm nghiêm ng t thì được goi là đơn đi u nghiêm ng t.
Định nghĩa 2.4. Cho hàm so y = f(x) xác định trên D và m®t điem x0 ∈ D. Hàm so
f(x) được goi là liên tục tại điem x0 neu lim
x→x0
f(x) = f(x0).
Hàm so y = f(x) liên tục trên D neu nó liên tục tại moi điem thu®c D.
Định lj 2.1. Neu hàm so f(x) liên tục và đơn đi u trên (a, b) và f(a).f(b) < 0 thì phương
trình f(x) = 0 có nghi m duy nhat thu®c (a, b).
2
⇔
xy =
−3
⇔
∗
∗
⇔
4
= 0
2
⇔
32
Viết đề tài giá sinh viên – ZALO:0973.287.149-TEAMLUANVAN.COM
Định lj 2.2. Neu hàm so f(x) liên tục trên (a, b) và f(a).f(b) < 0 thì phương trình f(x) = 0
có ít nhat m®t nghi m thu®c (a, b).
Định lj 2.3. Giả sả hàm so y = f(x) đơn đi u và liên tục trên khoảng (a; b). Khi đó ta
có f(u) = f(v) ⇔ u = v((u; v) ∈ (a; b))
2.3.1. N i dung phương pháp
Bước 1: Thực hi n các phéo bien đői tương đương trên các phương trình của h đe
được m®t phương trình có dạng f(u) = f(v) ho c f(u) = 0.
Bước 2: Đ t f(t) = f(u) ho c f(v), cháng minh sự đơn đi u và liên tục của hàm so f(t) trên
t p xác định của f(t).
Bước 3: Sả dụng các tính chat của hàm đơn đi u nêu trên đoi với f(t) đe tìm được moi
liên h giǎa x và y đe giải h phương trình ban đau.
• Chú j: phương pháp này được sả dụng trong các trường hợp sau:
- Loại I: M®t phương trình của h có dạng f(x) = f(y). M®t phương trình cho ta biet t p
giá trị của x ho c y. Tà đó ta suy ra hàm f(x) đơn đi u, suy ra x = y.
- Loại II: H đoi xáng mà sau khi bien đői thường đưa ve dạng f(x) = f(y) ho c f(x) = 0.
Trong đó f là hàm đơn đi u.
2.3.2. Bài t p sfi dnng tính đơn đi u của hàm so
Bài toán 2.17 (xem [5]). Giải h phương trình:
36x2y − 60x2 + 25y = 0
36xy2 − 60y2 + 25x = 0
Bài giai.
H phương trình đã cho tương đương với
x =
60y2
36y2 + 25
60x2
Suy ra x, y không âm.
y =
36x2 + 25
Neu x = 0 thì y = 0, suy ra (0; 0) là m®t nghi m của h phương trình.
60t2
Neu x > 0 thì y > 0. Xét hàm so f(t) =
36t2 + 25
, t > 0.
Ta có f(
J
t)
3000t
=
(36t2 + 25)2 > 0, ∀t > 0.
33
Viết đề tài giá sinh viên – ZALO:0973.287.149-TEAMLUANVAN.COM
6
y
√
−
3
ex + √
x
− 2007 = 0
−
Do đó f(t) đong bien trên (0; +∞).
H phương trình được viet lại
x = f(y)
y = f(x)
Tà tính đong bien của f(t) suy ra x = y.
Thay vào h phương trình, ta được
x(36x2 − 60x + 25) = 0 ⇔ x(6x − 5)2 = 0 ⇔ x = 0 ho c x =
5
.
5 5
Với x =
6
⇒ y = x =
6
5 5
V y h phương trình có hai nghi m là (x; y) = (0; 0), ( ; ).
6 6
Bài toán 2.18 (xem [5]). Cháng minh rang h phương trình
ex = 2007 − √
y
y2 1
x
e = 2007
x2 − 1
có đúng hai nghi m thỏa mãn đieu ki n x > 0; y > 0
(Đe thi dự bị 1 - Khoi B - Năm 2007)
Bài giai.
Đ t f(t) = et, g(t)
t
=
t2 − 1
−1
Suy ra g(
J
t) = < 0, ∀ | t |> 1.
(t2 − 1)2
Ta có hàm so f tăng nghiêm ng t trên tàng khoảng xác định, và hàm so g giảm nghiêm
ng t trên tàng khoảng xác định.
H đã cho tương đương với
f(x) + g(y) = 2007
f(y) + g(x) = 2007
⇒ f(x) + g(y) = f(y) + g(x)(∗)
Neu x > y thì
f(x) > f(y)
g(x) < g(y)
Do (∗) suy ra y > x (do g giảm nghiêm ng t) (Vô lí).
Tương tự y > x cũng dan đen vô lí.
Do v y h đã cho tương đương với
x = y
x2 − 1 (2)
√
34
Viết đề tài giá sinh viên – ZALO:0973.287.149-TEAMLUANVAN.COM
∞
x
−
Xét h(x) = ex + √
x
x2 − 1
— 2007, (| x |> 1)
Neu x < −1 thì h(x) < e−1 − 2007 < 0 suy ra h vô nghi m.
Neu x > 1 thì h(x) = ex + √
−3
x
có
x2 − 1
hJ
(x) = ex − (x2 − 1) 2
3
−5
3x
h”(x) = ex + (x2 1)
2
2 .2x = ex + 5
> 0
(x2 − 1)2
Và lim
x→1+
h(x) = + ; lim
→+∞
h(x) = +∞
V y h(x) liên tục và có đo thị là đường cong lõm trên (1; +∞). Do đó đe cháng minh (2)
có hai nghi m dương, ta chỉ can cháng minh ∃x0 > 1 mà h(x) < 0.
Chon x0 = 2 suy ra h(2) = e2
x1 > 1, x2 > 1.
2
+ √
3
− 2007 < 0. Suy ra h(x) = 0 có đúng hai nghi m
Bài toán 2.19 (xem [5]). Giải h phương trình
x2 − 2x + 1 = 2y
y2 − 2y + 1 = 2x
Bài giai.
Ta có 2y = x2 − 2x + 1 = (x + 1)2 ≥ 0. Tương tự x ≥ 0.
Đ t f(t) = t2 − 2t + 1, t ≥ 0, thì f(
J
t) = 2t − 2 = 2(t − 1) nên fđong bien trên (1; +∞) và
nghịch bien trên (0; 1).
Đ t g(t) = 2t, (t ≥ 0) thì g(
J
t) = 2 thì g đong bien trên (1; +∞).
Ta có h phương trình
f(x) = g(y)
g(x) = f(y)
Giả sả x = min {x; y}. Xét x ≤ y.
- Neu x > 1 thì 1 < x ≤ y ⇒ f(x)≤f(y)
⇒ g(y) ≤ g(x) ⇒ y ≤ x. Do đó y = x
Ta có phương trình t2 − 4t + 1 = 0 nên chon nghi m x = y = 2 +
√
3.
- Neu 0 ≤ x ≤ 1 thì f(0) ≥ f(x) ≥ f(1) ⇒ 0 ≤ f(x) ≤ 1
Nên 0 ≤ g(x) ≤ 1 ⇒ 0 ≤ y ≤ 1 ⇒ f(0) ≥ f(y) ≥ f(1) ⇒ 0 ≤ f(y) ≤ 1
⇒ 0 ≤ g(x) ≤ 1 ⇒ 0 ≤ x ≤ 1.
Do đó, xét x ≤ y ⇒ f(x) ≥ f(y) ⇒ g(y) ≥ g(x) ⇒ y ≥ x.
Suy ra x = y.
Ta có phương trình t2 − 4t + 1 = 0 nên chon nghi m x = y = 2 −
√
3.
35
Viết đề tài giá sinh viên – ZALO:0973.287.149-TEAMLUANVAN.COM
√ + √
(1 − y)
√
x − y + x = 2 + (x − y − 1)
√
y (1)
(4x2 + 1)x + (y − 3)
√
5 − 2y = 0
V y h có hai nghi m x = y = 2 +
√
3; x = y = 2 −
√
3.
Bài toán 2.20 (xem [5]). Giải h phương trình
Bài giai.
2y2 − 3x + 6y + 1 = 2
√
x − 2y −
√
4x − 5y − 3 (2)
Đieu ki n x − y ≥ 0; y ≥ 0; x − 2y ≥ 0; 4x − 5y − 3 ≥ 0.
(1)(1 − y)
√
x − y + (x − y − 1) + (y − 1) − (x − y − 1)
√
y = 0
⇔ (1 − y)(
√
x − y − 1) + (x − y − 1)(1 −
√
y) = 0
(1 − y)(x − y − 1)
x − y + 1
(x − y − 1)(1 − y)
= 0
1 + y
1 1
⇔ (1 − y)(x − y − 1)(√
x − y + 1
+
1 +
√
y
) = 0
⇔ (1 − y)(x − y − 1) = 0
⇔ y = 1 ho c x = y + 1
∗ Khi y = 1, (2) ⇔ 9 − 3x = 2
⇔ 9 − 3x = 0 ⇔ x = 3
x − 2 − 4x − 8
∗ Khi x = y + 1
(2) ⇔ 2y2 + 3y − 2 = 2
√
1 − y −
√
1 − 2y
⇔ 2y2 + 3y − 2 =
√
1 − y(∗)
⇔ 2y2 + y = 2(
√
1 − y)2 +
√
1 − y(∗∗)
Xét f(t) = 2t2 + t, t ≥ 0, f(
J
t) = 4t + 1 > 0 nên f(t) đong bien trên [0; +∞).
Do đó (∗∗) ⇔ y =
√
1 − y ⇔ y2 + y − 1 = 0 ⇔ y =
−1 +
√
5
ho c y =
−1 −
√
5
(loại)
Neu y = −1 +
√
5
2
thì x =
1 +
√
5
.
2
1 +
√
5
2 2
−1 +
√
5
V y h đã cho có hai nghi m (x; y) = (3; 1), ( ; )
2 2
Bài toán 2.21 (xem [5]). Giải h phương trình
Bài giai.
3
4x2 + y2 + 2
√
3 − 4x = 7
5
Đieu ki n x ≤
4
; y ≤
2
.
Ta có (4x2 + 1)x + (y − 3)
√
5 − 2y = 0
⇔ (4x2 + 1).2x = (5 − 2y − 1)
√
5 − 2y
√ √
⇔
36
Viết đề tài giá sinh viên – ZALO:0973.287.149-TEAMLUANVAN.COM
2
4
√
∞
⇔ 2x =
√
5 − 2y ⇔
x ≥ 0
x2(1 + y2) + y2(1 + x2) = 4
√
xy
Vì x −
√
1 + x2 < 0 và 1 −
√
1 + y2 <
r
0 nên suy ra y > 0. Do đó x > 0
√ x x x2
Ta có f(t) = 1 − √
1 + t2
− 1 +
x2 x2 x2
Xét hàm so f(t) = (t2 + 1)t với ∀t ∈ R thì f(
J
t) = 3t2 + 1 > 0 nên f đông bien trên R, do
đó (4x2 + 1).2x = (5 − 2y − 1)
√
5 − 2y
⇔ f(2x) = f(
√
5−2y)
y =
5 − 4x2
5 − 4x2
2
The y =
2 vào phương trình sau ta được
4x2 + (
5
− 2x2)2 + 2
√
3 − 4x = 7
3
Với x = 0; x =
4
3
thì không thỏa mãn nên ta chỉ xét khi 0 < x <
4
Thì g(
J
x)
5
= 8x − 8x(
2
)
4
— √
3 −
3
4x
= 4x(4x2 − 3) − √
4
3 − 4x
< 0.
Nên g(x) nghịch bien trên (0; ), mà g 1
( )
2
= 7 nên phương trình sau có nghi m duy nhat
1
x = , suy ra y = 2 : Chon
2
1
V y h có nghi m (x; y) = ( ; 2).
2
Bài toán 2.22 (xem [5]). Giải h phương trình
Bài giai.
Đieu ki n xy ≥ 0.
x2y
√
1 + y2 −
√
1 + x2 = x2y − x
Phương trình thá hai của h tương đương với
x −
√
1 + x2 = x2y(1 − 1 + y2)
Ta có x = 0 không thỏa mãn phương trình
1 1 1 √
Phương trình tương đương với − 1 + = y − y 1 + y2
Xét hàm so f(t) = t − t 1 + t2 trên (0; +∞)
J t2 √ √
Suy ra hàm so nghịch bien trên (0; + ).
1
Phương trình f 1
( )
x
= f(y) ⇔
x
= y ⇔ xy = 1.
Thay vào phương trình thá nhat của h , ta có:
x2(1 +
1
) +
1
(1 + x2) = 4 ⇔ x2 +
1
= 2
t < 0, ∀t ∈ (0; +∞).
37
Viết đề tài giá sinh viên – ZALO:0973.287.149-TEAMLUANVAN.COM
√
− −
2
√
x + 1 +
√
4
x − 1 −
√
y4 + 2 = 0
3. |a| + |b| ≥ |a + b|. Dau ” = ” xảy ra khi a
b ≥ 0.
⇔ (x2 − 1)2 = 0 ⇔ x2 = 1 ⇔ x = ±1.
Ket hợp kieu ki n, suy ra x = y = 1.
V y h đã cho có nghi m (x; y) = (1; 1).
Bài toán 2.23 (xem [5]). Giải h phương trình
Bài giai.
Đieu ki n x ≥ 1.
x2 + 2x(y − 1) + y2 − 6y + 1 = 0
Ta có x2 + 2(y − 1)x + y2 − 6y + 1 = 0
⇔ (x + y − 1)2 − 4y = 0
⇔ 4y = (x + y − 1)2(∗) nên y ≥ 0.
Và
√
x + 1 +
√
4
x 1 y4 + 2 = y
⇔
√
x + 1 +
√
4
x − 1 =
√
(y4 + 1) + 1 +
√
4 (y4 + 1) − 1(∗∗)
Đ t f(t) =
√
t + 1 +
√
4
t − 1 thì f(t) đong bien trên [1; +∞)
Nên (∗∗) ⇔ f(x) = f(y4)+1 ⇔ x = y4 + 1
The vào (∗) ta có 4y = (y4 + y)2 = y8 + 2y5 + y2
⇔ y = 0 ho c y7 + 2y4 + y = 4
⇔ y = 0, x = 1 ho c y = 1, x = 2.
Vì g(y) = y7 + 2y4 + y đong bien trên [0; +∞) và g(1) = 4
V y nghi m của h phương trình là (x; y) = (1; 0) ho c (2; 1).
2.4 Phương pháp sfi dnng bat đang thfíc
2.4.1. Các bat đang thfíc cơ bản thư ng g p
1. |A| ≥ A. Dau ” = ” xảy ra khi A = 0.
2. a2 ≥ 0. Dau ” = ” xảy ra khi a = 0.
4. |a| − |b| ≤ |a − b|. Dau ” = ” xảy ra khi
ab ≥ 0
.
|a| ≥ |b|
5. a2 + b2 ≥ 2ab. Dau ” = ” xảy ra khi a = b.
6. (a + b)2 ≥ 4ab ⇔ ab ≤ (
a + b
)2. Dau ” = ” xảy ra khi a = b.
38
Viết đề tài giá sinh viên – ZALO:0973.287.149-TEAMLUANVAN.COM
≥
bn
√ √ √
√
bn
1 2 n 1 2 n
Dạng 2: |a1b1 + a2b2 + · · · + anbn| ≤
√
(a2 + a2 + · · · + a2 )(b2 + b2 + · · · + b2 )
Dạng 3: a1b1 + a2b2 + · · · + anbn ≤
√
(a2 + a2 + · · · + a2 )(b2 + b2 + · · · + b2 )
1 1 2 2 n n
1 1 4
7.
a
+
b
≥
a + b
(a; b ≥ 0). Dau ” = ” xảy ra khi a = b.
a b
8.
b
+
a
≥ 2(ab > 0). Dau ” = ” xảy ra khi a = b.
9. Bat đȁng thác Caushy (AM - GM)
Với n so thực dương a1; a2; · · · ; an.
Dạng 1:
a1 + a2 + · · · + an
n ≥
√
n
a1 .a2. · · · .an
Dạng 2: a1 + a2 + · · · + an ≥ n.
√
n
a1.a2. · · · .an
Dạng 3: (
a1 + a2 + · · · + an
)n a
n
.a2 . · · · .an
Dau ” = ” xảy ra khi a1 = a2 = · · · = an
11. Bat đȁng thác BCS (Bunhiakovsky).
Với hai b® so thực bat kì: (a1; a2; · · · ; an); (b1; b2; · · · ; bn)
Dạng 1: (a1b1 + a2b2 + · · · + anbn)2 ≤ (a2 + a2 + · · · + a2 )(b2 + b2 + · · · + b2 )
Dau ” = ” xảy ra khi
a1
=
a2
1 2
= · · · =
an
n 1 2 n
b1 b2 bn
Dau ” = ” xảy ra khi
a1
b1
=
a2
b2
1
= · · · =
an
2
> 0.
n 1 2 n
12. Bat đȁng thác BCS dạng c®ng mau (Caushy - Swarchz)
Với ∀xi > 0, i = 1, 2, · · · , n, ta có
a2 a2 a2 (a1 + a2 + · · · + an)2
1
+ 2 + · · · + n ≥
x1 x2 xn x1 + x2 + · · · + xn
13. Bat đȁng thác Minkopsky
Cho hai dãy so thực dương (a1; a2; · · · ; an); (b1; b2; · · · ; bn), ta có
a2 + b2 + a2 + b2 + · · · + a2 + b2 ≥ (a1 + a2 + · · · + an)2 + (b1 + b2 + · · · + bn)2
Dau ” = ” xảy ra khi
a1
b1
=
a2
b2
= · · · =
an
2.4.2. N i dung phương pháp
Phương pháp sả dụng bat đȁng thác đe giải h phương trình là m®t phương pháp
đánh giá mới, thực hi n qua các bước sau:
Bước 1: Tà các phương trình đã cho trong h , ta sả dụng các phép bien đői tương đương
đe làm xuat hi n các bat đȁng thác đã được cháng minh.
Bước 2: Đoi với moi bat đȁng thác, đe cho dau đȁng thác xảy ra tương áng với dau đȁng
thác trong h phương trình, ta tìm được moi liên h của các ȁn. Bieu dien các ȁn trong
h thông qua m®t ȁn.
1
39
Viết đề tài giá sinh viên – ZALO:0973.287.149-TEAMLUANVAN.COM
x1 + x2 + · · · + x2000 = a
2 2000
Bước 3: Thay the các ȁn vàa được bieu dien vào m®t phương trình trong h , giải phương
trình m®t ȁn roi tìm các ȁn còn lại qua ȁn vàa tìm được.
2.4.3. Bài t p sfi dnng tính chat bat đang thfíc giải h phương
trình phi tuyen
Bài toán 2.24 (xem [2]). Giải h phương trình:
x6 + y8 + z10 = 1 (1)
x2013 + y2015 + z2017 = 1 (2)
Bài giai.
Tà (1), ta suy ra −1 ≤ x, y, z ≤ 1.
Tà đó, ta có:
x6 − x2013 = x6(1 − x2007) ≥ 0 ⇔ x6 ≥ x2013.
Dau ” = ” xảy ra ⇔ x = 0 ho c | x |= 1.
y8 − y2015 = y8(1 − y2007) ≥ 0 ⇔ y8 ≥ y2015
Dau ” = ” xảy ra ⇔ y = 0 ho c | y |= 1.
z10 − z2017 = z10(1 − z2007) ≥ 0 ⇔ z10 ≥ z2015.
Dau ” = ” xảy ra ⇔ z = 0 ho c | z |= 1.
C®ng tàng ve của các bat đȁng thác trên, ta suy ra
1 = x6 + y8 + z10 ≥ x2013 + y2015 + z2017 = 1
Do đó dau đȁng thác xảy ra, tác là:
x6(1 − x2007) = 0
y8(1 − y2007) = 0
z10(1 − z2007) = 0
Ket hợp với (1) và (2), ta suy ra
(x; y; z) = (1; 0; 0), (0; 1; 0), (0; 0; 1).
Bài toán 2.25 (xem [5]). Giải h phương trình:
x2 + x2 + · · · + x2
= a2
1 2
. . .
2000
x2000 + x2000 + · · · + x2000 = a2000
1
40
Viết đề tài giá sinh viên – ZALO:0973.287.149-TEAMLUANVAN.COM
√ √
−
−
√
a a a a a a
a a a a a a
√
Bài giai.
∗ Neu a = 0, tà phương trình thá hai, ta suy ra x1 = x2 = . . . = x2000 = 0.
∗ Neu a /= 0, tà phương trình thá hai, ta suy ra
(
x1
)2 + (
x2
)2 + (
x3
)2 + · · · + (
x2000
)2 = 1
a x1
a x2
a x2000
a
⇒|
a
|≤ 1; |
a
|≤ 1; . . . ; |
a
|≤ 1
⇒ (
x1
)3 ≤ (
x1
)2; (
x2
)3 ≤ (
x2
)2; . . . ; (
x2000
)3 ≤ (
x2000
)2.
C®ng tàng ve của các bat phương trình trên, ta có:
1 = (
x1
)3 + (
x2
)3 + · · · + (
x2000
)3 ≤ (
x1
)2 + (
x2
)2 + · · · + (
x2000
)2 = 1.
Dau đȁng thác xảy ra khi
(
x1
)2 = (
x1
)3, . . . , (
x2000
)2 = (
x2000
)3.
a a a a
H có các nghi m:
(x1; x2; . . . ; x2000) = (a; 0; 0; . . . ; 0), (0; a; 0; . . . ; 0), . . . , (0; 0; 0; . . . ; a).
Bài toán 2.26 (xem [5]). Giải h phương trình
√
1
1 + 2x2
1
+
1 + 2y2
2
=
1 + 2xy
x(1 2x) + y(1 2y) =
2
9
(Đe thi HSG Quoc Gia năm 2009)
Bài giai.
1
Đieu ki n x, y ∈ [0;
2
].
Ta cháng minh bat đȁng thác
1 1 2
√
1 + 2x2
+ √
1 + 2y2
≤ √
1 + 2xy
⇔
1 + 2x2
+
1 + 2y2
+ √
1 + 2x2
√
1 + 2y2
≤
1 + 2xy
Ta có x, y ∈ [0;
2
] nên
√
1 + 2x2
√
1 + 2y2
≤
1 + 2xy
⇔
√
1 + 2x2 1 + 2y2 ≥ 1 + 2xy.
⇔ (1 + 2x2)(1 + 2y2) ≥ (1 + 2xy)2
⇔ (x − y)2 ≥ 0 (Luôn đúng)
1 1 2
Và
1 + 2x2
+
1 + 2y2
≤
1 + 2xy
(1 − 4xy)(x − y)2
⇔
(1 + 2xy)(1 + 2x2)(1 + 2y2)
≥ 0 : Đúng
Do dau đȁng thác xảy ra nên x = y, ta có phương trình
9
√
x(1 − 2x) = 1 ⇔ 162x2 − 81x + 1 = 0
1 1 2 4
1
2 2
√
41
Viết đề tài giá sinh viên – ZALO:0973.287.149-TEAMLUANVAN.COM
⇔
1 1 1 x y z
x + y2 + z3 = 14 (1)
x =
81 ±
√
5913
324
V y h phương√trình đã cho√có hai nghi m√ √
81 +
(x; y) = (
5913 81 +
; 5913
), (
81 − 5913
;
81 − 5913
).
324 324 324 324
Bài toán 2.27 (xem [5]). Giải h phương trình
x + y + z = 1
x4 + y4 + z4 = xyz
.
Bài giai.
Ta có x4 + y4 ≥ 2x2y2
z4 + y4 ≥ 2z2y2
x4 + z4 ≥ 2x2z2
⇒ x4 + y4 + z4 ≥ x2y2 + y2z2 + x2z2
M t khác x2y2 + x2z2 ≥ 2x2yz
x2y2 + y2z2 ≥ 2y2xz
z2y2 + x2z2 ≥ 2z2yx
⇔ x2y2 + x2z2 + y2z2 ≥ xyz(x + y + z) = xyz
Suy ra x4 + y4 + z4 ≥ x2y2 + y2z2 + x2z2 ≥ xyz
Dau đȁng thác xảy ra x = y = z và x + y + z = 1
1
Suy ra x = y = z =
3 1 1 1
V y h phương trình có nghi m (x; y; z) = ( ; ; ).
3 3 3
Bài toán 2.28 (xem [5]). Giải h phương trình
(
2x
+
3y
+
6z
)(
2
+
3
+
6
) = 1 (2)
.
(x, y, z là các so dương)
Bài giai.
3 2 1
Phương trình (2) ⇔ (
x
+
y
+
z
)(3x + 2y + z) = 36
x y x z z y
⇔ 6(
y
+
x
) + 3(
z
+
x
) + 2(
y
+
z
) = 22
Vì x, y, z > 0
⇔
42
Viết đề tài giá sinh viên – ZALO:0973.287.149-TEAMLUANVAN.COM
4 4 4
x
+ y = 2a
1
+
4
≤ 3
x y
x y x z z y
⇒ 6(
y
+
x
) ≥ 12; 3(
z
+
x
) ≥ 6; 2(
y
+
z
) ≥ 4
3 2 1
⇒ (
x
+
y
+
z
)(3x + 2y + z) = 36
x y x z z y
⇔ 6(
y
+
x
) + 3(
z
+
x
) + 2(
y
+
z
) ≥ 22
Dau đȁng thác xảy ra ⇔ x = y = z
Thay ⇔ x = y = z vào (1), ta có
x3 + x2 + x − 14 = 0
⇔ (x − 2)(x2 + 3x + 7) = 0
⇔ x − 2 = 0 ( Vì x2 + 3x + 7 = (x +
3
)2 +
19
≥
19
> 0)
⇔ x = 2.
2 4 4
V y h có nghi m duy nhat x = y = z = 2.
Bài toán 2.29 (xem [5]). Tìm tat cả các so dương x, y thỏa mãn
x + y = 3
(Thi vào 10 Năng khieu Toán - Tin, ĐHQG TP.HCM 1997 - 1998 Vòng 2)
Bài giai.
1 4
Tà giả thiet suy ra (
x
+
y
)(x + y) ≤ 9
Áp dụng bat đȁng thác Schwartz, ta có
1 4 √ 2 √ 2
√ 2 √ 2 1 √ 2 √ 2
( + )(x + y) = [(
x y
x) + ( y) ][( x) + ( y) ] ≥ (√
x x + √
y
y) = 9
1 4
⇒ 9 ≤ (
x
+
y
)(x + y) ≤ 9
x y
Dau đȁng thác xảy ra ⇔
1
=
2
và x + y = 3 ⇔ x = 1; y = 2
V y h đã cho có nghi m (x; y) = (1; 2).
Bài toán 2.30 (xem [5]). Giải h phương trình với tham so a ≥ 0
x + y + xy = a2 + 2a
(Thi chon đ®i tuyen toán 9 Thanh Hóa 1995 - 1996)
Bài giai.
Ta có x4 + y4 ≥ 2x2y2 ⇔ 2a4 ≥ 2x2y2 ⇔ (xy)2 ≤ a4 ⇔ xy ≤ a2
43
Viết đề tài giá sinh viên – ZALO:0973.287.149-TEAMLUANVAN.COM
1
1
x1 + x2 + · · · + xn = 9 (1)
⇒ n2 ≤ 9 ⇒ n ≤ 3 Màn ∈ N∗
⇒ n = 1, 2, 3
∗ Neu n = 2, ta có h
x1 + x2 = 9
⇔
x1 + x2 = 9
1 2
2 2
2
2
2
(x + y)2 ≤ 2(x2 + y2) mà (x2 + y2)2 ≤ 2(x4 + y4) = 4a4
⇒ (x + y)2 ≤ 4a2 ⇒ x + y ≤ 2a
Suy ra x + y + xy ≤ a2 + 2a
Dau đȁng thác xảy ra ⇔ x = y = a
V y h có nghi m duy nhat (x; y) = (a; a).
Bài toán 2.31 (xem [5]). Tìm tat cả các so dương x1, x2, . . . , xn thỏa mãn h phương
trình
(n là so nguyên dương)
Bài giai.
x1
1 1
+
x
+ · · · +
x
= 1 (2)
1
Tà (1) và (2) suy ra (x1 + x2 + · · · + xn)(
x
1 1
+
x
+ · · · +
x
) = 9
M t khác
n2 = (1 + 1 + . . . + 1)2 = (
√
x
1
1
1 √
x1
1
+
√
x
1
2 √
x2
1
+ . . . +
√
x
1
n √
xn
≤ ((x1 + x2 + · · · + xn)(
x
+
x
+ · · · +
x
) = 9
∗ Neu n = 3, ta có h
x1 + x2 + x3 = 9
⇒ (x + x 1
+ x )( 1 1
+ + ) = 9
1 1 1
+ + = 1
x1 x2 x3
1 2 3
x1
x2 x3
1
Mà (x1 + x2 + x3)(
1
1 1
+ + )
x2 x3
= [(x )2 + (
√
x )2 + (
√
x )2][(
1
)2 + (
1
)2 + (
1
)2] ≥ (1 + 1 + 1)2 = 9
1 2 3 √
x1
√
x2
√
x3
D
√
au đȁng
√thác xả
√
y ra khi và chỉ khi
x1
1
√
x1
x1
=
1
√
x1
x1
=
1
√
x1
⇔ x1 = x2 = x3
Mà x1 + x2 + x3 = 9 nên x1 = x2 = x3 = 3.
1 1
+ = 1
x1 x2
x1x2 = 9
Suy ra x1, x2 là các
√nghi m c
√
ủa phương√
trình t2 −
√
9t + 9 = 0
⇔ (x ; x ) = (
9 − 45
;
9 + 45
), (
9 + 45
;
9 − 45
)
∗ Neu n = 1, ta có h
x1 = 9
Vô nghi m
= 1
x1
V y h đã cho có nghi m x1 = x2 = x3 = 3
n
1 n
x
2 2
)2
1 n
44
Viết đề tài giá sinh viên – ZALO:0973.287.149-TEAMLUANVAN.COM
2x2000 = x1 +
x
2
2
1
Ho c (x ; x ) = (
9 −
√
45
;
9 +
√
45
), (
9 +
√
45
;
9 −
√
45
).
1 2 2 2 2 2
Bài toán 2.32 (xem [5]). Giải h phương trình 2000 ȁn so:
2x1
1
= x2 +
2
1
(1)
2x2 = x3 +
x3
(2)
. . .
1
2x1999 = x2000 + (1999)
1
x2000
Bài giai.
Đieu ki n : xi 0, i = 1, . . . , 2000
Tà (1) ⇒ x1, x2 cùng dau.
Tà (2) ⇒ x2, x3 cùng dau.
. . .
Tà (1999) ⇒ x1999, x2000 cùng dau.
Tà (2000) ⇒ x1, x2000 cùng dau.
Như v y các ȁn so xi cùng dau.
M t khác, neu (x1; x2; . . . ; x2000) là m®t nghi m thì (−x1; −x2; . . . ; −x2000) cũng là nghi m,
do đó ta chỉ can xét x1; x2; . . . ; x2000 > 0.
1
Khi đó: 2x1 = x2 +
2
1
1
≥ 2 ⇒ x1 ≥ 1 ⇒
x
1
≤ 1.
2x2 = x3 +
3
≥ 2 ⇒ x2 ≥ 1 ⇒
x
1
≤ 1.
Tương tự ta có 1
x2000
1
⇒
x1
1
+
x
+ · · · +
x
1
2000
≤ x1 + x2 + . . . + x2000.
M t khác neu c®ng tàng ve 2000 phương trình của h , ta có
1
x1 + x2 + . . . + x2000 =
1
1
+
x
+ · · · +
x
1
2000
Dau đȁng thác xảy ra khi và chỉ khi x1 = x2 = . . . = x2000 = 1.
Tóm lại h có hai nghi m (x1; x2; . . . ; x2000) = (1; 1; . . . ; 1), (−1; −1; . . . ; −1)
x 1
x
(2000)
x
x 2
≤
45
Viết đề tài giá sinh viên – ZALO:0973.287.149-TEAMLUANVAN.COM
x + 1
2
⇒ ≥
2 = y
y2
Bài toán 2.33 (xem [5]). Giải h phương trình
2x2
y2 + 1
= z
4z4
Bài giai.
Xét hai trường hợp sau:
∗ Trường hợp 1
z6 + z4 + z2 + 1
= x
Trong ba so x, y, z có ít nhat m®t so bang 0
Giả sả x = 0 ⇒ y = 0 ⇒ z = 0.
V y (x; y; z) = (0; 0; 0) là m®t nghi m.
∗ Trường hợp 2
Không có so nào bang 0
2x2
Vì y =
x2 + 1
> 0 ⇒ y > 0 ⇒ z > 0 ⇒ x > 0
V y x, y, z > 0
2x 2x2
Dě thay ≤ 1 ⇒ ≤ x ⇒ y ≤ x
x2 + 1 x2 + 1
Ta lại có y4 − 2y2 + 1 ≥ 0 ⇒ y4 + y2 + 1 ≥ 3y2
3y2 3y3
⇒
y4 + y2 + 1
≤ 1 ⇒
y4 + y2 + 1
≤ y ⇒ z ≤ y
Ta có
z6 + z4 + z2 + 1 − 4z3 = (z6 − 2z3 + 1) + z2(z2 − 2z + 1) = (z3 − 1)2 + z2(z − 1)2 ≥ 0
z6 + z4 + z2 + 1 4z3
4z3
⇒
z6 + z4 + z2 + 1
≤ 1
4z4
⇒
z6 + z4 + z2 + 1
≤ z hay x ≤ z
Suy ra x ≤ z ≤ y ≤ x ⇒ x = y = z
Thay vào phương trình (1) ta có
x(x − 1)2 = 0 ⇒ x = y = z = 0( Không thỏa mãn đk) ho c x = y = z = 1 (Thỏa mãn)
The x = y = z = 1 vào h phương trình thay nghi m đúng h phương trình.
V y h đã cho có nghi m (x; y; z) = (0; 0; 0), (1; 1; 1).
Bài toán 2.34 (xem [5]). Cho x, y, z là các so thực thỏa mãn đieu ki n
x + y + z = 5
x2 + y2 + z2 = 9
46
Viết đề tài giá sinh viên – ZALO:0973.287.149-TEAMLUANVAN.COM
3
3 2
x + 2xy + 1
2
y
= 0
⇒
7
Cháng minh rang 1 ≤ x, y, z ≤
3
(Thi vào 10 chuyên Toán - Tin ĐHTH TP. HCM 23/ 06/ 1996)
Bài giai.
Tà x + y + z = 5 ⇒ y + z = 5 − x ⇒ (y + z)2 = (5 − x)2
Ta có 2(y2 + z2) ≥ (y + z)2 ⇒ 2(9 − x2) ≥ (5 − x)2
⇒ 3x2 − 10x + 7 ≤ 0 ⇔ (x − 1)(3x − 7) ≤ 0 ⇔ 1 ≤ x ≤
7
.
7
Vì x, y, z có vai trò như nhau, cháng minh tương tự, ta có 1 ≤ x, y, z ≤
3
.
2.5 Phoi h p nhieu phương pháp giải h phương trình
2.5.1. N i dung phương pháp
Đoi với nhieu h phương trình phác tạp mà chỉ dùng m®t trong bon phương pháp nêu
trên không đưa ra được ket quả, ta can phoi hợp nhieu phương pháp lại với nhau đe đưa
ra lời giải cho bài toán đó.
2.5.2. Bài t p áp dnng nhieu phương pháp giải h phương trình
Bài toán 2.35 (xem [2]). Giải h phương trình:
x2 + 8y2 = 12
(Thi lớp 10 chuyên toán ĐHTH Hà N®i 2008 - 2009)
Bài giai.
Nh n thay neu h có nghi m (x; y) thì y /= 0 vì neu y = 0 thì
x2 = 12
x3 = 0
(Vô lý).
Khi y =
/ 0, thay 12 = x2 + 8y2 vào phương trình thá hai, ta có:
x3 + 2xy2 + (x2 + 8y2)y = 0
⇔ x3 + 2xy2 + x2y + 8y3
x 3
⇔ (
y
)
x x
+ ( )2 + 2
y y
+ 8 = 0.
Đ t
x
= t t3 + t2 + 2t + 8 = 0
y
⇔ (t + 2)(t2 − t − 4) = 0
47
Viết đề tài giá sinh viên – ZALO:0973.287.149-TEAMLUANVAN.COM
5
2 4
16
2
r
r
16
4
⇔ u
⇔ (t + 2)[(t −
1
)2 +
15
] = 0
⇔ t = −2 ho c (t −
1
)2 +
15
= 0 (Vô lý).
x 2 4
Với t = −2 ⇒
y
= −2 ⇒ x = −2y. Thay vào phương trình thá nhat, ta có
4y2 + 8y2 = 12 ⇔ y2 = 1 ⇔ y = ±1.
Khi y = 1 ⇒ x = −2
Khi y = −1 ⇒ x = 2
H phương trình (2.9) có hai nghi m (x; y) = (2; −1), (−2; 1).
Bài toán 2.36 (xem [2]). Giải h phương trình:
x2 + y + x3y + xy2 + xy =
5
4
5
x4 + y2 + xy(1 + 2x) =
4
(Đe TS Đại hoc, Cao đȁng 2008 - Khoi A)
Bài giai.
H đã cho tương đương với
x2 + y + xy + xy(x2 + y) =
5
4 (∗)
Đ t
u = x2 + y
(x2 + y)2 + xy =
4
. H (∗) trở thành
v = xy
u + v + uv =
−5
v =
−5
− u2
4
u2 + v =
−5
4
4
u3 + u2 + = 0
4
⇔ u = 0; v =
−5
ho c u =
−1
, v =
−3
.
٨ Với u = 0; v =
−5
, ta có h phương trình:
4
x2 + y = 0
xy =
−5
4
x = 3 −5
⇔
25
y = − 3
٨ Với u =
−1
, v =
−3
, ta có h phương trình:
3
2
1
2
x2 − + = 0 2x3 + x − 3 = 0 x = 1
2x 2
y =
−3
2x
y =
−3
2x
y =
−3
2
V y h phương trình đã cho có hai nghi m :
(x; y) = (
r
3
−5
; −
r
3 25
), (1;
−3
).
⇔
⇔
4 2 2
4
48
Viết đề tài giá sinh viên – ZALO:0973.287.149-TEAMLUANVAN.COM
/
1 1 7
xyzt = 648
1
+
1
=
5
Suy ra
z t 18
126
1 1 1 1
x + y + z = 3
1 1 7 7 ( Vô nghi m)
Bài toán 2.37 (xem [5]). Giải h phương trình
x + y + z + t = 22
+ =
x y 12
Bài giai.
Đieu khi n x, y, z, t = 0
7
z t 18
5
Ta có x + y = xy; z + t = zt
12 18
7 5 7 5
⇒ (x + y)(z + t) =
12
xy.
18
zt =
Và (x + y) + (z + t) = 22
.
12 18
.648 = 105
Suy ra (x + y), (z + t) là hai nghi m của phương trình X2 − 22X + 105 = 0
⇔ X1 = 15, X2 = 7
Xét hai trường hợp:
∗ Trường hợp 1
x + y = 7
z + t = 15
x + y = 7
xy = 12 (x; y) = (3; 4), (4; 3)
+ =
x y 12
1
+
1
=
5
z + t = 15
zt = 54
(z; t) = (6; 9), (9; 6)
∗ Trường hợp 2
x + y = 15 x + y = 15
z + t = 7
xy =
180
+ =
x y 12
1
+
1
=
5
z + t = 7
zt =
5
V y h đã cho có bon nghi m
(x; y; z; t) = (3; 4; 6; 9), (3; 4; 9; 6), (4; 3; 6; 9), (4; 3; 9; 6).
Bài toán 2.38 (xem [5]). a, Cho x, y, z là các so thực thỏa mãn
x
+
y
+
z
=
3
7
1
1 ⇔
⇔
z t 18
49
Cháng minh rang có ít nhat 1 trong 3 so x, y, z = 3
Viết đề tài giá sinh viên – ZALO:0973.287.149-TEAMLUANVAN.COM
x + y + z = 3
∗Trường hợp 1
y = −z
y + 2z
1 1
∗Trường hợp 3
x = −y
y + 2.3 = 1
y + 2z2 = 1
b, Áp dụng câu a, giải h phương trình
1 1 1 1
+ + =
x y z 3
Bài giai.
a, Tà giả thiet, suy ra
1 1 1
+ +
x y z
1
=
x + y + y
1 1 1
⇔ (
x
+
y
)(
z
1
—
x + y + z
) ⇔ (x + y)[z(x + y + z) + xy] = 0
⇔ (x + y)(y + z)(x + x) = 0
⇔ x + y = 0 ⇒ z = 3 ho c y + z = 0 ⇒ x = 3 ho c x + z = 0 ⇒ y = 3
b, Tà hai phương trình đau suy ra x = 3 ho c y = 3 ho c z = 3, the vào h phương trình,
ta có
x = 3
y + z = 0
⇔
x = 3
⇔
x = 3
+ = 0
y z
2
2y 2
− y − 1 = 0
1
y = 1 ho c y =
2
⇒ (x; y; z) = (3; 1; −1), (3;
−1
;
1
)
∗ Trường hợp 2
y = 3
x + z = 0
y = 3
⇔
x = −z
+ = 0
x z
3 + 2z2 = 1
+ = 0
x z
2z2 = −2 ( Vô lí)
z = 3
x + y = 0
⇔
z = 3
⇔
z = 3
+ = 0
x y 2
⇒ (x; y; z) = (17; −17; 3)
1
1
Suy ra h vô nghi m.
1
1
1
1
2
y + 2z2 = 1
y = −z
= 0
x = −y
y = −17
2
50
Viết đề tài giá sinh viên – ZALO:0973.287.149-TEAMLUANVAN.COM
−
V y h đã cho có 3 nghi m
(x; y; z) = (3; 1; 1), (3;
−1
;
2
1
2
), (17; −17; 3).
Bài toán 2.39 (Xem [5]). Giải h phương trình
ux3 + vy3 = 14 (1)
ux2 + vy2 = 5 (2)
ux + vy = 2 (3)
u + v = 1 (4)
(Thi HSG TP. HCM Ngày 17/ 12/ 1994 Vòng 2)
Bài giai. Tà (4) suy ra v = 1 − u, the vào (1)
xu3 + (1 − u)y3 = 14
⇒ u(x3 − y3) = 14 − y3
⇒ u(x − y)(x2 + xy + y2) = 14 − y3(5)
Tương tự
Neu the v = 1 − u vào (2), ta có
u(x − y)(x + y) = 5 − y2(6)
The v = 1 − u vào (3), ta có
u(x − y) = 2 − y(7)
Tà (5) và (7) suy ra (2 − y)(x2 + xy + y2) = 14 − y3
Tà (6) và (7) suy ra (2 − y)(x + y) = 5 − y2
Khai trien phương trình cuoi cùng, ta có
2(x + y)2 − xy(x + y) − 2xy − 14 = 0
2(x + y) − xy − 5 = 0
Đ t x + y = a, xy = b
2a2 − ab − 2b − 14 = 0
2a − b = 5
⇔
a = 4
b = 3
⇔
x + y = 4
xy = 3
⇒ (x; y) = (3; 1), (1; 3)
The x và y ta tìm được các nghi m của h
1
(x; y; u; v) = (3; 1; ;
2
1 1 1
), (1; 3; ; )
2 2 2
Bài toán 2.40 (xem [5]). Giải h phương trình
x4 + x2y2 + y4 = 482
x2 + xy + y2 = 37
⇒
51
Viết đề tài giá sinh viên – ZALO:0973.287.149-TEAMLUANVAN.COM
∗
xy = 12
Bài giai.
Đ t S = x + y; P = xy
Ta có h phương trình
vào (1)
S4 − 4PS2 + 3P 2 = 481 (1)
S2 − P = 37 (2)
Tà (2) suy ra S2 = P + 37 the
Suy ra (P + 37)2 − 4P(P + 37) + 3P 2 = 481
⇔ P 2 + 74P + 1369 − (4P 2+ ⇔ 148P) + 3P 2 = 481
⇔ −74P = −888
⇔ P = 12
⇔ S = ±7
Xét hai trường hợp
Trường hợp 1
x + y = 7
xy = 12
Nên x, y là nghi m của phương trình X2 − 7X + 12 = 0
⇔ X1 = 3 ho c X2 = 4
∗ Trường hợp 2
x + y = −7
Nên x, y là nghi m của phương trình X2 + 7X + 12 = 0
⇔ X1 = −3 ho c X2 = −4
V y h phương trình có 4 nghi m (x; y) = (3; 4), (4; 3), (−3; −4), (−4, −3).
Bài toán 2.41 (xem [5]). Giải h phương trình
x − y2 − yz − z = 0 (1)
x − y − y2 − z2 = 0 (2)
x + y − y3 − z = 0 (3)
(Thi HSG Qu n I 1995 - 1996 Vòng 1)
Bài giai.
Lay (2) − (1), ta được :
yz − y + z − z2 = 0 ⇔ (z − 1)(y − z) = 0
Do đó h đã cho tương đương với :
(z − 1)(y − z) = 0
x − y − y2 − z2 = 0 ⇔
z − 1 = 0
x − y − y2 − z2 = 0 (4) ho c
y − z = 0
x − y − y2 − z2 = 0 (5)
x + y − y3 − z = 0 x + y − y3 − z = 0 x + y − y3 − z = 0
52
Viết đề tài giá sinh viên – ZALO:0973.287.149-TEAMLUANVAN.COM
− −
z = 1
y = 0; y = −1; y = 2
1
2 3
Giải h (4):
z − 1 = 0
x − y − y2 − z2 = 0 ⇔
x + y y3 z = 0
z = 1
z = 1
x − y − y2 − 1 = 0
x + y − y3 − 1 = 0
⇔ y3 − y2 − 2y = 0
x = y3 − y + 1
⇔ y(y2 − y − 2) = 0
x = y3 − y + 1
z = 1
⇔ y1 = 0; y2 = −1; y3 = 2
x = y3 − y + 1
z = 1
x1 = 1; x2 = 1; x3 = 7
Giải h (5):
z = y
x − y − 2y2 = 0 ⇔
x − y3 = 0
z = y
y3 − 2y2 − y = 0
x = y3
z = y
⇔ y(y2 − 2y − 1) = 0
x = y3
z = y
⇔ y1 = 0; y2 = 1 −
√
2; 1 +
√
2
x = y3
⇔ (x; y; z) = (0; 0; 0), ((1 +
√
2)3; 1 +
√
2; 1 +
√
2), ((1 −
√
2)3; 1 −
√
2; 1 −
√
2)
V y h phương trình có sáu nghi m
(x; y; z) = (1; 0; 1), (1; −1; 1), (7; 2; 1), (0; 0; 0),
((1 +
√
2)3; 1 +
√
2; 1 +
√
2), ((1 −
√
2)3; 1 −
√
2; 1 −
√
2)
Bài toán 2.42 (xem [5]). Giải h phương trình
x + y + z = 6 (1)
x2 + y2 + z2 = 18 (2)
√
x +
√
y +
√
z = 4 (3)
(Thi HSG Toàn Quoc 1996 - 1997 Bảng A)
Bài giai.
Tà (1) ⇒ 36 = (x + y + z)2 ⇔ 36 = x2 + y2 + z2 + 2(xy + xz + yz)
⇔ 36 = 18 + 2(xy + xz + yz) ⇔ xy + xz + yz = 9
Tà (3) ⇒ 16 = (
√
x +
√
y +
√
z)2 ⇔ x + y + z + 2(
√
xy +
√
xz +
√
yz) = 16
⇔
53
Viết đề tài giá sinh viên – ZALO:0973.287.149-TEAMLUANVAN.COM
⇔
⇔
√
xy +
√
xz +
√
yz = 5 ⇔ (
√
xy +
√
xz +
√
yz)2 = 25
⇔ xy + yz + xz + 2(
√
xy2z +
√
xyz2 +
√
x2yz) = 25
⇔
√
xyz(
√
x +
√
y +
√
z) = 8 ⇔ xyz = 4
V y h phương trình đã cho tương đương với:
4
Tà (5) ⇒ yz =
x
(De thay x, y, z > 0).
(4) ⇔ xy + yz + xz + x2 = 9 + x2
⇔ x(x + y + z) + yz = 9 + x2
6x +
4
= 9 + x2
x
⇔ x3 − 6x2 + 9x − 4 = 0
⇔ (x − 1)2(x − 4) = 0
⇔ x = 1 ho c x = 4
The vào, ta suy ra h có các nghi m
(x; y; z) = (1; 1; 4), (1; 4; 1), (4; 1; 1).
x + y + z = 6 (1)
xy + yz + xz = 9 (4)
xyz = 4 (5)
Bài toán 2.43 (xem [5]). Giải h phương trình
x2 + y2 + xy = 37 (1)
x2 + z2 + xz = 28 (2)
y2 + z2 + yz = 19 (3)
(Thi HSG Toàn Quoc 1994 - 1995)
Bài giai.
Lay phương trình (1) − (2), ta được :
y2 − z2 + xy − xz = 9 ⇔ (y − z)(x + y + z) = 9
Lay phương trình (2) − (3), ta được :
x2 − y2 + xz − yz = 9 ⇔ (x − y)(x + y + z) = 9
Suy ra : (y − z)(x + y + z) = (x − y)(x + y + z).
Mà x + y + z /= 0 ⇒ y − z = x − y
x + z = 2y x + y + z = 3y
3
Do đó : (x − y)(x + y + z) = 9 ⇔ (x − y)3y = 9 ⇔ (x − y)y = 3 ⇔ x = y +
y
.
Thay vào phương trình (1), ta có:
⇔
54
Viết đề tài giá sinh viên – ZALO:0973.287.149-TEAMLUANVAN.COM
⇒
y y
3 3 3 3 3 3
(y +
1
)2 + y2 + (y +
1
)y = 37 ⇔ 3y4 − 28y2 + 9 = 0
y2 = 9 ho c y2 =
1
3
⇔ y1 = 3; y2 = −3; y3 =
√
3
3
; y4 =
−
√
3
3
- Với y1 = 3 ⇒ x1 = 4, z1 = 2
- Với y2 = −
√3 ⇒ x2 = −4,√
z2 = −2 √
- Với y3
3
=
3√
⇒ x3
10 3
= ,z3
3 =
−8 3
3
- Với y4 =
− 3
x
3 =
−10
√
3
, z
3
8
√
3
=
3
V y h phương trình đã cho có bon √
nghi√
m √ √ √ √
(x; y; z) = (4; 2; 3), (−4; −3; −2), (
10 3
;
3
,
−8 3
), (
−10 3
;
− 3
;
8 3
)
⇔
4
3
55
Viết đề tài giá sinh viên – ZALO:0973.287.149-TEAMLUANVAN.COM
Chương 3
M t so fíng dnng của h phương
trình phi tuyen
3.1 Ứng dnng của h phương trình đa thfíc trong giải
các bài toán cfic trị và chfíng minh bat đang thfíc.
Bài toán 3.1 (xem [5]). Cho phương trình
x3 − 2002x2 + 2001bx − 2000a = 0
Tìm giá trị lớn nhat của a sao cho ∃b đe phương trình trên có ba nghi m trên [−2002; 2002].
Bài giai.
Giả sả x1, x2, x3 là ba nghi m của phương trình. Khi đó theo định lí Viete cho phương
trình b c ba, ta có:
x1 + x2 + x3 = 2002
x1x2 + x2x3 + x1x3 = 2001b
x1x2x3 = 2000a
Xảy ra các trường hợp sau :
+ Neu x2 < 0 ho c x3 < 0, khi đó x1 + x2 + x3 < x3 ≤ 2002 (Mâu thuan (3.1))
+ Neu x1 < 0 ≤ x2 ≤ x3, theo bat đȁng thác Caushy, ta có
√ √ 20023
2002 = x1 + x2 + x3 ≥ 3 3
x1x2x3 = 3 3
2000a ⇒ a ≤
2002
54000
20023
Dau ” = ” xảy ra khi x1 = x2 = x3 =
Khi đó phương trình đã cho trở thành
và khi ay b =
3
.
6003
56
Viết đề tài giá sinh viên – ZALO:0973.287.149-TEAMLUANVAN.COM
2 2 2
x + 3y + 4z
=101 (2)
− ⇔
x2 − y2 = 21 (1)
x3 − 2002x2 +
20022
2
x −
20022
33 = 0 ⇔ (x −
2002
)3 = 0
3
Phương trình trên có ba nghi m x1 = x2 = x3 =
20023
V y amax =
54000
là giá trị lớn nhat can tìm.
2002
3
∈ [−2002; 2002].
Bài toán 3.2 (xem [6]). Cho bieu thác M = x2 + y2 + 2z2 + t2 Hãy tìm giá trị nhỏ nhat
của M và các giá trị nguyên không âm tương áng của x, y, z, t biet chúng thỏa mãn đong
thời:
x2 − y2 + t2 = 21 (1)
(Thi HSG Quoc Gia 1985 - 1986 Bảng A)
Bài giai.
C®ng (1) và (2), ta có: 2x2 + 2y2 + 4z2 + t2 = 122 ⇔ 2(x2 + y2 + 2z2 + t2) − t2 = 122
⇒ 2M = 122 + t2 ≥ 122 ⇒ M ≥ 61 ⇒ min M = 61
Khi t = 0 ⇒
x2 + 3y2 + 4z2 = 101 (2)
Vì x, y nguyên, không âm nên
(x y)(x + y) = 21
x − y = 1
x + y = 21
⇔ (x; y) = (11; 0), (5; 2).
ho c
x − y = 3
x + y = 7
∗ Trường hợp 1: Thay x = 11, y = 0 vào (2), ta có 112 + 0 + 4z2 = 101
⇒ 4z2 = −20( Vô lí)
∗ Trường hợp 2: Thay x = 5, y = 2 vào (2), ta có 52 + 3.22 + 4z2 = 101
⇒ 4z2 = 64 ⇒ z = ±4
V y giá trị nhỏ nhat của M là 61, khi (x, y, z, t) = (5; 2; 4; 0), (5; 2; −4; 0).
Bài toán 3.3 (xem [6]). Cho x2 + y2 + z2 ≤ 27. Tìm giá trị lớn nhat và nhỏ nhat của
bieu thác:
P = x + y + z + xy + yz + xz
Bài giai.
Ta có (x + y + z)2 = x2 + y2 + z2 + 2(xy + yz + xz) ≤ 3(x2 + y2 + z2) ≤ 81
57
Viết đề tài giá sinh viên – ZALO:0973.287.149-TEAMLUANVAN.COM
−
2 2 2
4
≤
r
4
4
1
4 4 4
⇒ x + y + z ≤ 9
Mà xy + yz + xz ≤ x2 + y2 + z2 ≤ 27 ⇒ x + y + z + xy + yz + xz ≤ 36
V y Max(P ) = 36 khi x = y = z = 3
Đ t A = x + y + z; B = x2 + y2 + z2
⇒ P = A +
A2 B
=
2
B + 1
(A + 1)2
2
−
B + 1
2
≥ −
B + 1
.
2
Vì B ≤ 27 ⇒ −
2
≥ −14 ⇒ P ≥ −14.
Min(P) = 14 Khi
x + y + z = −1
x + y + z = 27
Hay x = −
√
13, y =
√
13, z = −1.
Bài toán 3.4 (xem [6]). Cháng minh rang neu 0 ≤ y ≤ x ≤ 1 thì x
√
y − y
√
x ≤
1
. Dau
đȁng thác xảy ra khi nào?
Bài giai.
Ta có 0 ≤ x ≤ 1 ⇒
√
x ≥ x2
Giả sả x
√
y − y
√
x
1
≤
4
⇔ x
√
y
1
+ y
√
x(1).
4
Theo Bat đȁng thác Caushy ta vó :
y
√
x +
1
≥ yx2 +
1
≥ 2 yx2.
1
= x
√
y
⇒ x
√
y − y
√
x ≤
1
.
Dau đȁng thác xảy ra khi
0 ≤ y ≤ x ≤ 1
√
x = x2
yx2 =
1
⇔
x = 1
y =
4
3.2 M t vài fíng dnng thfic te trong khoa hoc và đ i
song
Tình huong 3.1. Bà Lam có hai tài khoản ngân hàng gải tiet ki m, tại ngân hàng A bà
gải 150 tri u đong, tại ngân hàng B bà gải 250 tri u đong. Bạn của bà Lam hỏi mác lãi
suat của moi ngân hàng, nhưng bà Lam không nhớ chính xác, chỉ biet là ngân hàng B có
lãi suat cao hơn ngân hàng A là 0, 5% trong m®t năm, và sau hai năm so tien trong cả
hai tài khoản của bà là 459, 783 tri u. Hỏi lãi suat của moi ngân hàng là bao nhiêu?
Bài giai.
Goi mác lãi suat của moi ngân hàng tính trong m®t năm là xA% và xB%,(xA, xB > 0)
58
Viết đề tài giá sinh viên – ZALO:0973.287.149-TEAMLUANVAN.COM
x = 0, 5 + x
B
A
⇔
Vì tài khoản ban đau trong ngân hàng A là 150 tri u, nên sau hai năm, tőng so tien cả
goc và lãi tại ngân hàng A là 150.(1 + xA%)2.
Vì tài khoản ban đau trong ngân hàng B là 250 tri u, nên sau hai năm, tőng so tien cả
goc và lãi tại ngân hàng A là 250.(1 + xB%)2.
Theo bài ra, ta có h phương trình
xB − xA = 0, 5
150.(1 + xA%)2 + 250.(1 + xB%)2 = 459, 783
150.(1 + xA%)2 + 250.[1 + (xA + 0, 5)%]2 = 459, 783 (1)
(1) ⇔ 400.(xA%)2 + 802, 5.xA% − 57, 27675 = 0
⇔ xA = 6, 9 ho c xA = −2, 075 (loại).
Với xA = 6, 9% thì xB = 7, 4%.
Như v y lãi suat trong m®t năm của ngân hàng A là 6, 9%, ngân hàng B là 7, 4%.
Tình huong 3.2. Giám đoc m®t công ty X vàa khánh thành ngôi nhà của mình có di n
tích 4800m2, và phải xây 275 mét tường rào bao quanh. Bây giờ ông muon trong cây xanh
và hoa cho căn nhà thêm đep. Doc theo ngôi nhà ông định trong cây tùng, phía trước và
sau nhà ông định trong cây vạn tue, sao cho các cây trong đảm bảo ky thu t. Biet cőng
ra vào dài 5 mét và khu vườn có dạng hình chǎ nh t. Hãy tính xem can phải mua bao
nhiêu cây moi loại?
Bài giai.
Tình huong đ t ra là phải tính chính xác so cây can mua moi loại đe tiet ki m chi phí
thu mua và v n chuyen. Đe làm được đieu đó, can phải biet chieu dài và chieu r®ng của
khu vườn.
Goi chieu dài và chieu r®ng của khu vườn là x, y (m).x, y > 0
Vì di n tích của mảnh vườn là 4800m2 nên x.y = 4800
Vì tőng chieu dài tường rào là 275m và cőng dài 5m nên chu vi là:
(x + y).2 = 280 ⇔ x + y = 140
Ta có h phương trình :
59
Viết đề tài giá sinh viên – ZALO:0973.287.149-TEAMLUANVAN.COM
x.y = 4800
2
x + y = 140
Đây là m®t h phương trình phi tuyen đoi xáng loại 1 có nghi m (x; y) = (60; 80)
Coi khoảng cách giǎa các cây đúng kĩ thu t là cách nhau 2m.
80
Với chieu dài 80m thì so cây tùng trong hai bên theo chieu doc tường rào là 2.
2
(cây)
M t sau của khu vườn r®ng 60m thì trong được 30 cây vạn tue.
= 80
M t trước trà đi 5 m cőng, còn 55 m, như v y trong được nhieu nhat 28 cây vạn tue.
Tőng so cây vạn tue là 58, trong đó trà 4 góc trong cây tùng, v y can mua 54 cây vạn
tue.
V y so cây can mua là : 80 cây tùng và 54 cây vạn tue.
Tình huong 3.3. Nói ve sự chuyen đ®ng của các hành tinh trong thiên hà, moi m®t
hành tinh đeu có quy đạo chuyen đ®ng riêng của nó, quay xung quanh m t trời và quanh
các hành tinh khác. Quy đạo chuyen đ®ng của Trái đat và sao chői Halley’s Comet là hai
hình elip không giao nhau.
Ta đã biet phương trình của elip là m®t phương trình phi tuyen, v y thì h phương trình
phi tuyen tạo bởi phương trình chuyen đ®ng của trái đat và sao chői Halley’s Comet là
m®t h vô nghi m.
Tình huong 3.4. M®t bác nông dân dự tính đào m®t ao nuôi thả cá hình chǎ nh t, với
m®t so von xây dựng bờ kè không đői. Hãy tính toán phương án xây dựng đ® dài các cạnh
sao cho di n tích sả dụng là lớn nhat.
Bài giai.
Tình huong đ t ra là với cùng so von xây dựng bờ kè (coi chieu sâu của ao không đői),
can xác định chieu dài và chieu r®ng của ao đe di n tích sả dụng lớn nhat. Giả sả với so
von dự tính, xây được a(m) dài kè bao. Như v y a là chu vi ao cá và a không đői. Goi
x, y là đ® dài các cạnh của hình chǎ nh t, ta can giải quyet bài toán:
Cho h phương trình:
x + y =
a
(1)
x.y = m (2)
Tìm moi liên h giǎa x và y đe m đạt giá trị lớn nhat? Tà phương trình (2) suy ra
a a
y =
2
− x, suy ra xy = x(
2
− x)
60
Viết đề tài giá sinh viên – ZALO:0973.287.149-TEAMLUANVAN.COM
−
2
4x + 5xy = m (2)
≤
xy = x2 +
a
x
2
a 2 a2 a2
xy = −(x −
4
)
+
16 16
.
a a
Dau ” = ” xảy ra khi x =
4
⇒ x = y =
4
V y đe có được di n tích sả dụng lớn nhat thì nên xây ao cá hình vuông.
Tình huong 3.5. M®t bác nông dân can xây dựng m®t ho ga hình h®p chǎ nh t không
nap, có the tích 3200cm3, tỉ so giǎa chieu cao của ho và chieu r®ng của đáy bang 2. Hãy
xác định di n tích của đáy ho ga sao cho khi xây tiet ki m nguyên v t li u nhat?
Bài giai.
Goi x, y lan lượt là chieu r®ng và chieu dài của đáy ho ga.(x, y > 0)
h
Goi h là chieu cao của đáy ho ga (h > 0). Ta có
Suy ra:
= x, suy ra h = 2x.
2
- The tích ho ga là xyh = 3200 ⇔ 2x2y = 3200 ⇔ x2y = 1600.
- Di n tích tích toàn phan của ho ga (không nap) là 2(x + y)h + xy = 4x2 + 5xy = m
Ta có h phương trình:
x2y = 1600 (1)
Yêu cau của bài toán là tìm x, y sao cho m nhỏ nhat?
1600
Tà phương trình (1) suy ra y = , ta có
x
m = 4x2 + 5x.
1600
= 4x2 +
8000
.
x2
Khảo sát hàm so f(x)
x
= 4x2 +
8000
, x > 0 suy ra di n tích toàn phan của ho ga nhỏ
x
nhat bang 1200cm3, khi x = 10cm thì y = 6cm.
Suy ra di n tích đáy ho ga là 10.16 = 160(cm2).
61
Viết đề tài giá sinh viên – ZALO:0973.287.149-TEAMLUANVAN.COM
Ket lu n
Lu n văn Ve h phương trình phi tuyen và úng dựng trình bày nhǎng van đe
sau:
1. Lu n văn đã trình bày chi tiet m®t so kien thác cơ bản của h phương trình
nói chung và h phương trình phi tuyen nói riêng.
2. Trình bày các dạng toán cụ the của h phương trình phi tuyen.
3. Trình bày m®t so phương pháp giải h phương trình phi tuyen thường g p.
4. Trình bày các đe toán thi hoc sinh giỏi, tuyen sinh các trường chuyên toán
và thi đại hoc, cao đȁng liên quan đen h phương trình phi tuyen.
5. Nêu m®t vài áng dụng của h phương trình phi tuyen trong khoa hoc và đời
song.
62
Viết đề tài giá sinh viên – ZALO:0973.287.149-TEAMLUANVAN.COM
Tài li u tham khảo
Tieng Vi t
[1] Tran Thị Vân Anh (2009), Hướng dȁn giải các dạng bài t¾p tù các đe thi Quoc Gia
môn Toán của B® giáo dực và đào tạo, NXB ĐHQG Hà N®i.
[2] Hoàng Thị Dịu (2014), Lu n văn thạc sĩ M®t so phương pháp giải h phương trình
và h bat phương trình đại so, Trường ĐHKHTN - ĐHQG Hà N®i.
[3] Tran Trong Hu (2015), Đại so tuyen tính và hình hoc giải tích, NXB ĐHQG Hà
N®i.
[4] Nguyen Văn M u, Lê Ngoc Lăng, Phạm The Long, Nguyen Minh Tuan (2006), Các
đe thi Olympic Sinh viên toàn quoc, NXB Giáo dục.
[5] Lê Hoành Phò (2015), Phương pháp giải ba chuyên đe toán khó, NXB ĐHQG Hà
N®i.
[6] Nguyen Văn Vĩnh, Nguyen Đác Đong và m®t so đong nghi p (2011), 23 Chuyên đe
giải 1001 Bài toán sơ cap, NXB Giáo dục Vi t Nam.
————————————————————————————————–

More Related Content

Similar to Ve H Phương Trình Phi Tuyen Và Ứng Dụng.docx

Hàm Đơn Đi U, Tựa Đơn Đi U Và M T So Ứng Dụng Của Phép Đơn Đi U Hóa Hàm So.docx
Hàm Đơn Đi U, Tựa Đơn Đi U Và M T So Ứng Dụng Của Phép Đơn Đi U Hóa Hàm So.docxHàm Đơn Đi U, Tựa Đơn Đi U Và M T So Ứng Dụng Của Phép Đơn Đi U Hóa Hàm So.docx
Hàm Đơn Đi U, Tựa Đơn Đi U Và M T So Ứng Dụng Của Phép Đơn Đi U Hóa Hàm So.docx
DV Viết Luận văn luanvanmaster.com ZALO 0973287149
 
M T So L P Phương Trình Diophantine.docx
M T So L P Phương Trình Diophantine.docxM T So L P Phương Trình Diophantine.docx
M T So L P Phương Trình Diophantine.docx
DV Viết Luận văn luanvanmaster.com ZALO 0973287149
 
Bất đẳng thức Trong lớp các hàm lượng giác và lượng giác ngược.docx
Bất đẳng thức Trong lớp các hàm lượng giác và lượng giác ngược.docxBất đẳng thức Trong lớp các hàm lượng giác và lượng giác ngược.docx
Bất đẳng thức Trong lớp các hàm lượng giác và lượng giác ngược.docx
DV Viết Luận văn luanvanmaster.com ZALO 0973287149
 
Bat Đang Thức V I Hàm Loi B Ph N Và Ứng Dụng.docx
Bat Đang Thức V I Hàm Loi B Ph N Và Ứng Dụng.docxBat Đang Thức V I Hàm Loi B Ph N Và Ứng Dụng.docx
Bat Đang Thức V I Hàm Loi B Ph N Và Ứng Dụng.docx
DV Viết Luận văn luanvanmaster.com ZALO 0973287149
 
Bat đang thức trong so hoc và m t so Dạng toán liên quan.docx
Bat đang thức trong so hoc và m t so Dạng toán liên quan.docxBat đang thức trong so hoc và m t so Dạng toán liên quan.docx
Bat đang thức trong so hoc và m t so Dạng toán liên quan.docx
DV Viết Luận văn luanvanmaster.com ZALO 0973287149
 
Bat Đang Thức Và Bài Toán Cực Tr± Trong L P Các Đa Thức Và Phân Thức H So Ngu...
Bat Đang Thức Và Bài Toán Cực Tr± Trong L P Các Đa Thức Và Phân Thức H So Ngu...Bat Đang Thức Và Bài Toán Cực Tr± Trong L P Các Đa Thức Và Phân Thức H So Ngu...
Bat Đang Thức Và Bài Toán Cực Tr± Trong L P Các Đa Thức Và Phân Thức H So Ngu...
DV Viết Luận văn luanvanmaster.com ZALO 0973287149
 
M T So Dạng Toán Ve Dãy So Sinh B I Các Hàm So Sơ Cap.docx
M T So Dạng Toán Ve Dãy So Sinh B I Các Hàm So Sơ Cap.docxM T So Dạng Toán Ve Dãy So Sinh B I Các Hàm So Sơ Cap.docx
M T So Dạng Toán Ve Dãy So Sinh B I Các Hàm So Sơ Cap.docx
DV Viết Luận văn luanvanmaster.com ZALO 0973287149
 
Về Tổng Gauss Và Một Số Ứng Dụng.docx
Về Tổng Gauss Và Một Số Ứng Dụng.docxVề Tổng Gauss Và Một Số Ứng Dụng.docx
Về Tổng Gauss Và Một Số Ứng Dụng.docx
DV Viết Luận văn luanvanmaster.com ZALO 0973287149
 
Bat Phương Trình Hàm Sinh B I Các Đại Lư Ng Trung Bình B C Tùy Ý Và Các Dạng ...
Bat Phương Trình Hàm Sinh B I Các Đại Lư Ng Trung Bình B C Tùy Ý Và Các Dạng ...Bat Phương Trình Hàm Sinh B I Các Đại Lư Ng Trung Bình B C Tùy Ý Và Các Dạng ...
Bat Phương Trình Hàm Sinh B I Các Đại Lư Ng Trung Bình B C Tùy Ý Và Các Dạng ...
DV Viết Luận văn luanvanmaster.com ZALO 0973287149
 
Một số phương pháp tìm cực trị của các hàm phân thức Sinh bởi số tự nhiên.docx
Một số phương pháp tìm cực trị của các hàm phân thức Sinh bởi số tự nhiên.docxMột số phương pháp tìm cực trị của các hàm phân thức Sinh bởi số tự nhiên.docx
Một số phương pháp tìm cực trị của các hàm phân thức Sinh bởi số tự nhiên.docx
DV Viết Luận văn luanvanmaster.com ZALO 0973287149
 
Một Số Dạng Toán Cực Trị Trong Lîp Hàm Mũ Và Hàm Hyperbolic.docx
Một Số Dạng Toán Cực Trị Trong Lîp Hàm Mũ Và Hàm Hyperbolic.docxMột Số Dạng Toán Cực Trị Trong Lîp Hàm Mũ Và Hàm Hyperbolic.docx
Một Số Dạng Toán Cực Trị Trong Lîp Hàm Mũ Và Hàm Hyperbolic.docx
DV Viết Luận văn luanvanmaster.com ZALO 0973287149
 
Đa Thức Trong Các Bài Toán Thi Học Sinh Giỏi.docx
Đa Thức Trong Các Bài Toán Thi Học Sinh Giỏi.docxĐa Thức Trong Các Bài Toán Thi Học Sinh Giỏi.docx
Đa Thức Trong Các Bài Toán Thi Học Sinh Giỏi.docx
DV Viết Luận văn luanvanmaster.com ZALO 0973287149
 
Tổng quát về tích phân
Tổng quát về tích phân Tổng quát về tích phân
Tổng quát về tích phân
Hoàng Hải Huy
 
Tich phan %28 nguyen duy khoi%29
Tich phan %28 nguyen duy khoi%29Tich phan %28 nguyen duy khoi%29
Tich phan %28 nguyen duy khoi%29trongphuckhtn
 
Tich phan (nguyen duy khoi)
Tich phan (nguyen duy khoi)Tich phan (nguyen duy khoi)
Tich phan (nguyen duy khoi)roggerbob
 
Bài Toán Cực Trị Với Điều Kiện Ràng Buộc Bất Đẳng Thức, Hệ Bất Đẳng Thức.docx
Bài Toán Cực Trị Với Điều Kiện Ràng Buộc Bất Đẳng Thức, Hệ Bất Đẳng Thức.docxBài Toán Cực Trị Với Điều Kiện Ràng Buộc Bất Đẳng Thức, Hệ Bất Đẳng Thức.docx
Bài Toán Cực Trị Với Điều Kiện Ràng Buộc Bất Đẳng Thức, Hệ Bất Đẳng Thức.docx
DV Viết Luận văn luanvanmaster.com ZALO 0973287149
 
Bài Toán Phân Hoạch So Nguyên Dương.docx
Bài Toán Phân Hoạch    So Nguyên Dương.docxBài Toán Phân Hoạch    So Nguyên Dương.docx
Bài Toán Phân Hoạch So Nguyên Dương.docx
DV Viết Luận văn luanvanmaster.com ZALO 0973287149
 
Luận văn thạc sĩ - Một số lớp phương trình hàm trong số học.doc
Luận văn thạc sĩ - Một số lớp phương trình hàm trong số học.docLuận văn thạc sĩ - Một số lớp phương trình hàm trong số học.doc
Luận văn thạc sĩ - Một số lớp phương trình hàm trong số học.doc
Dịch vụ viết thuê đề tài trọn gói ☎☎☎ Liên hệ ZALO/TELE: 0973.287.149 👍👍
 
Ứng Dụng Hình Học Giải Tích Vào Giải Phương Trình, Bất Phương Trình Và Hệ Phư...
Ứng Dụng Hình Học Giải Tích Vào Giải Phương Trình, Bất Phương Trình Và Hệ Phư...Ứng Dụng Hình Học Giải Tích Vào Giải Phương Trình, Bất Phương Trình Và Hệ Phư...
Ứng Dụng Hình Học Giải Tích Vào Giải Phương Trình, Bất Phương Trình Và Hệ Phư...
Dịch vụ viết đề tài trọn gói 0934.573.149
 
V N Dụng Chuői Đieu Hòa Vào Giải M T So Bài Toán Dành Cho Hoc Sinh Giỏi.docx
V N Dụng Chuői Đieu Hòa Vào Giải M T So Bài Toán Dành Cho Hoc Sinh Giỏi.docxV N Dụng Chuői Đieu Hòa Vào Giải M T So Bài Toán Dành Cho Hoc Sinh Giỏi.docx
V N Dụng Chuői Đieu Hòa Vào Giải M T So Bài Toán Dành Cho Hoc Sinh Giỏi.docx
DV Viết Luận văn luanvanmaster.com ZALO 0973287149
 

Similar to Ve H Phương Trình Phi Tuyen Và Ứng Dụng.docx (20)

Hàm Đơn Đi U, Tựa Đơn Đi U Và M T So Ứng Dụng Của Phép Đơn Đi U Hóa Hàm So.docx
Hàm Đơn Đi U, Tựa Đơn Đi U Và M T So Ứng Dụng Của Phép Đơn Đi U Hóa Hàm So.docxHàm Đơn Đi U, Tựa Đơn Đi U Và M T So Ứng Dụng Của Phép Đơn Đi U Hóa Hàm So.docx
Hàm Đơn Đi U, Tựa Đơn Đi U Và M T So Ứng Dụng Của Phép Đơn Đi U Hóa Hàm So.docx
 
M T So L P Phương Trình Diophantine.docx
M T So L P Phương Trình Diophantine.docxM T So L P Phương Trình Diophantine.docx
M T So L P Phương Trình Diophantine.docx
 
Bất đẳng thức Trong lớp các hàm lượng giác và lượng giác ngược.docx
Bất đẳng thức Trong lớp các hàm lượng giác và lượng giác ngược.docxBất đẳng thức Trong lớp các hàm lượng giác và lượng giác ngược.docx
Bất đẳng thức Trong lớp các hàm lượng giác và lượng giác ngược.docx
 
Bat Đang Thức V I Hàm Loi B Ph N Và Ứng Dụng.docx
Bat Đang Thức V I Hàm Loi B Ph N Và Ứng Dụng.docxBat Đang Thức V I Hàm Loi B Ph N Và Ứng Dụng.docx
Bat Đang Thức V I Hàm Loi B Ph N Và Ứng Dụng.docx
 
Bat đang thức trong so hoc và m t so Dạng toán liên quan.docx
Bat đang thức trong so hoc và m t so Dạng toán liên quan.docxBat đang thức trong so hoc và m t so Dạng toán liên quan.docx
Bat đang thức trong so hoc và m t so Dạng toán liên quan.docx
 
Bat Đang Thức Và Bài Toán Cực Tr± Trong L P Các Đa Thức Và Phân Thức H So Ngu...
Bat Đang Thức Và Bài Toán Cực Tr± Trong L P Các Đa Thức Và Phân Thức H So Ngu...Bat Đang Thức Và Bài Toán Cực Tr± Trong L P Các Đa Thức Và Phân Thức H So Ngu...
Bat Đang Thức Và Bài Toán Cực Tr± Trong L P Các Đa Thức Và Phân Thức H So Ngu...
 
M T So Dạng Toán Ve Dãy So Sinh B I Các Hàm So Sơ Cap.docx
M T So Dạng Toán Ve Dãy So Sinh B I Các Hàm So Sơ Cap.docxM T So Dạng Toán Ve Dãy So Sinh B I Các Hàm So Sơ Cap.docx
M T So Dạng Toán Ve Dãy So Sinh B I Các Hàm So Sơ Cap.docx
 
Về Tổng Gauss Và Một Số Ứng Dụng.docx
Về Tổng Gauss Và Một Số Ứng Dụng.docxVề Tổng Gauss Và Một Số Ứng Dụng.docx
Về Tổng Gauss Và Một Số Ứng Dụng.docx
 
Bat Phương Trình Hàm Sinh B I Các Đại Lư Ng Trung Bình B C Tùy Ý Và Các Dạng ...
Bat Phương Trình Hàm Sinh B I Các Đại Lư Ng Trung Bình B C Tùy Ý Và Các Dạng ...Bat Phương Trình Hàm Sinh B I Các Đại Lư Ng Trung Bình B C Tùy Ý Và Các Dạng ...
Bat Phương Trình Hàm Sinh B I Các Đại Lư Ng Trung Bình B C Tùy Ý Và Các Dạng ...
 
Một số phương pháp tìm cực trị của các hàm phân thức Sinh bởi số tự nhiên.docx
Một số phương pháp tìm cực trị của các hàm phân thức Sinh bởi số tự nhiên.docxMột số phương pháp tìm cực trị của các hàm phân thức Sinh bởi số tự nhiên.docx
Một số phương pháp tìm cực trị của các hàm phân thức Sinh bởi số tự nhiên.docx
 
Một Số Dạng Toán Cực Trị Trong Lîp Hàm Mũ Và Hàm Hyperbolic.docx
Một Số Dạng Toán Cực Trị Trong Lîp Hàm Mũ Và Hàm Hyperbolic.docxMột Số Dạng Toán Cực Trị Trong Lîp Hàm Mũ Và Hàm Hyperbolic.docx
Một Số Dạng Toán Cực Trị Trong Lîp Hàm Mũ Và Hàm Hyperbolic.docx
 
Đa Thức Trong Các Bài Toán Thi Học Sinh Giỏi.docx
Đa Thức Trong Các Bài Toán Thi Học Sinh Giỏi.docxĐa Thức Trong Các Bài Toán Thi Học Sinh Giỏi.docx
Đa Thức Trong Các Bài Toán Thi Học Sinh Giỏi.docx
 
Tổng quát về tích phân
Tổng quát về tích phân Tổng quát về tích phân
Tổng quát về tích phân
 
Tich phan %28 nguyen duy khoi%29
Tich phan %28 nguyen duy khoi%29Tich phan %28 nguyen duy khoi%29
Tich phan %28 nguyen duy khoi%29
 
Tich phan (nguyen duy khoi)
Tich phan (nguyen duy khoi)Tich phan (nguyen duy khoi)
Tich phan (nguyen duy khoi)
 
Bài Toán Cực Trị Với Điều Kiện Ràng Buộc Bất Đẳng Thức, Hệ Bất Đẳng Thức.docx
Bài Toán Cực Trị Với Điều Kiện Ràng Buộc Bất Đẳng Thức, Hệ Bất Đẳng Thức.docxBài Toán Cực Trị Với Điều Kiện Ràng Buộc Bất Đẳng Thức, Hệ Bất Đẳng Thức.docx
Bài Toán Cực Trị Với Điều Kiện Ràng Buộc Bất Đẳng Thức, Hệ Bất Đẳng Thức.docx
 
Bài Toán Phân Hoạch So Nguyên Dương.docx
Bài Toán Phân Hoạch    So Nguyên Dương.docxBài Toán Phân Hoạch    So Nguyên Dương.docx
Bài Toán Phân Hoạch So Nguyên Dương.docx
 
Luận văn thạc sĩ - Một số lớp phương trình hàm trong số học.doc
Luận văn thạc sĩ - Một số lớp phương trình hàm trong số học.docLuận văn thạc sĩ - Một số lớp phương trình hàm trong số học.doc
Luận văn thạc sĩ - Một số lớp phương trình hàm trong số học.doc
 
Ứng Dụng Hình Học Giải Tích Vào Giải Phương Trình, Bất Phương Trình Và Hệ Phư...
Ứng Dụng Hình Học Giải Tích Vào Giải Phương Trình, Bất Phương Trình Và Hệ Phư...Ứng Dụng Hình Học Giải Tích Vào Giải Phương Trình, Bất Phương Trình Và Hệ Phư...
Ứng Dụng Hình Học Giải Tích Vào Giải Phương Trình, Bất Phương Trình Và Hệ Phư...
 
V N Dụng Chuői Đieu Hòa Vào Giải M T So Bài Toán Dành Cho Hoc Sinh Giỏi.docx
V N Dụng Chuői Đieu Hòa Vào Giải M T So Bài Toán Dành Cho Hoc Sinh Giỏi.docxV N Dụng Chuői Đieu Hòa Vào Giải M T So Bài Toán Dành Cho Hoc Sinh Giỏi.docx
V N Dụng Chuői Đieu Hòa Vào Giải M T So Bài Toán Dành Cho Hoc Sinh Giỏi.docx
 

More from DV Viết Luận văn luanvanmaster.com ZALO 0973287149

Ảnh Hưởng Của Marketing Quan Hệ Đến Lòng Trung Thành Của Khách Hàng.Tình Huốn...
Ảnh Hưởng Của Marketing Quan Hệ Đến Lòng Trung Thành Của Khách Hàng.Tình Huốn...Ảnh Hưởng Của Marketing Quan Hệ Đến Lòng Trung Thành Của Khách Hàng.Tình Huốn...
Ảnh Hưởng Của Marketing Quan Hệ Đến Lòng Trung Thành Của Khách Hàng.Tình Huốn...
DV Viết Luận văn luanvanmaster.com ZALO 0973287149
 
Phát triển nguồn nhân lực tại Uỷ ban nhân dân huyện Trà Bồng, tỉnh Quảng Ngãi...
Phát triển nguồn nhân lực tại Uỷ ban nhân dân huyện Trà Bồng, tỉnh Quảng Ngãi...Phát triển nguồn nhân lực tại Uỷ ban nhân dân huyện Trà Bồng, tỉnh Quảng Ngãi...
Phát triển nguồn nhân lực tại Uỷ ban nhân dân huyện Trà Bồng, tỉnh Quảng Ngãi...
DV Viết Luận văn luanvanmaster.com ZALO 0973287149
 
Báo cáo tốt Nghiệp tài chính hợp nhất tại tổng công ty Indochina gol...
Báo cáo tốt Nghiệp  tài chính hợp nhất tại tổng công ty Indochina gol...Báo cáo tốt Nghiệp  tài chính hợp nhất tại tổng công ty Indochina gol...
Báo cáo tốt Nghiệp tài chính hợp nhất tại tổng công ty Indochina gol...
DV Viết Luận văn luanvanmaster.com ZALO 0973287149
 
Tạo động lực thúc đẩy nhân viên làm việc tại ngân hàng TMCP Ngoại Thương Việt...
Tạo động lực thúc đẩy nhân viên làm việc tại ngân hàng TMCP Ngoại Thương Việt...Tạo động lực thúc đẩy nhân viên làm việc tại ngân hàng TMCP Ngoại Thương Việt...
Tạo động lực thúc đẩy nhân viên làm việc tại ngân hàng TMCP Ngoại Thương Việt...
DV Viết Luận văn luanvanmaster.com ZALO 0973287149
 
Phát triển công nghiệp trên địa bàn Thành phố Tam Kỳ, Tỉnh Quảng Na...
Phát triển công nghiệp trên địa bàn Thành phố Tam Kỳ, Tỉnh Quảng Na...Phát triển công nghiệp trên địa bàn Thành phố Tam Kỳ, Tỉnh Quảng Na...
Phát triển công nghiệp trên địa bàn Thành phố Tam Kỳ, Tỉnh Quảng Na...
DV Viết Luận văn luanvanmaster.com ZALO 0973287149
 
Giải pháp phát triển cho vay xuất nhập khẩu tại ngân hàng NN&PTNN ch...
Giải pháp phát triển cho vay xuất nhập khẩu tại ngân hàng NN&PTNN ch...Giải pháp phát triển cho vay xuất nhập khẩu tại ngân hàng NN&PTNN ch...
Giải pháp phát triển cho vay xuất nhập khẩu tại ngân hàng NN&PTNN ch...
DV Viết Luận văn luanvanmaster.com ZALO 0973287149
 
Hoàn thiện công tác lập báo cáo tài chính hợp nhất tại tổng công ...
Hoàn thiện công tác lập báo cáo tài chính hợp nhất tại tổng công ...Hoàn thiện công tác lập báo cáo tài chính hợp nhất tại tổng công ...
Hoàn thiện công tác lập báo cáo tài chính hợp nhất tại tổng công ...
DV Viết Luận văn luanvanmaster.com ZALO 0973287149
 
Luận Văn Thạc Sĩ Quản trị thành tích nhân viên tại Cục Hải quan TP Đà Nẵng.doc
Luận Văn Thạc Sĩ  Quản trị thành tích nhân viên tại Cục Hải quan TP Đà Nẵng.docLuận Văn Thạc Sĩ  Quản trị thành tích nhân viên tại Cục Hải quan TP Đà Nẵng.doc
Luận Văn Thạc Sĩ Quản trị thành tích nhân viên tại Cục Hải quan TP Đà Nẵng.doc
DV Viết Luận văn luanvanmaster.com ZALO 0973287149
 
Hoàn thiện công tác quản lý thuế thu nhập cá nhân tại cục thuế Tỉ...
Hoàn thiện công tác quản lý thuế thu nhập cá nhân tại cục thuế Tỉ...Hoàn thiện công tác quản lý thuế thu nhập cá nhân tại cục thuế Tỉ...
Hoàn thiện công tác quản lý thuế thu nhập cá nhân tại cục thuế Tỉ...
DV Viết Luận văn luanvanmaster.com ZALO 0973287149
 
Đề Tài Phát triển bền vững nông nghiệp Huyện Ba Tơ, Tỉnh Quảng Ngãi....
Đề Tài Phát triển bền vững nông nghiệp Huyện Ba Tơ, Tỉnh Quảng Ngãi....Đề Tài Phát triển bền vững nông nghiệp Huyện Ba Tơ, Tỉnh Quảng Ngãi....
Đề Tài Phát triển bền vững nông nghiệp Huyện Ba Tơ, Tỉnh Quảng Ngãi....
DV Viết Luận văn luanvanmaster.com ZALO 0973287149
 
Hoàn thiện công tác bảo trợ xã hội trên địa bàn huyện Phong Điền, tỉnh Thừa T...
Hoàn thiện công tác bảo trợ xã hội trên địa bàn huyện Phong Điền, tỉnh Thừa T...Hoàn thiện công tác bảo trợ xã hội trên địa bàn huyện Phong Điền, tỉnh Thừa T...
Hoàn thiện công tác bảo trợ xã hội trên địa bàn huyện Phong Điền, tỉnh Thừa T...
DV Viết Luận văn luanvanmaster.com ZALO 0973287149
 
Đề Tài Luận VănPhát triển sản phẩm du lịch tại thành phố Đà Nẵng.doc
Đề Tài Luận VănPhát triển sản phẩm du lịch tại thành phố Đà Nẵng.docĐề Tài Luận VănPhát triển sản phẩm du lịch tại thành phố Đà Nẵng.doc
Đề Tài Luận VănPhát triển sản phẩm du lịch tại thành phố Đà Nẵng.doc
DV Viết Luận văn luanvanmaster.com ZALO 0973287149
 
Đào tạo nghề cho lao động thuộc diện thu hồi đất trên địa bàn Thàn...
Đào tạo nghề cho lao động thuộc diện thu hồi đất trên địa bàn Thàn...Đào tạo nghề cho lao động thuộc diện thu hồi đất trên địa bàn Thàn...
Đào tạo nghề cho lao động thuộc diện thu hồi đất trên địa bàn Thàn...
DV Viết Luận văn luanvanmaster.com ZALO 0973287149
 
Tóm Tắt Luận Văn Thạc Sĩ Quản Trị Kinh Doanh Xây dựng chính sách Marketing tạ...
Tóm Tắt Luận Văn Thạc Sĩ Quản Trị Kinh Doanh Xây dựng chính sách Marketing tạ...Tóm Tắt Luận Văn Thạc Sĩ Quản Trị Kinh Doanh Xây dựng chính sách Marketing tạ...
Tóm Tắt Luận Văn Thạc Sĩ Quản Trị Kinh Doanh Xây dựng chính sách Marketing tạ...
DV Viết Luận văn luanvanmaster.com ZALO 0973287149
 
Đề Tài Nghiên cứu rủi ro cảm nhận đối với mua hàng thời trang trực tuyến.docx
Đề Tài Nghiên cứu rủi ro cảm nhận đối với mua hàng thời trang trực tuyến.docxĐề Tài Nghiên cứu rủi ro cảm nhận đối với mua hàng thời trang trực tuyến.docx
Đề Tài Nghiên cứu rủi ro cảm nhận đối với mua hàng thời trang trực tuyến.docx
DV Viết Luận văn luanvanmaster.com ZALO 0973287149
 
Giải pháp nâng cao động lực thúc đẩy người lao động tại công ty khai...
Giải pháp nâng cao động lực thúc đẩy người lao động tại công ty khai...Giải pháp nâng cao động lực thúc đẩy người lao động tại công ty khai...
Giải pháp nâng cao động lực thúc đẩy người lao động tại công ty khai...
DV Viết Luận văn luanvanmaster.com ZALO 0973287149
 
Giải pháp phát triển dịch vụ ngân hàng điện tử tại ngân hàng đầu ...
Giải pháp phát triển dịch vụ ngân hàng điện tử tại ngân hàng đầu ...Giải pháp phát triển dịch vụ ngân hàng điện tử tại ngân hàng đầu ...
Giải pháp phát triển dịch vụ ngân hàng điện tử tại ngân hàng đầu ...
DV Viết Luận văn luanvanmaster.com ZALO 0973287149
 
Giải pháp phát triển dịch vụ ngân hàng điện tử tại ngân hàng đầu ...
Giải pháp phát triển dịch vụ ngân hàng điện tử tại ngân hàng đầu ...Giải pháp phát triển dịch vụ ngân hàng điện tử tại ngân hàng đầu ...
Giải pháp phát triển dịch vụ ngân hàng điện tử tại ngân hàng đầu ...
DV Viết Luận văn luanvanmaster.com ZALO 0973287149
 
Quản trị quan hệ khách hàng tại Chi nhánh Viettel Đà Nẵng – Tập đoàn Viễn thô...
Quản trị quan hệ khách hàng tại Chi nhánh Viettel Đà Nẵng – Tập đoàn Viễn thô...Quản trị quan hệ khách hàng tại Chi nhánh Viettel Đà Nẵng – Tập đoàn Viễn thô...
Quản trị quan hệ khách hàng tại Chi nhánh Viettel Đà Nẵng – Tập đoàn Viễn thô...
DV Viết Luận văn luanvanmaster.com ZALO 0973287149
 
Đề Tài Đánh giá thành tích đội ngũ giảng viên trường Đại Học Phạm ...
Đề Tài Đánh giá thành tích đội ngũ giảng viên trường Đại Học Phạm ...Đề Tài Đánh giá thành tích đội ngũ giảng viên trường Đại Học Phạm ...
Đề Tài Đánh giá thành tích đội ngũ giảng viên trường Đại Học Phạm ...
DV Viết Luận văn luanvanmaster.com ZALO 0973287149
 

More from DV Viết Luận văn luanvanmaster.com ZALO 0973287149 (20)

Ảnh Hưởng Của Marketing Quan Hệ Đến Lòng Trung Thành Của Khách Hàng.Tình Huốn...
Ảnh Hưởng Của Marketing Quan Hệ Đến Lòng Trung Thành Của Khách Hàng.Tình Huốn...Ảnh Hưởng Của Marketing Quan Hệ Đến Lòng Trung Thành Của Khách Hàng.Tình Huốn...
Ảnh Hưởng Của Marketing Quan Hệ Đến Lòng Trung Thành Của Khách Hàng.Tình Huốn...
 
Phát triển nguồn nhân lực tại Uỷ ban nhân dân huyện Trà Bồng, tỉnh Quảng Ngãi...
Phát triển nguồn nhân lực tại Uỷ ban nhân dân huyện Trà Bồng, tỉnh Quảng Ngãi...Phát triển nguồn nhân lực tại Uỷ ban nhân dân huyện Trà Bồng, tỉnh Quảng Ngãi...
Phát triển nguồn nhân lực tại Uỷ ban nhân dân huyện Trà Bồng, tỉnh Quảng Ngãi...
 
Báo cáo tốt Nghiệp tài chính hợp nhất tại tổng công ty Indochina gol...
Báo cáo tốt Nghiệp  tài chính hợp nhất tại tổng công ty Indochina gol...Báo cáo tốt Nghiệp  tài chính hợp nhất tại tổng công ty Indochina gol...
Báo cáo tốt Nghiệp tài chính hợp nhất tại tổng công ty Indochina gol...
 
Tạo động lực thúc đẩy nhân viên làm việc tại ngân hàng TMCP Ngoại Thương Việt...
Tạo động lực thúc đẩy nhân viên làm việc tại ngân hàng TMCP Ngoại Thương Việt...Tạo động lực thúc đẩy nhân viên làm việc tại ngân hàng TMCP Ngoại Thương Việt...
Tạo động lực thúc đẩy nhân viên làm việc tại ngân hàng TMCP Ngoại Thương Việt...
 
Phát triển công nghiệp trên địa bàn Thành phố Tam Kỳ, Tỉnh Quảng Na...
Phát triển công nghiệp trên địa bàn Thành phố Tam Kỳ, Tỉnh Quảng Na...Phát triển công nghiệp trên địa bàn Thành phố Tam Kỳ, Tỉnh Quảng Na...
Phát triển công nghiệp trên địa bàn Thành phố Tam Kỳ, Tỉnh Quảng Na...
 
Giải pháp phát triển cho vay xuất nhập khẩu tại ngân hàng NN&PTNN ch...
Giải pháp phát triển cho vay xuất nhập khẩu tại ngân hàng NN&PTNN ch...Giải pháp phát triển cho vay xuất nhập khẩu tại ngân hàng NN&PTNN ch...
Giải pháp phát triển cho vay xuất nhập khẩu tại ngân hàng NN&PTNN ch...
 
Hoàn thiện công tác lập báo cáo tài chính hợp nhất tại tổng công ...
Hoàn thiện công tác lập báo cáo tài chính hợp nhất tại tổng công ...Hoàn thiện công tác lập báo cáo tài chính hợp nhất tại tổng công ...
Hoàn thiện công tác lập báo cáo tài chính hợp nhất tại tổng công ...
 
Luận Văn Thạc Sĩ Quản trị thành tích nhân viên tại Cục Hải quan TP Đà Nẵng.doc
Luận Văn Thạc Sĩ  Quản trị thành tích nhân viên tại Cục Hải quan TP Đà Nẵng.docLuận Văn Thạc Sĩ  Quản trị thành tích nhân viên tại Cục Hải quan TP Đà Nẵng.doc
Luận Văn Thạc Sĩ Quản trị thành tích nhân viên tại Cục Hải quan TP Đà Nẵng.doc
 
Hoàn thiện công tác quản lý thuế thu nhập cá nhân tại cục thuế Tỉ...
Hoàn thiện công tác quản lý thuế thu nhập cá nhân tại cục thuế Tỉ...Hoàn thiện công tác quản lý thuế thu nhập cá nhân tại cục thuế Tỉ...
Hoàn thiện công tác quản lý thuế thu nhập cá nhân tại cục thuế Tỉ...
 
Đề Tài Phát triển bền vững nông nghiệp Huyện Ba Tơ, Tỉnh Quảng Ngãi....
Đề Tài Phát triển bền vững nông nghiệp Huyện Ba Tơ, Tỉnh Quảng Ngãi....Đề Tài Phát triển bền vững nông nghiệp Huyện Ba Tơ, Tỉnh Quảng Ngãi....
Đề Tài Phát triển bền vững nông nghiệp Huyện Ba Tơ, Tỉnh Quảng Ngãi....
 
Hoàn thiện công tác bảo trợ xã hội trên địa bàn huyện Phong Điền, tỉnh Thừa T...
Hoàn thiện công tác bảo trợ xã hội trên địa bàn huyện Phong Điền, tỉnh Thừa T...Hoàn thiện công tác bảo trợ xã hội trên địa bàn huyện Phong Điền, tỉnh Thừa T...
Hoàn thiện công tác bảo trợ xã hội trên địa bàn huyện Phong Điền, tỉnh Thừa T...
 
Đề Tài Luận VănPhát triển sản phẩm du lịch tại thành phố Đà Nẵng.doc
Đề Tài Luận VănPhát triển sản phẩm du lịch tại thành phố Đà Nẵng.docĐề Tài Luận VănPhát triển sản phẩm du lịch tại thành phố Đà Nẵng.doc
Đề Tài Luận VănPhát triển sản phẩm du lịch tại thành phố Đà Nẵng.doc
 
Đào tạo nghề cho lao động thuộc diện thu hồi đất trên địa bàn Thàn...
Đào tạo nghề cho lao động thuộc diện thu hồi đất trên địa bàn Thàn...Đào tạo nghề cho lao động thuộc diện thu hồi đất trên địa bàn Thàn...
Đào tạo nghề cho lao động thuộc diện thu hồi đất trên địa bàn Thàn...
 
Tóm Tắt Luận Văn Thạc Sĩ Quản Trị Kinh Doanh Xây dựng chính sách Marketing tạ...
Tóm Tắt Luận Văn Thạc Sĩ Quản Trị Kinh Doanh Xây dựng chính sách Marketing tạ...Tóm Tắt Luận Văn Thạc Sĩ Quản Trị Kinh Doanh Xây dựng chính sách Marketing tạ...
Tóm Tắt Luận Văn Thạc Sĩ Quản Trị Kinh Doanh Xây dựng chính sách Marketing tạ...
 
Đề Tài Nghiên cứu rủi ro cảm nhận đối với mua hàng thời trang trực tuyến.docx
Đề Tài Nghiên cứu rủi ro cảm nhận đối với mua hàng thời trang trực tuyến.docxĐề Tài Nghiên cứu rủi ro cảm nhận đối với mua hàng thời trang trực tuyến.docx
Đề Tài Nghiên cứu rủi ro cảm nhận đối với mua hàng thời trang trực tuyến.docx
 
Giải pháp nâng cao động lực thúc đẩy người lao động tại công ty khai...
Giải pháp nâng cao động lực thúc đẩy người lao động tại công ty khai...Giải pháp nâng cao động lực thúc đẩy người lao động tại công ty khai...
Giải pháp nâng cao động lực thúc đẩy người lao động tại công ty khai...
 
Giải pháp phát triển dịch vụ ngân hàng điện tử tại ngân hàng đầu ...
Giải pháp phát triển dịch vụ ngân hàng điện tử tại ngân hàng đầu ...Giải pháp phát triển dịch vụ ngân hàng điện tử tại ngân hàng đầu ...
Giải pháp phát triển dịch vụ ngân hàng điện tử tại ngân hàng đầu ...
 
Giải pháp phát triển dịch vụ ngân hàng điện tử tại ngân hàng đầu ...
Giải pháp phát triển dịch vụ ngân hàng điện tử tại ngân hàng đầu ...Giải pháp phát triển dịch vụ ngân hàng điện tử tại ngân hàng đầu ...
Giải pháp phát triển dịch vụ ngân hàng điện tử tại ngân hàng đầu ...
 
Quản trị quan hệ khách hàng tại Chi nhánh Viettel Đà Nẵng – Tập đoàn Viễn thô...
Quản trị quan hệ khách hàng tại Chi nhánh Viettel Đà Nẵng – Tập đoàn Viễn thô...Quản trị quan hệ khách hàng tại Chi nhánh Viettel Đà Nẵng – Tập đoàn Viễn thô...
Quản trị quan hệ khách hàng tại Chi nhánh Viettel Đà Nẵng – Tập đoàn Viễn thô...
 
Đề Tài Đánh giá thành tích đội ngũ giảng viên trường Đại Học Phạm ...
Đề Tài Đánh giá thành tích đội ngũ giảng viên trường Đại Học Phạm ...Đề Tài Đánh giá thành tích đội ngũ giảng viên trường Đại Học Phạm ...
Đề Tài Đánh giá thành tích đội ngũ giảng viên trường Đại Học Phạm ...
 

Recently uploaded

BÀI TẬP BỔ TRỢ TIẾNG ANH I-LEARN SMART WORLD 9 CẢ NĂM CÓ TEST THEO UNIT NĂM H...
BÀI TẬP BỔ TRỢ TIẾNG ANH I-LEARN SMART WORLD 9 CẢ NĂM CÓ TEST THEO UNIT NĂM H...BÀI TẬP BỔ TRỢ TIẾNG ANH I-LEARN SMART WORLD 9 CẢ NĂM CÓ TEST THEO UNIT NĂM H...
BÀI TẬP BỔ TRỢ TIẾNG ANH I-LEARN SMART WORLD 9 CẢ NĂM CÓ TEST THEO UNIT NĂM H...
Nguyen Thanh Tu Collection
 
PLĐC-chương 1 (1).ppt của trường ĐH Ngoại thương
PLĐC-chương 1 (1).ppt của trường  ĐH Ngoại thươngPLĐC-chương 1 (1).ppt của trường  ĐH Ngoại thương
PLĐC-chương 1 (1).ppt của trường ĐH Ngoại thương
hieutrinhvan27052005
 
CHUYÊN ĐỀ DẠY THÊM HÓA HỌC LỚP 10 - SÁCH MỚI - FORM BÀI TẬP 2025 (DÙNG CHUNG ...
CHUYÊN ĐỀ DẠY THÊM HÓA HỌC LỚP 10 - SÁCH MỚI - FORM BÀI TẬP 2025 (DÙNG CHUNG ...CHUYÊN ĐỀ DẠY THÊM HÓA HỌC LỚP 10 - SÁCH MỚI - FORM BÀI TẬP 2025 (DÙNG CHUNG ...
CHUYÊN ĐỀ DẠY THÊM HÓA HỌC LỚP 10 - SÁCH MỚI - FORM BÀI TẬP 2025 (DÙNG CHUNG ...
Nguyen Thanh Tu Collection
 
Quan Tri Doi Moi Sang Tao_ Innovation Management
Quan Tri Doi Moi Sang Tao_ Innovation ManagementQuan Tri Doi Moi Sang Tao_ Innovation Management
Quan Tri Doi Moi Sang Tao_ Innovation Management
ChuPhan32
 
Ngân hàng điện tử số ptit - giảng viên cô Hà
Ngân hàng điện tử số ptit - giảng viên cô HàNgân hàng điện tử số ptit - giảng viên cô Hà
Ngân hàng điện tử số ptit - giảng viên cô Hà
onLongV
 
THONG BAO nop ho so xet tuyen TS6 24-25.pdf
THONG BAO nop ho so xet tuyen TS6 24-25.pdfTHONG BAO nop ho so xet tuyen TS6 24-25.pdf
THONG BAO nop ho so xet tuyen TS6 24-25.pdf
QucHHunhnh
 
Halloween vocabulary for kids in primary school
Halloween vocabulary for kids in primary schoolHalloween vocabulary for kids in primary school
Halloween vocabulary for kids in primary school
AnhPhm265031
 
40 câu hỏi - đáp Bộ luật dân sự năm 2015 (1).doc
40 câu hỏi - đáp Bộ  luật dân sự năm  2015 (1).doc40 câu hỏi - đáp Bộ  luật dân sự năm  2015 (1).doc
40 câu hỏi - đáp Bộ luật dân sự năm 2015 (1).doc
NguynDimQunh33
 
insulin cho benh nhan nam vien co tang duong huyet
insulin cho benh nhan nam vien co tang duong huyetinsulin cho benh nhan nam vien co tang duong huyet
insulin cho benh nhan nam vien co tang duong huyet
lmhong80
 
Chương 3 Linh kien ban dan và KD dien tu - Copy.ppt
Chương 3 Linh kien ban dan và KD dien tu - Copy.pptChương 3 Linh kien ban dan và KD dien tu - Copy.ppt
Chương 3 Linh kien ban dan và KD dien tu - Copy.ppt
PhiTrnHngRui
 
Biểu tượng trăng và bầu trời trong tác phẩm của Nguyễn Quang Thiều
Biểu tượng trăng và bầu trời trong tác phẩm của Nguyễn Quang ThiềuBiểu tượng trăng và bầu trời trong tác phẩm của Nguyễn Quang Thiều
Biểu tượng trăng và bầu trời trong tác phẩm của Nguyễn Quang Thiều
lamluanvan.net Viết thuê luận văn
 
Smartbiz_He thong MES nganh may mac_2024june
Smartbiz_He thong MES nganh may mac_2024juneSmartbiz_He thong MES nganh may mac_2024june
Smartbiz_He thong MES nganh may mac_2024june
SmartBiz
 
Văn 7. Truyện ngụ ngôn Rùa và thỏ+ Viết PT nhân vật.docx
Văn 7. Truyện ngụ ngôn Rùa và thỏ+ Viết PT nhân vật.docxVăn 7. Truyện ngụ ngôn Rùa và thỏ+ Viết PT nhân vật.docx
Văn 7. Truyện ngụ ngôn Rùa và thỏ+ Viết PT nhân vật.docx
metamngoc123
 
FSSC 22000 version 6_Seminar_FINAL end.pptx
FSSC 22000 version 6_Seminar_FINAL end.pptxFSSC 22000 version 6_Seminar_FINAL end.pptx
FSSC 22000 version 6_Seminar_FINAL end.pptx
deviv80273
 
LỊCH SỬ 12 - CHUYÊN ĐỀ 10 - TRẮC NGHIỆM.pptx
LỊCH SỬ 12 - CHUYÊN ĐỀ 10 - TRẮC NGHIỆM.pptxLỊCH SỬ 12 - CHUYÊN ĐỀ 10 - TRẮC NGHIỆM.pptx
LỊCH SỬ 12 - CHUYÊN ĐỀ 10 - TRẮC NGHIỆM.pptx
12D241NguynPhmMaiTra
 
100 DẪN CHỨNG NGHỊ LUẬN XÃ HỘiI HAY.docx
100 DẪN CHỨNG NGHỊ LUẬN XÃ HỘiI HAY.docx100 DẪN CHỨNG NGHỊ LUẬN XÃ HỘiI HAY.docx
100 DẪN CHỨNG NGHỊ LUẬN XÃ HỘiI HAY.docx
khanhthy3000
 
Giải phẫu tim sau đại học- LÊ QUANG TUYỀN
Giải phẫu tim sau đại học- LÊ QUANG TUYỀNGiải phẫu tim sau đại học- LÊ QUANG TUYỀN
Giải phẫu tim sau đại học- LÊ QUANG TUYỀN
linh miu
 
Sinh-12-Chuyên-2022-2023.dành cho ôn thi hsg
Sinh-12-Chuyên-2022-2023.dành cho ôn thi hsgSinh-12-Chuyên-2022-2023.dành cho ôn thi hsg
Sinh-12-Chuyên-2022-2023.dành cho ôn thi hsg
vivan030207
 

Recently uploaded (18)

BÀI TẬP BỔ TRỢ TIẾNG ANH I-LEARN SMART WORLD 9 CẢ NĂM CÓ TEST THEO UNIT NĂM H...
BÀI TẬP BỔ TRỢ TIẾNG ANH I-LEARN SMART WORLD 9 CẢ NĂM CÓ TEST THEO UNIT NĂM H...BÀI TẬP BỔ TRỢ TIẾNG ANH I-LEARN SMART WORLD 9 CẢ NĂM CÓ TEST THEO UNIT NĂM H...
BÀI TẬP BỔ TRỢ TIẾNG ANH I-LEARN SMART WORLD 9 CẢ NĂM CÓ TEST THEO UNIT NĂM H...
 
PLĐC-chương 1 (1).ppt của trường ĐH Ngoại thương
PLĐC-chương 1 (1).ppt của trường  ĐH Ngoại thươngPLĐC-chương 1 (1).ppt của trường  ĐH Ngoại thương
PLĐC-chương 1 (1).ppt của trường ĐH Ngoại thương
 
CHUYÊN ĐỀ DẠY THÊM HÓA HỌC LỚP 10 - SÁCH MỚI - FORM BÀI TẬP 2025 (DÙNG CHUNG ...
CHUYÊN ĐỀ DẠY THÊM HÓA HỌC LỚP 10 - SÁCH MỚI - FORM BÀI TẬP 2025 (DÙNG CHUNG ...CHUYÊN ĐỀ DẠY THÊM HÓA HỌC LỚP 10 - SÁCH MỚI - FORM BÀI TẬP 2025 (DÙNG CHUNG ...
CHUYÊN ĐỀ DẠY THÊM HÓA HỌC LỚP 10 - SÁCH MỚI - FORM BÀI TẬP 2025 (DÙNG CHUNG ...
 
Quan Tri Doi Moi Sang Tao_ Innovation Management
Quan Tri Doi Moi Sang Tao_ Innovation ManagementQuan Tri Doi Moi Sang Tao_ Innovation Management
Quan Tri Doi Moi Sang Tao_ Innovation Management
 
Ngân hàng điện tử số ptit - giảng viên cô Hà
Ngân hàng điện tử số ptit - giảng viên cô HàNgân hàng điện tử số ptit - giảng viên cô Hà
Ngân hàng điện tử số ptit - giảng viên cô Hà
 
THONG BAO nop ho so xet tuyen TS6 24-25.pdf
THONG BAO nop ho so xet tuyen TS6 24-25.pdfTHONG BAO nop ho so xet tuyen TS6 24-25.pdf
THONG BAO nop ho so xet tuyen TS6 24-25.pdf
 
Halloween vocabulary for kids in primary school
Halloween vocabulary for kids in primary schoolHalloween vocabulary for kids in primary school
Halloween vocabulary for kids in primary school
 
40 câu hỏi - đáp Bộ luật dân sự năm 2015 (1).doc
40 câu hỏi - đáp Bộ  luật dân sự năm  2015 (1).doc40 câu hỏi - đáp Bộ  luật dân sự năm  2015 (1).doc
40 câu hỏi - đáp Bộ luật dân sự năm 2015 (1).doc
 
insulin cho benh nhan nam vien co tang duong huyet
insulin cho benh nhan nam vien co tang duong huyetinsulin cho benh nhan nam vien co tang duong huyet
insulin cho benh nhan nam vien co tang duong huyet
 
Chương 3 Linh kien ban dan và KD dien tu - Copy.ppt
Chương 3 Linh kien ban dan và KD dien tu - Copy.pptChương 3 Linh kien ban dan và KD dien tu - Copy.ppt
Chương 3 Linh kien ban dan và KD dien tu - Copy.ppt
 
Biểu tượng trăng và bầu trời trong tác phẩm của Nguyễn Quang Thiều
Biểu tượng trăng và bầu trời trong tác phẩm của Nguyễn Quang ThiềuBiểu tượng trăng và bầu trời trong tác phẩm của Nguyễn Quang Thiều
Biểu tượng trăng và bầu trời trong tác phẩm của Nguyễn Quang Thiều
 
Smartbiz_He thong MES nganh may mac_2024june
Smartbiz_He thong MES nganh may mac_2024juneSmartbiz_He thong MES nganh may mac_2024june
Smartbiz_He thong MES nganh may mac_2024june
 
Văn 7. Truyện ngụ ngôn Rùa và thỏ+ Viết PT nhân vật.docx
Văn 7. Truyện ngụ ngôn Rùa và thỏ+ Viết PT nhân vật.docxVăn 7. Truyện ngụ ngôn Rùa và thỏ+ Viết PT nhân vật.docx
Văn 7. Truyện ngụ ngôn Rùa và thỏ+ Viết PT nhân vật.docx
 
FSSC 22000 version 6_Seminar_FINAL end.pptx
FSSC 22000 version 6_Seminar_FINAL end.pptxFSSC 22000 version 6_Seminar_FINAL end.pptx
FSSC 22000 version 6_Seminar_FINAL end.pptx
 
LỊCH SỬ 12 - CHUYÊN ĐỀ 10 - TRẮC NGHIỆM.pptx
LỊCH SỬ 12 - CHUYÊN ĐỀ 10 - TRẮC NGHIỆM.pptxLỊCH SỬ 12 - CHUYÊN ĐỀ 10 - TRẮC NGHIỆM.pptx
LỊCH SỬ 12 - CHUYÊN ĐỀ 10 - TRẮC NGHIỆM.pptx
 
100 DẪN CHỨNG NGHỊ LUẬN XÃ HỘiI HAY.docx
100 DẪN CHỨNG NGHỊ LUẬN XÃ HỘiI HAY.docx100 DẪN CHỨNG NGHỊ LUẬN XÃ HỘiI HAY.docx
100 DẪN CHỨNG NGHỊ LUẬN XÃ HỘiI HAY.docx
 
Giải phẫu tim sau đại học- LÊ QUANG TUYỀN
Giải phẫu tim sau đại học- LÊ QUANG TUYỀNGiải phẫu tim sau đại học- LÊ QUANG TUYỀN
Giải phẫu tim sau đại học- LÊ QUANG TUYỀN
 
Sinh-12-Chuyên-2022-2023.dành cho ôn thi hsg
Sinh-12-Chuyên-2022-2023.dành cho ôn thi hsgSinh-12-Chuyên-2022-2023.dành cho ôn thi hsg
Sinh-12-Chuyên-2022-2023.dành cho ôn thi hsg
 

Ve H Phương Trình Phi Tuyen Và Ứng Dụng.docx

  • 1. ĐẠI HOC THÁI NGUYÊN TRƯ NG ĐẠI HOC KHOA HOC Tải tài liệu tại sividoc.com Viết đề tài giá sinh viên – ZALO:0973.287.149-TEAMLUANVAN.COM NGUYEN TH± ANH VE H PHƯƠNG TRÌNH PHI TUYEN VÀ ỨNG DỤNG LU N VĂN THẠC SĨ TOÁN HOC Thái Nguyên - 2017
  • 2. ĐẠI HOC THÁI NGUYÊN TRƯ NG ĐẠI HOC KHOA HOC Tải tài liệu tại sividoc.com Viết đề tài giá sinh viên – ZALO:0973.287.149-TEAMLUANVAN.COM NGUYEN TH± ANH VE H PHƯƠNG TRÌNH PHI TUYEN VÀ ỨNG DỤNG LU N VĂN THẠC SĨ TOÁN HOC Chuyên ngành: Phương pháp Toán sơ cap Mã so: 60 46 01 13 NGƯ I HƯ NG DAN KHOA HOC PGS.TS. NÔNG QUOC CHINH Thái Nguyên - 2017
  • 3. i Viết đề tài giá sinh viên – ZALO:0973.287.149-TEAMLUANVAN.COM Mnc lnc M đau 1 1 M t so kien thfíc bo tr ve h phương trình 3 1.1 H phương trình tuyen tính (xem [3])................................................................... 3 1.2 H phương trình phi tuyen.................................................................................8 2 Nhfing phương pháp thư ng dùng đe giải h phương trình phi tuyen 17 2.1 Phương pháp the...................................................................................................... 17 2.2 Phương pháp đ t ȁn phụ......................................................................................... 25 2.3 Phương pháp sả dụng tính đơn đi u của hàm so(xem [2]).................................. 30 2.4 Phương pháp sả dụng bat đȁng thác..................................................................... 36 2.5 Phoi hợp nhieu phương pháp giải h phương trình............................................ 45 3 M t so fíng dnng của h phương trình phi tuyen 54 3.1 Úng dụng của h phương trình đa thác trong giải các bài toán cực trị và cháng minh bat đȁng thác. ................................................................................. 54 3.2 M®t vài áng dụng thực te trong khoa hoc và đời song...................................... 56 Ket lu n 60 Tài li u tham khảo 61
  • 4. 1 Viết đề tài giá sinh viên – ZALO:0973.287.149-TEAMLUANVAN.COM M đau Giải h phương trình là m®t phan quan trong trong chương trình toán THPT và trong các chuyên ngành Đại so, Giải tích. Khi đe c p đen vi c giải h phương trình, hoc sinh trung hoc cơ sở de dàng tìm được các phương pháp giải h phương trình tuyen tính như giải h bang phương pháp c®ng đại so, phương pháp the của hoc sinh lớp 9, hay sả dụng định thác của lớp 10. Tuy nhiên, đoi với h phương trình phi tuyen, gan như có rat ít tài li u nghiên cáu sâu ve lĩnh vực này. Trong các kì thi hoc sinh giỏi các cap, các kì thi Olympic trong nước và quoc te, các h phương trình phi tuyen thường là loại khó và ít có định dạng cũng như phương pháp giải cụ the. Vì v y hoc sinh g p rat nhieu khó khăn trong vi c giải các bài toán ve h phương trình phi tuyen trong chương trình toán phő thông và trong các kì thi hoc sinh giỏi các cap. Đe tài lu n văn nghiên cáu m®t so h phương trình phi tuyen thường g p trong chương trình toán phő thông và các phương pháp giải h phương trình này cũng như áng dụng của chúng dùng trong ôn luy n hoc sinh giỏi lớp 9 và ôn thi tuyen sinh vào lớp 10 đ c bi t là dành cho hoc sinh ôn thi chuyên toán. Ngoài phan mở đau và ket lu n, lu n văn được trình bày gom ba chương: Chương 1. M t so kien thfíc bo tr ve h phương trình. Chương này trình bày các kien thác bő trợ, các khái ni m, định nghĩa cơ bản ve h phương trình, t p nghi m của h phương trình nói chung và h phương trình phi tuyen nói riêng.
  • 5. 2 Viết đề tài giá sinh viên – ZALO:0973.287.149-TEAMLUANVAN.COM Chương 2. Nhfing phương pháp thư ng dùng đe giải h phương trình phi tuyen. Chương 2, trình bày các dạng toán thường g p của h phương trình phi tuyen và cách giải chúng được tìm hieu qua các tài li u [1], [2], [3], [4], [5]. Chương 3. M t so fíng dnng của h phương trình phi tuyen. Lu n văn được hoàn thành tại trường Đại hoc Khoa hoc - Đại hoc Thái Nguyên với sự hướng dan của PGS.TS. Nông Quoc Chinh. Tác giả xin được bày tỏ lòng biet ơn sâu sac đoi với sự quan tâm hướng dan của thay, tới các thay cô trong Ban giám hi u, Phòng đào tạo và Khoa Toán - Tin của trường Đại hoc Khoa hoc. Đong thời tác giả xin được cảm ơn các anh chị cùng khóa đã chỉ bảo, hướng dan cho tác giả hoc t p và hoàn thành ke hoạch hoc t p. Thái Nguyên, ngày 25 tháng 4 năm 2017. Hoc viên Nguyen Thị Anh
  • 6. 3 Viết đề tài giá sinh viên – ZALO:0973.287.149-TEAMLUANVAN.COM Σ Chương 1 M t so kien thfíc bo tr ve h phương trình Chương này tôi xin trình bày các kien thác bő trợ, các khái ni m, định nghĩa cơ bản ve h phương trình, t p nghi m của h phương trình nói chung và h phương trình phi tuyen nói riêng. 1.1 H phương trình tuyen tính (xem [3]) 1.1.1 H phương trình tuyen tính tong quát Định nghĩa 1.1. Cho K là m®t trường. H m phương trình tuyen tính n ȁn x1, x2, . . . , xn h so trên trường K là h có dạng a11x1 + a12x2 + · · · + a1nxn = b1 a21x1 + a22x2 + · · · + a2nxn = b2 · · · am1x1 + am2x2 + · · · + amnxn = bm hay có the viet gon hơn: n (aikxk) = bi, i = 1, . . . , m. k=1 Trong đó aik, bi là các phan tả thu®c trường K, aik goi là h so của ȁn xk, bi là h so tự do, i = 1, . . . , m; k = 1, . . . , n.
  • 7. 4 Viết đề tài giá sinh viên – ZALO:0973.287.149-TEAMLUANVAN.COM Σ . . . Σ . . H phương trình (1.1) goi là h phương trình tuyen tính tőng quát. Đăc bi t neu b1 = . . . = bm = 0 thì h (1.1) có dạng n (aikxk) = 0, i = 1, . . . , m. k=1 được goi là h phương trình tuyen tính thuan nhat. 1.1.2 M t so phương pháp giải h phương trình tuyen tính 1.1.2.1 Quy tac Cramer Định nghĩa 1.2. M®t h phương trình tuyen tính có so phương trình bang so ȁn và ma tr n A của h có định thác |A| = 0 được goi là h Cramer. Định lj 1.1. Định lý Cramer (Cramer’s rule – công thác xác định công thác nghi m của h Cramer) H Cramer a11x1 + a12x2 + · · · + a1nxn = b1 a21x1 + a22x2 + · · · + a2nxn = b2 · · · an1x1 + an2x2 + · · · + annxn = bn Là m®t h xác định. Nghi m duy nhat của h được xác định bởi công thác ∆j xj = ∆ (6) Trong đó ∆ = |A| và a11 · · · b1 · · · a1n ∆j = · · · an1 · · · bn · · · ann Công thác (6) được goi là công thác Cramer. Chú j: Định thác ∆j là định thác của ma tr n nh n được tà ma tr n A của h (1.2) bang cách thay các phan tả ở c®t thá j (H so của ȁn xj) bởi các phan tả b1, · · · , bn (Các h so tự do). Định lj 1.2. H n phương trình tuyen tính thuan nhat n ȁn so n aikxk = 0, i = 1, . . . , n k=1 .
  • 8. 5 Viết đề tài giá sinh viên – ZALO:0973.287.149-TEAMLUANVAN.COM x1 + x2 + · · · + xn − nxn = n − 1 k n n n x2 + 2x3 + 3x4 + · · · + nx1 = 2 (x1 + x2 + · · · + xn) − nxn = k − (k + 1) bx1 + ax2 + bx3 + · · · + bx2001 + bx2002 = 2 chỉ có nghi m tam thường Θ = (0, · · · , 0) khi và chỉ khi định thác |A| = 0. Ví dn 1.1 (Xem [4]). Giải h phương trình: x1 + 2x2 + 3x3 + · · · + nxn = 1 · · · xn + 2x1 + 3x2 + · · · + nxn−1 = n (Đe thi OLYMPIC Toán Sinh viên năm 1999) Bài giai. C®ng tat cả các phương trình của h , suy ra x1 + x2 + · · · + xn = 1 Tiep theo, lan lượt lay phương trình thá k trà cho phương trình thá k − 1(1 < k < n); trà phương trình thá n cho phương trình thá nhat. Suy ra x = (x1 + x2 + · · · + xn) + 1 = 2 x = − n − 2 = 2 − n n n n xk = 2 V y h phương trình có nghi m xn n = 2 − n n Với k = 1, 2, · · · , n − 1. Ví dn 1.2 (xem [4]). Cho h phương trình ax1 + bx2 + bx3 + · · · + bx2001 + bx2002 = 1 · · · · · · · · · bx1 + bx2 + bx3 + · · · + bx2001 + ax2002 = 2002 Tìm đieu ki n của a đoi với b đe h đã cho có nghi m duy nhat. ⇔
  • 9. 6 Viết đề tài giá sinh viên – ZALO:0973.287.149-TEAMLUANVAN.COM . . . . 11 12 · · · 1r · · · 1n 1 . b a b · · · b b . . 1 a b · · · b b . · · · · · · · · · (Đe thi OLYMPIC toán Sinh viên 2002) Bài giai. Kí hi u D là định thác của h phương trình. Ta có a b b · · · b b D = · · · b b · · · a b . b b · · · b a . 2002×2002 C®ng tat cả các c®t với c®t đau tiên, ta có 1 b b · · · b b (a + 2001b) · · · 1 b · · · a b . 1 b · · · b a . 2002×2002 (Nhân hàng đau với −1 roi c®ng vào các hàng còn lại) = (a + 2001b).(a − b)2001 V y h đã cho có nghi m duy nhat ⇔ a /= b và a −2001b. 1.1.2.2 Thu t toán Gaoxơ (Gauss) Trong đại so tuyen tính, thu t toán Gauss là m®t thu t toán có the được sả dụng đe tìm nghi m của m®t h phương trình tuyen tính, tìm hạng (hay rank) của m®t ma tr n, đe tính ma tr n nghịch đảo của m®t ma tr n vuông khả nghịch. thu t toán Gauss được đ t theo tên của nhà toán hoc Đác là Carl Friedrich Gauss. • N®i dung của thu t toán Gauss đe giải h phương trình tuyen tính là : Thực hi n liên tiep các phép bien đői tương đương đe tà h phương trình (1.1) ban đau chúng ta nhân được m®t h mới tương đương có so phương trình ít hơn, và có dạng bâc thang sau: aJ xi1 + aJ xi2 + + aJ xir + + aJ xin = bJ aJ 22xi2 + · · · + a2 J nxir + · · · + aJ 2nxin = bJ 2 aJ rrxir + · · · + aJ rnxin = bJ r Trong đó xij(j = 1, · · · , n) là m®t hoán vị nào đó của các ȁn xi, (i = 1, · · · , n); aik /= 0 với k = 1, · · · , r.
  • 10. 7 Viết đề tài giá sinh viên – ZALO:0973.287.149-TEAMLUANVAN.COM x1 + x2 + x3 + · · · + xn−1 + xn = x2 + x3 + · · · + xn−1 + xn = 20052 − 2 a + x = + · · · + x a 2004 + x1 (Đe thi OLYMPIC toán Sinh viên 2002) • Các trường hợp sau có the xảy ra: Trường hợp 1: Neu trong quá trình bien đői g p phương trình vô nghi m dạng 0x1 +· · ·+ 0xn = b, b 0, thì ta dàng lại và ket lu n h vô nghi m. Trường hợp 2: Neu r = r, khi đó phương trình cuoi cùng của h có dạng aJ mnxin = bJ n. H (1.6) là m®t h Cramer, do đó h có duy nhat m®t nghi m. Có the tìm nghi m đóbang cách giải tàng phương trình của h (1.6) tà dưới lên và thay the dan giá trị của cácȁn tìm được tà phương trình dưới vào các phương trình trên. Trường hợp 3 : Neu r < n thì h phương trình đã cho có vô so nghi m. Đe giải h 1.6 ta có the xem các ȁn xir+1 , · · · , xin là các tham so (lay giá trị tùy ý) và chuyen các so hàng cháa các ȁn đó sang ve phải; roi dùng phương pháp ở trường hợp 2 đoi với các ȁn xi1 , · · · , xir xem xir+1 , · · · , xin là tham so. Trong thực te đe bien đői h phương trình tuyen tính 1.1 ban đau ve dạng b c thang, người ta thường thực hi n các phép bien đői tương đương trên ma tr n bő sung AJ. Các phép bien đői đó được goi là các phép bien đői sơ cap. Ví dn 1.3 (xem [3]). Giải h phương trình a · · · · · · · · · 1 n−1 n 2005n − 1 Bài giai. C®ng thêm bieu thác x1 + x2 + · · · + xi−1 vào cả hai ve của phương trình thá i của h đã cho, với i = 2, 3, · · · , n, ta có x1 + x2 a + · · · + xn a = a + x1 + x2 + · · · + xi−1 + x 2005i − 1 2005i − 2005 + x2 + · · · + xi−1 ⇒ 2004 = 2005i − 1 ( V y với i = 2, 3,··· , n ) 2004 xi = (x1 + x2 + · · · + xi) − (x1 + x2 + · · · + xi−1) a 2005i+1 − 2005 a 2005i − 2005 = 2005i+1 . a = 2005i 2004 − 2005i . 2004 x 1
  • 11. 8 Viết đề tài giá sinh viên – ZALO:0973.287.149-TEAMLUANVAN.COM g = g (x;y) (y;x) ∗ Ph ương pháp giải: a Lay phương trình thá nhat trà đi phương trình thá hai, ta được x1 = 2005 a a V y xi = 2005i (i = 1, 2, · · · , n − 1); xn = 2004.2005n−1 1.2 H phương trình phi tuyen 1.2.1 Khái ni m Định nghĩa 1.3 (xem [6]). M®t phương trình đại so không phải là tuyen tính được goi là phương trình phi tuyen. Định nghĩa 1.4 (xem [6]). H phương trình phi tuyen là m®t h gom nhieu phương trình, nhieu ȁn, trong đó có ít nhat m®t phương trình phi tuyen. 1.2.2 M t vài dạng h phương trình phi tuyen 1.2.2.1 H phương trình đại so 1.2.2.1.1 H phương trình đoi xfíng loại I (xem [2]) Định nghĩa 1.5. H phương trình đoi xáng loại I là h phương trình có dạng f(x;y) = 0 g(x;y) = 0 Với f(x;y) = f(y;x) Khi tráo đői vị trí của x và y trong h phương trình thì h không thay đői. Nh n xét 1.1. Neu h có nghi m (x0; y0) thì (y0; x0) cũng là m®t nghi m của h . Đ t x + y = S xy = P Đieu ki n S2 − 4P ≥ 0 Ta được h phương trình F(S;P ) = 0 E(S;P ) = 0 Ta tìm được S; P. Khi đó x, y là nghi m của h phương trình X2 − SX + P = 0(∗) Giả sả phương trình có hai nghi m X1; X2
  • 12. 9 Viết đề tài giá sinh viên – ZALO:0973.287.149-TEAMLUANVAN.COM 2 2 x y + x y = a − ∗ − ∆ = S2 − 4P ≥ 0 2x3 = a x = 3 + Neu ∆ > 0 thì X1 X2 nên h (1.5) có hai nghi m phân bi t (X1; X2), (X2; X1). + Neu ∆ = 0 thì X1 = X2 = x0 nên h có nghi m duy nhat (x0; x0). + H có ít nhat m®t nghiêm thỏa mãn x ≥ 0 ⇔ h (∗) có ít nhat m®t nghi m (S; P) thỏa mãn S ≥ 0 P ≥ 0 ∗ M t so bieu thfíc đoi xfíng thư ng g p: x2 + y2 = (x + y)2 − 2xy = S2 − 2P x3 + y3 = (x + y)(x2 + y2 − xy) = S3 − 3SP x4 + y4 = (x2 + y2)2 − 2x2y2 = (S2 − 2P)2 − 2P 2 Ví dn 1.4. Tìm a đe h phương trình sau có nghi m duy nhat: xy + x + y = a + 1 (Đe thi HSG lớp 9 tỉnh Hà N®i năm 2009 - 2010) Bài giai. Đ t S = x + y P = xy ta có h mới S + P = a + 1 SP = a Theo định lý Viest, S và P là hai nghi m của phương trình X2 − (a + 1)X + a = 0(1). Hơn nǎa, cũng theo Viest x, y là nghi m của phương trình Y 2 − SY + P = 0(2). Do đó đe (1) có nghi m duy nhat thì (2) có nghi m duy nhat, tác ∆(2) = 0 ⇔ S2 = 4P ⇔ x = y (Ho c có the dùng nh n xét, do vai trò của x, y trong moi phương trình như nhau nên h có nghi m (m; n) thì nó cũng có nghi m (n; m). Như v y đe h có nghi m duy nhat thì x = y.) The vào ta được x2 + 2x = a + 1 x2 + 2x (a + 1) = 0 ⇔ qa (∗) Đe h có nghi m duy nhat thì h ( ) có m®t nghi m x = −2 = 1 2.1 ⇒ q 3 a 2 = −1 ⇔ a = −2. 2
  • 13. 10 Viết đề tài giá sinh viên – ZALO:0973.287.149-TEAMLUANVAN.COM 1 1 7 xyzt = 648 x y 12 z t 18 (Thi HSG TP HCM 1986 - 1987 Vòng 1) (Thi HSG TP HCM 07/01/1993) Thả lại thay thỏa mãn. V y đe h đã cho có nghi m duy nhat thì a = −2. • Trong m®t vài trường hợp, h phương trình đoi xáng loại I còn có the g p ở h phương trình phi tuyen 3 ȁn, 4 ȁn,... thỏa mãn đieu ki n: vai trò của các ȁn trong moi phương trình của h như nhau. Ví dn 1.5. Giải h phương trình x + y + z = 1 x2 + y2 + z2 = 1 x3 + y3 + z3 = 1 Ví dn 1.6. Giải h phương trình x + y + z + t = 22 + = 1 + 1 = 5 Ví dn 1.7. Giải h phương trình x + y + z = 6 x2 + y2 + z2 = 18 √ x + √ y + √ z = 4 (Thi HSG Toàn Quoc 1996 - 1997 Bảng A) 1.2.2.1.2 H phương trình đoi xfíng loại II (xem [1]) Định nghĩa 1.6. H phương trình đoi xáng loại II là h gom hai phương trình mà khi ta thay đői vị trí của hai bien thì phương trình trên thành phương trình dưới và phương trình dưới thành phương trình trên. Khi đó h phương trình có dạng f(x;y) = 0 (1) f(y;x) = 0 (2)
  • 14. 11 Viết đề tài giá sinh viên – ZALO:0973.287.149-TEAMLUANVAN.COM − x − 3y = 4 y y x y ∗ Phương pháp giải: Lay ve trà ve của hai phương trình, ta được: (y − x)h(x;y) = 0 ⇔ x = y ho c h(x;y) = 0. Ket hợp với (1), giải phương trình, tìm (x; y) Ví dn 1.8 (xem [5]). Giải h phương trình: x y 3x = 4 x y (Đe thi tuyen sinh ĐH Quoc Gia Hà N®i - 1997) Bài giai. y x (x − y) + 3(x − y) = 4( − ) x + y x − 3y = 4 x (x − y)(1 + ⇔ y ) = 0 (1) xy x − 3y = 4 x (2) (1) ⇔ x − y = 0 ho c 1 + x + y = 0 ⇔ x = y ho c x = −y . xy ٨ Với x = y, thay vào (2), ta có x − 3x = 4 ⇔ −2x = 4 ⇔ x = y = −2. y + 1 ٨ Với x = −y y + 1 ,(Đieu ki n y /= −1) thay vào (2), ta có −y y + 1 y + 1 — 3y = 4.y. −y −y y + 1 −y y + 1 — 3y + 4y + 4 = 0 + y + 4 = 0 ⇒ y2 + 4y + 4 = 0 ⇔ (y + 2)2 = 0 ⇔ y = −2.(Thỏa mãn) Thay y = −2 ⇒ x = −2 V y h phương trình đã cho có nghi m (x; y) = (−2; −2). Ví dn 1.9 (xem [5]). Giải h phương trình: x2 + y2 + xy = 37 y2 + z2 + xz = 28 y2 + z2 + yz = 19 ⇔ ⇔ Đieu ki n (x; y) (0; 0) H đã cho tương đương với
  • 15. 12 Viết đề tài giá sinh viên – ZALO:0973.287.149-TEAMLUANVAN.COM 2 2 x + 4 x y −2y = −6 2 2 2x − 13xy + 15y = 0 (2) (Thi HSG Toàn Quoc 1994 - 1995) 1.2.2.1.3 H phương trình phi tuyen v i ve trái đang cap. (xem [2]) Dạng tőng quát của h phương trình phi tuyen có ve trái đȁng cap là ax2 + bxy + cy2 = d aJx2 + bJxy + cJy2 = dJ ∗ Phương pháp giải ٨ Xét x = 0, thay vào h (1.11), ta được cy2 = d cJy2 = dJ Neu h phương trình này có nghi m thì h (1.11) có nghi m, không thì h vô nghi m. ٨ Xét x /= 0 - Neu m®t trong hai d ho c dJ bang 0. Giả sả d = 0, thì ta chia cả hai ve của phương trình thá nhat cho x2, tà đó ta thu được phương trình có dạng A( y y )2 x y + B + C = 0. x Giải phương trình b c hai, tìm tìm được y, tà đó thu được x. , tà đó rút y theo x, lại thay vào phương trình thá hai, x - Neu cả d và dJ đeu khác 0 thì ta có the tạo ra m®t phương trình thuan nhat (h so tự do bang 0), sau đó thực hi n cách giải như trên. Ví dn 1.10 (xem [2]). Giải h phương trình: x2 − 2xy + 3y2 = 9 (1) Ví dn 1.11 (xem [2]). Giải h phương trình: 2x2 − xy + 3y2 = 13 (Đe thi lớp 10 chuyên toán 31.04.1994 TP Hà N®i) Bài giai. H đã cho tương đương với h : x2 + 4xy − 2y2 = −6 25x2 + 46xy − 8y2 = 0 (1)
  • 16. 13 Viết đề tài giá sinh viên – ZALO:0973.287.149-TEAMLUANVAN.COM ∗ xy = 12 a + b = 5 xz 36 Giải phương trình (1), đ t t = y , ta có: x 8t2 − 46t − 25 = 0 ⇔ t1 = −1 2 25 ho c t2 = 4 ∗ Với t = −1 , ta có h phương trình 2 x = −2y x2 + 4xy − 2y2 = −6 25 ⇔ (x; y) = (−2; 1), (2; −1) Với t = x = 4 y , ta có h phương trình 4 4 25 −4 −25 25 x2 + 4xy − 2y2 = −6 ⇔ (x; y) = (√139 ; 139 ), (√139 ; ). 139 4 25 √ −4 −25 V y h đã cho có 4 nghi m (x; y) = (−2; 1), (2; −1), ( 139 ; √ 139 ), (√ 139 ; √ 139 ). 1.2.2.1.4 H phương trình đoi xfíng dạng phân thfíc Ví dn 1.12 (xem [2]). Giải h phương trình: x + y 5 yz 18 = y + z 5 Bài giai. 1 Đ t a = x ;b = 1 1 ; c = y z x + z 13 Khi đó h đã cho tương đương với h : 12 5 b + c = 18 a + c = 13 36 Đây là m®t h phương trình tuyen tính, giải được nghi m 1 1 (a; b; c) = ( ; ; 4 6 1 9 ) ⇒ (x; y; z) = (4; 6; 9) V y h phương trình có nghi m (x; y; z) = (4; 6; 9). Ví dn 1.13 (xem [2]). Giải h phương trình: x2(y + z)2 = (3x2 + x + 1)y2z2 y2(x + z)2 = (4y2 + y + 1)x2z2 z2(x + y)2 = (5z2 + z + 1)x2y2 = √ √
  • 17. 14 Viết đề tài giá sinh viên – ZALO:0973.287.149-TEAMLUANVAN.COM yz x x2 ( y + z )2 = 3 + 1 + 1 xz y y2 ( x + y )2 = 5 + 1 + 1 Bài giai. Neu x = 0 ⇒ y = 0; z = 0, do đó (0; 0; 0) là m®t nghi m của h . Ta tìm các nghi m khác (0; 0; 0). Chia cả hai ve cho x2y2z2 ta được h tương đương Đ t a = 1 ;b = x xy 1 1 ; c = . y z z z2 Khi đó h đã cho tương đương với (a + b)2 = c2 + c + 5 (1) (b + c)2 = a2 + a + 3 (2) (a + c)2 = b2 + b + 4 (3) Lay (2) − (3) ⇒ (a − b)(2(a + b + c) + 1) = 1 Lay (1) − (2) ⇒ (b − c)(2(a + b + c) + 1) = 1 ⇒ a − b = b − c ⇒ a + c = 2b Thay vào (2) ⇒ 3b2 − b + 4 = 0 (Vô nghi m). Suy ra h đã cho không ton tại nghi m khác (0; 0; 0) ∗ Chú ý: Hai kieu đoi xáng loại I và loại II là nhǎng dạng rat cơ bản. Tuy nhiên khi g p các bài toán giải h phương trình có nhieu hơn hai ȁn, không nên chỉ nhìn hình thác roi r p khuôn theo lời giải của dạng. 1.2.2.2 H phương trình mũ và Lôgarit (xem [2]) ∗ Các phương trình mũ và loogarit: - Phương trình: ax = b(a > 0; a /= 1), vô nghi m neu b ≤ 0; phương trình có nghi m x = loga b neu b > 0. - Phương trình loga x = b(a > 0; a /= 1) ⇔ x = ab. - Phương trình af(x) = ag(x) (a > 0) ⇔ a = 1hoca /= 1, f(x) = g(x). - Phương trình loga f(x) = loga g(x), (a > 0, a 1) ⇔ f(x) > 0 hay g(x) > 0 . f(x) = g(x) ( x + z )2 = 4 + 1 + 1
  • 18. 15 Viết đề tài giá sinh viên – ZALO:0973.287.149-TEAMLUANVAN.COM 2 x2 + 1 x y2 + 1 y xy + 1 1 ∗ Phương pháp giải: Vi c giải h phương trình mũ và logarit giong như vi c giải các h phương trình đại so với các bien đői ve bieu thác mũ và loogarit. Phương pháp chung : Rút the, c®ng đại so, đ t ȁn phụ, bien đői tích; dánh giá, bat đȁng thác, dùng đạo hàm;... đe đưa ve giải phương trình mũ và loogarit với 4 hướng thông dụng là : Đưa ve cùng m®t cơ so; đ t ȁn phụ, loogarit hóa hay mũ hóa; sả dụng tính chat của hàm so;... Ví dn 1.14 (xem [2]). Giải h phương trình: xlog8 y + ylog8 x = 4 (1) log4 x − log4 y = 1 (2) Bài giai. Đieu ki n x, y > 0 ta có: x (2) ⇔ log4 y = 1 ⇔ x = 4y nên (1): (4y)log8 y + ylog8 4y = 4 ⇔ 4log8 y.ylog8 y + ylog8 4.ylog8 y = 4 2 2 y 3 .ylog8 y + y 3 .ylog8 y 2 y3 +log8 y = 2 2 1 ⇔ ( 3 + log8 y). log8 y = 3 ⇔ 3 log8 y + 2 log8 y − 1 = 0 ⇔ log8 y = −1 ho c log8 y = 3 1 ⇔ y = 8 ho c y = 2 1 Do đó x = 2 ho c x = 8. 1 1 V y h PT có hai nghi m là (x; y) = ( ; 2 ); (8; 2). 8 Ví dn 1.15 (xem [2]). Giải h phương trình: 1 + 1 = 2 (1) Bài giai. Đieu ki n x, y > 0 log2 2 . log3 3 = 1 (2) ⇔ ⇔
  • 19. 16 Viết đề tài giá sinh viên – ZALO:0973.287.149-TEAMLUANVAN.COM − 1 x 1 1 1 2 Ta có (1) x2 + 1 + y2 + 1 = xy + 1 ⇔ (xy + 1)(x2 + y2 + 2) = 2(x2 + 1)(y2 + 1) ⇔ xy(x2 + y2) + 2xy + x2 + y2 + 2 = 2x2y2 + 2(x2 + y2) + 2 ⇔ xy(x − y)2 = (x − y)2 ⇔ (x − y)2(xy − 1) = 0 - Neu x = y thì x = y = 1 là nghi m của h phương trình. Xét trường hợp x = y /= 1 thì : (1)(log2 x − 1)(log 3x + 1) = 1 ⇔ log2 x. log3 x = log2 x + log3 x 1 ⇔ log2 1 + = 1 x log3 x ⇔ logx 2 + logx 3 = 1 ⇔ logx 6 = 1 ⇔ x = 6 - Neu xy = 1 thì y = x và x = / 1, ta có log2 2 . log3 3x = 1 ⇔ (log2 x − 1)(log3 x + 1) = −1 1 ⇔ log2 x. log3 x = log3 x − log2 x ⇔ log 2 1 = 1 x log3 x 2 ⇔ logx 2 − logx 3 = 1 ⇔ logx 3 = 1 ⇔ x = 3 . 2 2 V y h phương trình có ba nghi m là (x; y) = (1; 1), (6; 6), ( ; ). 3 3 2
  • 20. 17 Viết đề tài giá sinh viên – ZALO:0973.287.149-TEAMLUANVAN.COM Chương 2 Nhfing phương pháp thư ng dùng đe giải h phương trình phi tuyen 2.1 Phương pháp the 2.1.1. N i dung phương pháp Thông thường ta bieu dien 1 ȁn ho c 1 bieu thác thích hợp tà m®t phương trình của h theo các ȁn khác roi thay vào các phương trình còn lại đe được m®t h mới tương đương với h đã cho và có so ȁn ít hơn so với h ban đau. Ví dụ đoi với h hai phương trình hai ȁn ta có the thực hi n các bước sau: + Bước 1: Tà m®t phương trình của h đã cho, ta bieu dien m®t ȁn theo ȁn kia roi thay the vào phương trình còn lại đe được phương trình mới chỉ có m®t ȁn. + Bước 2: Dùng phương trình mới vàa có được, thay the cho m®t trong hai phương trình của h , tạo thành h mới tương đương với h phương trình ban đau. + Bước 3: Giải phương trình m®t ȁn vàa có roi suy ra nghi m của h đã cho. ∗ Chú j: - Phương trình m®t ȁn này phải giải được. - Trong quá trình giải h bang phương pháp the, ta thay xuat hi n phương trình có các h so của cả hai ȁn đeu bang 0 thì h đã cho có the vô nghi m ho c vô so nghi m. - Đoi với h gom nhieu phương trình, nhieu ȁn, vi c sả dụng phương pháp the làm giảm so ȁn trong phương trình đe được h mới tương đương với h đã cho, ta giải h mới, tìm được nghi m và thay ngươc trở lại tìm ȁn đã the.
  • 21. 18 Viết đề tài giá sinh viên – ZALO:0973.287.149-TEAMLUANVAN.COM 2 xy − z = 1 (2) z = 0 2 2 x + 3y − xy + 2x − 5y − 4 =0 (2) ⇔ − 9 9 2.1.2. Bài t p áp dnng phương pháp the Bài toán 2.1 (Xem [5]). Giải h phương trình: x + y = 2 (1) (Thi HSG TPHCM 1979 - 1980 vòng 2) Bài giai. Tà (1) ⇒ y = 2 − x, the vào (2), ta có x(2 − x) − z2 = 1 ⇔ 2x − x2 − z2 − 1 = 0 ⇔ (x2 − 2x + 1) + z2 = 0 ⇔ (x − 1)2 + z2 = 0 ⇔ x = 1 Suy ra y = 1. V y h đã cho có nghi m (x; y; z) = (1; 1; 0). Bài toán 2.2 (xem [4]). Giải h phương trình: x + 2y = 4 (1) ( Thi HSG TPHCM 1981 - 1982 vòng 2) Bài giai. Tà (1) ⇒ x = 4 − 2y, the vào phương trình (2): (4 − 2y)2 + 3y2 − (4 − 2y)y + 2(4 − 2y) − 5y = 0 9y2 29y + 20 = 0 20 ⇔ (9 − 9y)(y − ) = 0 9 20 ⇔ y = 1ho c y = 9 Với y = 1 ⇒ x = 4 − 2.1 = 2 Với y = 2 ⇒ x = 4 − 2. 20 = −4 V y h phương trình đã cho có hai nghi m (x; y) = (2; 1), ( −4 ; 20 ). 9 9
  • 22. 19 Viết đề tài giá sinh viên – ZALO:0973.287.149-TEAMLUANVAN.COM √ √ ⇔ ⇔ xy + x + y = x2 − 2y2 2.1.2.1 Phép the đại so Định nghĩa 2.1. Phép the đại so là phép the sả dụng các bieu thác đại so ho c các ȁn có trong phương trình đe bieu dien m®t ȁn qua các ȁn còn lại roi thay the vào các phương trình của h đe được m®t h mới tương đương với h ban đau, với so ȁn ít hơn. Bài toán 2.3 (xem [1]). Giải h phương trình: x 2y − y x − 1 = 2x − 2y (∀x, y ∈ R) (Đe thi TS Đại hoc - Cao đȁng - Khoi D - 2008) Bài giai. Đieu ki n x ≥ 1; y ≥ 0. xy + x + y = x2 − 2y2 ⇔ (x + y)(x − 2y − 1) = 0 ⇔ x = −y ho c x = 2y + 1 ٨ Trường hợp 1: x = −y. Vì y ≥ 0 nên x ≤ 0 (Vô lý) ٨ Trường hợp 2: x = 2y + 1, the vào phương trình thá hai, ta có: (2y + 1) √ 2y − y √ 2y = 2y + 2 ⇔ (y + 1)( √ 2y − 2) = 0 ⇔ y = −1 (loại) ho c y = 2. Với y = 2 ⇒ x = 5. V y h đã cho có nghi m (x; y) = (5; 2). Bài toán 2.4 (xem [6]). Giải h phương trình: x + y + z = 0 (1) 2x + 3y + z = 0 (2) Bài giai. (x + 1)2 + (y + 2)2 + (z + 3)2 = 14 (3) Xét x + y = −z 2x + 3y = −z coi như h phương trình hai ȁn (x; y) 2x + 2y = 2z 2x + 3y = −z y = z x = −2z The vào phương trình (3), ta có:
  • 23. 20 Viết đề tài giá sinh viên – ZALO:0973.287.149-TEAMLUANVAN.COM x + y = 4 Suy ra (1 − 2z)2 + (z + 2)2 + (z + 3)2 = 14 ⇔ 6z2 + 6z = 0 ⇔ 6z(z + 1) = 0 ⇔ z = 0 ho c z = −1. ∗ Với z = 0 ⇒ (x; y; z) = (0; 0; 0) ∗ Với z = −1 ⇒ (x; y; z) = (2; −1; −1). V y h đã cho có hai nghi m (x; y; z) = (0; 0; 0), (2; −1; −1). Bài toán 2.5 (xem [6]). Giải h phương trình ȁn x, y, u, v : x3u + y3v = 14 (1) x2u + y2v = 5 (2) xu + yv = 2 (3) u + v = 1 (4) (Thi HSG TP HCM ngày 17/12/1994 Vòng 2) Bài giai. Tà (4) ⇒ v = 1 − u. The vào (1) ux3 + y3(1 − u) = 14 ⇒ u(x3 − y3) = 14 − y3 ⇒ u(x − y)(x2 + xy + y2) = 14 − y3.(5) Tương tự The v = 1 − u vào (2), ta có u(x − y)(x + y) = 5 − y2.(6) The v = 1 − u vào (3), ta có u(x − y) = 2 − y.(7) Tà (5) và (7) suy ra (2 − y)(x2 + xy + y2) = 14 − y3. Tà (6) và (7) suy ra (2 − y)(x + y) = 5 − y2. Khai trien phương trình cuoi, ta có 2(x + y)2 − xy(x + y) − 2xy − 14 = 0 2(x + y) − xy − 5 = 0 Đ t x + y = a, xy = b suy ra 2a2 − ab − 2b − 14 = 0 2a − b − 5 = 0 xy = 3 ⇔ a = 4 b = 3 Suy ra (x; y) = (3; 1) ho c (1; 3).
  • 24. 21 Viết đề tài giá sinh viên – ZALO:0973.287.149-TEAMLUANVAN.COM y = cos α (α ∈ [0; 2Π]) √ √ √ √ The x; y vào, ta tìm được các nghi m của h : 1 (x; y; u; v) = (3; 1; ; 2 1 1 1 ), (1; 3; ; ). 2 2 2 2.1.2.2 Phép the lư ng giác Định nghĩa 2.2. Phép the lượng giác là phép the sả dụng các bieu thác là các hàm lượng giác thay the cho các ȁn có trong h phương trình đe được m®t h phương trình mới tương đương với h đã cho nhưng các ȁn được thay the hoàn toàn bang ȁn mới. Bài toán 2.6 (xem [6]). Giải h phương trình x2 + y2 = 1 (1) √ 2(x − y)(1 + 4xy) = √ 3 (2) Bài giai. Tà phương trình (1) cho ta gợi ý đ t ȁn phụ đưa ve lượng giác. Đ t x = sin α Khi đó phương trình (2) được viet dưới dạng: 6 (sin α − cos α)(1 + 2 sin 2α) = 2 6 ⇔ sin α − cos α + 2 sin 2α sin α − 2 sin 2α cos α = 2 6 ⇔ sin α − cos α + cos α − cos 3α − sin 3α − sin α = 2 ⇔ sin 3α + cos 3α = − √ 6 Π ⇔ cos(3α + 4 ) = 2 — 3 = cos 5Π 2 6 7Π k2Π ⇔ α = + ho c α = −13Π + k2Π (k ∈ Z) 36 3 36 7Π 7Π 7Π 3 31Π 55Π 11Π 35Π 59Π Vì α ∈ [0; 2Π] nên α ∈ { 36 ; 36 ; 36 ; 36 ; ; ; 36 36 ; 36 36 7Π 7Π V y h có nghi m (x; y) = (sin α; cos α) với α ∈ { 36 ; 36 ; Bài toán 2.7 (xem [5]). Giải h phương trình 7Π 31Π ; ; 36 36 55Π ; 36 11Π ; 36 35Π ; 36 59Π 36 }. z2 + 2xyz = 1 (1) 3x2y2 + 3xy2 = 1 + x3y4 (2) z + zy4 + 4y3 = 4y + 6y2z (3) }
  • 25. 22 Viết đề tài giá sinh viên – ZALO:0973.287.149-TEAMLUANVAN.COM − − − Bài giai. Vì z = 0 không là nghi m của h phương trình nên: 1 − z2 (1) ⇔ xy = 2z Đ t z = tan ϕ(∗) với ϕ ∈ −Π ; Π {0} Ta có xy = 1 − z2 2z 2 1 tan2 ϕ = 2 tan ϕ 2 = cot 2ϕ Thay vào (2) ta được: 3 cot2 2ϕ + 3y cot 2ϕ = 1 + y cot3 2ϕ 3 cot2 2ϕ − 1 1 ⇔ y = cot3 2ϕ 3 cot 2ϕ = cot 6ϕ = tan 6ϕ Suy ra: x = cot 2ϕ. cot 6ϕ, thay vào (3) ta được: 4 tan 6ϕ 4 tan3 6ϕ z = 1 − 6 tan2 6ϕ + tan4 6ϕ = tan 24ϕ(∗∗) Tà (∗) và (∗∗) ta có: tan 24ϕ = tan ϕ ⇔ 24ϕ = ϕ + kΠ, k ∈ Z ⇔ ϕ = Với ϕ ∈ −Π ; Π {0} ta thu được : kΠ 23 , k ∈ 2 2 Π 2Π 11Π ϕ = ± 23 ; ± 23 ; . . . ; ± 23 V y h phương trình có các nghi m là : Π 2Π 11Π (x; y; z) = (cot 2ϕ. cot 6ϕ; tan 6ϕ; tan ϕ) Với ϕ ∈ ± 23 ; ± 23 ; . . . ; ± 23 Bài toán 2.8 (xem [5]). Giải h phương trình 2z(x + y) + 1 = x2 − y2 (1) y2 + z2 = 1 + 2xy + 2xz − 2yz (2) y(3x2 − 1) = −2x(x2 + 1) (3) Bài giai. 1 Vì x = ±√ 3 không thỏa mãn phương trình (3) nên: −2(x2 + 1) 3x3 − x − 2x(x2 + 1) x3 − 3x (3) ⇔ y = 3x2 − 1 ⇔ x + y = 3x2 − 1 ⇔ x + y = 3x2 − 1 Z
  • 26. 23 Viết đề tài giá sinh viên – ZALO:0973.287.149-TEAMLUANVAN.COM − — − ⇔ − − − − Π sin 6ϕ cos2 ϕ 22 11 2 22 11 2 2 2 6 6 22 22 22 22 22 Đ t x = tan ϕ, ϕ ∈ − Π ; Π {− Π ; Π } 2 2 6 6 ⇒ cos ϕ /= 0; cos 3ϕ /= 0 Ta có: tan ϕ + y = tan3 ϕ 3 tan ϕ 3 tan2 ϕ − 1 ⇔ y = tan 3ϕ − tan ϕ (1) ⇔ z = x2 y 1 2(x + y) (do x = −y không thỏa mãn phương trình (1) ⇒ tan 3ϕ = / 0) z = (2 tan ϕ − tan 3ϕ) tan 3ϕ − 1 2 tan 3ϕ 2 tan ϕ. tan 3ϕ − tan2 3ϕ − 1 z = z = tan ϕ − 2 tan 3ϕ tan 3ϕ + cot 3ϕ 2 z = tan ϕ − 1 sin 3ϕ ( + 2 cos 3ϕ 1 cos 3ϕ ) sin 3ϕ z = tan ϕ − sin 6ϕ (2) ⇔ x2 + y2 + z2 − 2xy − 2xz + 2yz = 1 + x2 ⇔ (y + z − x)2 = 1 + x2 (tan 3ϕ tan ϕ + tan ϕ 1 tan ϕ)2 = 1 + tan2 ϕ sin 6ϕ ⇔ ( sin 3ϕ − 1 − tan ϕ)2 = 1 cos 3ϕ 2 sin3 ϕ cos 3ϕ cos2 ϕ 2 sin2 3ϕ 1 2 ⇔ ( 2. sin 3ϕ. cos 3ϕ − tan ϕ) 1 = cos2 ϕ ⇔ ( cos 6ϕ + tan ϕ)2 = 1 ⇔ ( cos 5ϕ )2 = 1 sin 6ϕ. cos ϕ cos2 ϕ ⇔ cos 5ϕ = ± sin 6ϕ ⇔ cos 5ϕ = ± cos( 2 − 6ϕ) Π Π ⇔ cos 5ϕ = cos( 2 − 6ϕ) ho c cos 5ϕ = cos( 2 + 6ϕ) Π Π ⇔ 5ϕ = ±( 2 − 6ϕ) + k2Π ho c 5ϕ = ±( 2 + 6ϕ) + k2Π Π k2Π Π −Π k2Π −Π ⇔ ϕ = + , ϕ = − 2kΠ ho c ϕ = + , ϕ = − 2kΠ(k ∈ Z) Với ϕ ∈ − Π ; Π {− Π ; Π } ⇒ ϕ = ± Π ; ± 3Π ; ± 5Π ; ± 7Π ; ± 9Π V y h phương trình đã cho có nghi m: 1 (x; y; z) = (tan ϕ; tan 3ϕ − tan ϕ; tan ϕ − sin 6ϕ ) ⇔
  • 27. 24 Viết đề tài giá sinh viên – ZALO:0973.287.149-TEAMLUANVAN.COM 1 1 1 1 1 1 tan α tan β + tan β tan γ + tan γ tan α = 1 (3) Π 3Π 5Π 7Π 9Π Với ϕ = ± 22 ; ± 22 ; ± 22 ; ± 22 ; ± 22 Bài toán 2.9 (xem [1]). Giải h phương trình xy + yz + xz = 1 (1) Bài giai. 20(x + x ) = 11(y + y ) = 2007(z + z ) (2) Đieu ki n : xyz /= 0 Neu (x; y; z) là m®t nghi m của h phương trình thì (−x; −y; −z) cũng là m®t nghi m của h , và tà (1) suy ra x, y, z cùng dau. Ta chỉ can xét (x; y; z) dương. Với ∀x, y, zR và khác 0, ta đ t: x = tan α y = tan β z = tan γ Với 0 < α, β, γ < 2 Tà đó (1) và (2) trở thành: 20(tan α + tan α ) = 11(tan β + tan β ) = 2007(tan γ + tan γ ) (4) Ta có: (3) ⇔ tan α(tan β + tan γ) = 1 − tan α tan γ tan α = 1 − tan β tan γ tan β + tan γ Π = cot(β + γ) ⇔ α + β + γ = 2 ⇔ 2α, 2β, 2γ là các góc của m®t tam giác. (4) ⇔ 20 tan2 α + 1 = 11 tan α tan2 β + 1 = 2007 tan β tan2 γ + 1 tan γ 20 11 2007 ⇔ sin 2α = sin 2β = sin 2γ Áp dụng định lý sin ta tính được ba cạnh của tam giác có ba góc 2α, 2β, 2γ là a = 20 b = 11 c = 2007 (Không thỏa mãn bat đȁng thác tam giác) Do đó tam giác không ton tại. V y h vô nghi m. Π ⇔ (∀x, y ∈ R)
  • 28. 25 Viết đề tài giá sinh viên – ZALO:0973.287.149-TEAMLUANVAN.COM ut = 6 2.2 Phương pháp đ t an phn Đoi với m®t so bài toán phác tạp, bang cách đ t ȁn phụ, sě làm bài toán trở lên de dàng hơn. (Thường áp dụng đoi với nhǎng h phương trình xuat hi n cụm ȁn nào đó được l p lại nhieu lan.) 2.2.1. N i dung phương pháp Điem mau chot của phương pháp này là phải phát hi n ȁn phụ u = f(x,y) và v = g(x,y) ngay trong tàng phương trình của h ho c sau các phép bien đői. Thông thường các phép bien đői thường xoay quanh vi c c®ng, trà hai phương trình của h ho c chia cả hai ve của tàng phương trình cho các so hạng khác không có sȁn trong các phương trình của h đe tìm ra phan chung mà sau đó ta đ t ȁn phụ đe đưa h phương trình ban đau ve m®t h phương trình mới tương đương với h đã cho. Tìm ȁn phụ và thay the ngược trở lại đe tìm được nghi m của phương trình ban đau. 2.2.2. Bài t p áp dnng phương pháp đ t an phn Bài toán 2.10 (xem [6]). Giải h phương trình: x2 − y2 + x − y = 5 x3 − x2y − xy2 + y3 = 6 Bài giai. Bien đői h đã cho ⇔ (x2 − y2) + (x − y) = 5 (x2 − y2)(x − y) = 6 Đ t u = x2 − y2 và t = x − y; ta có h phương trình u + t = 5 ⇒ u và t là hai nghi m của phương trình X2 − 5X + 6 = 0 ⇔ (X − 2)(X − 3) = 0 ⇔ X = 2 ho c X = 3. Xét hai trường hợp: - Trường hợp 1: u = 2, t = 3
  • 29. 26 Viết đề tài giá sinh viên – ZALO:0973.287.149-TEAMLUANVAN.COM 3 2 x y − x − xy = 1 ⇔ Đ t u = −x2 + xy, v = x3y. H trở th ành: x2 − y2 = 2 x − y = 3 x + y = 3 2 x − y = 3 x = 11 6 . y = −7 6 - Trường hợp 2:u = 3, t = 2 x2 − y2 = 3 x − y = 2 x + y = 3 2 x − y = 2 x = 7 4 . y = −1 4 V y h đã cho có hai nghi m (x; y) = ( 11 ; −7 ), ( 7 ; −1 ). 6 6 4 4 Bài toán 2.11 (xem [1]). Giải h phương trình: x4 − x3y2 + x2y2 = 1 (Đe dự bị khoi A - 2007) Bài giai. H đã cho tương đương với (−x2 + xy)2 + x3y2 = 1 (−x2 + xy) − x3y = 1 u2 + v = 1 u + v = 1 u2 − u = 0 u + v = 1 ⇔ u(u − 1) = 0 u + v = 1 u = 1 v = 0 ho c u = 0 . v = 1 Xét các trường hợp: - Trường hợp 1: u = 0; v = 1 ⇔ −x2 + xy = 0 x = 0; x = y ⇔ x = y ⇔ x = y x3y = 1 y = x3 x4 = 1 x = ±1 - Trường hợp 2: u = 1; v = 0 −x2 + xy = 1 x3y = 0 ⇒ y = 0 x2 = −1 (Vô nghi m) V y h đã cho có hai nghi m (x; y) = (1; 1), (−1; −1). Bài toán 2.12 (xem [1]). Giải h phương trình x2 + y2 + x + y = 4 x(x + y + 1) + y(y + 1) = 2 1 ⇒ ⇒ ⇔ ⇒ ⇔ ⇔ ⇔ ⇒ ⇒
  • 30. 27 Viết đề tài giá sinh viên – ZALO:0973.287.149-TEAMLUANVAN.COM 2 2 2 x + y= S − 2P P = xy = −2 (Đe thi dự bị - Khoi A - Năm 2005) Bài giai. H đã cho tương đương với x2 + y2 + x + y = 4 x2 + y2 + x + y + xy = 2 x2 + y2 + x + y = 4 xy = −2 Đ t x + y = S xy = P suy ra S2 = x2 + y2 + 2xy S2 − 2P + S = 4 P = −2 H đã cho tương đương với ∗Trường hợp 1 S2 − P + S = 2 S = 0 ho c S = −1 S = x + y = 0 P = xy = −2 ⇔ x, y là nghi m của phương trình X2 − 0X − 2 = 0 Suy ra (x; y) = ( √ 2; − √ 2), (− √ 2; √ 2). ∗ Trường hợp 2 S = x + y = −1 ⇔ x, y là nghi m của phương trình Y 2 + Y − 2 = 0 ⇔ Y = 1 ho c Y = −2 Suy ra (x; y) = (1; −2) ho c (−2; 1). V y h đã cho có bon nghi m (x; y) = ( √ 2; − √ 2), (− √ 2; √ 2), (1; −2), (−2; 1). Bài toán 2.13 (xem [1]). Giải h phương trình x2 − xy + y2 = 3(x − y) x2 + xy + y2 = 7(x − y)3 (∀x, y ∈ R) Bài giai. Đ t x + y = u . xy = v H đã cho trở thành u2 − 3u + v = 0 v = 2u2 3u2 − 3u = 0 v = 2u2 Suy ra u = 0 ho c u = 1 ∗Trường hợp 1 u = 0 ⇔ x − y = 0 ⇔ x = 0 v = 0 xy = 0 y = 0 (Đe thi dự bị 1 - Khoi D - Năm 2006) ⇔ ⇔ ⇔
  • 31. 28 Viết đề tài giá sinh viên – ZALO:0973.287.149-TEAMLUANVAN.COM 2 2 x + 4 x y −2y = −6 −1 x = y 25 √ ∗Trường hợp2 u = 1 ⇔ x − y = 1 ⇔ x = 2 Ho c x = −1 v = 2 xy = 2 y = 1 y = −2 V y h đã cho có nghi m (x; y) = (0; 0), (2; 1), (−1; −2). Bài toán 2.14 (xem [6]). Giải h phương trình 2x2 − xy + 3y2 = 13 (Đe thi lớp 10 chuyên toán 31/07/1994) Bài giai. H đã cho tương đương với x2 + 46xy − 8y2 = 0 x2 + 4xy − 2y2 = −6 x2 + 4xy − 2y2 = −6 x2 + 4xy − 2y2 = −6 ⇔ t = x ⇔ x = yt y 8t2 − 46t − 25 = 0 t = −1 ho c t = 25 2 4 ∗ Trường hợp 1 2 x2 + 4xy − 2y2 = −6 ∗Trường hợp 2 ⇔ (x; y) = (−2; 1), (2; −1). x = 4 y 4 ⇔ (x; y) = (√ ; 25 ), (√ −4 ; √ −25 ). x2 + 4xy − 2y2 = −6 139 139 4 √ 139 25 139 −4 −25 V y h có bon nghi m ?(x; y) = (−2; 1), (2; −1), ( 139 ; √ 139 ), (√ 139 ; √ 139 ).
  • 32. 29 Viết đề tài giá sinh viên – ZALO:0973.287.149-TEAMLUANVAN.COM Bài toán 2.15 (xem [5]). Cho x; y là hai so nguyên dương sao cho xy + x + y = 71 x2y − xy2 = 880 Tìm giá trị của bieu thác M = x2 + y2 (Thi HSG Đại Hoc IOWA Mĩ 04/1991)
  • 33. 30 Viết đề tài giá sinh viên – ZALO:0973.287.149-TEAMLUANVAN.COM xy = P ∗ S = 55 S.P = 880 H đã cho tương đương với xy + (x + y) = 71 Bài giai. Đ t x + y = S Đieu ki n S2 − 4P ≥ 0 xy(x + y) = 880 H đã cho tương đương với S + P = 71 Suy ra S và P là hai nghi m của phương trình X2 − 71X + 880 = 0 ⇒ X1 = 55; X2 = 16. Xét hai trường hợp: Trường hợp 1 P = 55 S = 16 Ta có S2 − 4P = 162 − 4.55 = 36 > 0(Thỏa mãn) Suy ra x, y là hai nghi m của phương trình A2 − 16A + 55 = 0 ⇔ A1 = 11, A2 = 5 (∃(x; y) thỏa mãn đieu ki n x, y nguyên dương) Ta có M = x2 + y2 = (x + y)2 − 2xy M = 162 − 2.55 = 146. ∗ Trường hợp 2 P = 16 Ta có S2 − 4P = 552 − 4.16 = 2961 > 0 Suy ra x, y là hai nghi m của phương trình B2 − 55B + 16 = 0 Ta có ΔB = 2961 > 0 ⇒ ∃(x; y). Nhưng √ o là so vô tỉ nên không thỏa mãn đieu ki n x, y nguyên dương của bài toán. V y M = 146. Bài toán 2.16 (xem [5]). Giải h phương trình x2 + y + x3y + xy2 + xy = −5 4 x4 + y2 + xy(1 + 2x) = −5 4 Bài giai. Đ t x2 + y = u và xy = v thì h đã cho tương đương với u + v + uv = −5 4 u2 + v = −5 4 (1) (1)
  • 34. 31 Viết đề tài giá sinh viên – ZALO:0973.287.149-TEAMLUANVAN.COM − 4 ⇔ r x( 1 − x2 = ) x3 + 1 x − y = 2 2 2 2 4 r 16 − r ⇔ 25 Tà (2) suy ra v = −5 u2, thay vào phương trình (1), ta có 4 5 u − 4 − u2 + u( 5 − 4 − u3) = −5 4 ⇔ 4u3 + 4u2 + u = 0 ⇔ u(2u + 1)2 = 0 u = 0 ho c u = −1 2 Xét hai trường hợp Trường hợp 1: Neu u = 0 thì v = −5 4 ta có x2 + y = 0 xy = −5 4 y = −x2 −x3 = −5 x = r 3 5 y = − 3 Trường hợp 2: Với u = −1 2 thì v = −3 , ta có h phương trình 2 x2 + y = −1 y = −1 − x2 y = −1 − x2 x = 1 2 2 − 2 −3 3 −3 V y h đã cho có hai nghi m (x; y) = (1; −3 ), ( 3 2 5 ; 3 25 ). 16 2.3 Phương pháp sfi dnng tính đơn đi u của hàm so(xem [2]) Định nghĩa 2.3. Hàm so f(x) xác định trên [a, b] được goi là tăng (tương áng tăng nghiêm ng t) neu với ∀x1, x2 ∈ [a, b] và x1 < x2 ta có f(x1) ≤ f(x2) (tương áng f(x1) < f(x2)). Tương tự được goi là giảm (tương áng giảm nghiêm ng t) neu với ∀x1, x2 ∈ [a, b], x1 < x2 ta có f(x1) ≥ f(x2) (tương áng f(x1) > f(x2)). Nhǎng hàm so tăng ho c giảm trên [a, b] được goi là đơn đi u trong đoạn đó. Với trường hợp tăng nghiêm ng t ho c giảm nghiêm ng t thì được goi là đơn đi u nghiêm ng t. Định nghĩa 2.4. Cho hàm so y = f(x) xác định trên D và m®t điem x0 ∈ D. Hàm so f(x) được goi là liên tục tại điem x0 neu lim x→x0 f(x) = f(x0). Hàm so y = f(x) liên tục trên D neu nó liên tục tại moi điem thu®c D. Định lj 2.1. Neu hàm so f(x) liên tục và đơn đi u trên (a, b) và f(a).f(b) < 0 thì phương trình f(x) = 0 có nghi m duy nhat thu®c (a, b). 2 ⇔ xy = −3 ⇔ ∗ ∗ ⇔ 4 = 0 2 ⇔
  • 35. 32 Viết đề tài giá sinh viên – ZALO:0973.287.149-TEAMLUANVAN.COM Định lj 2.2. Neu hàm so f(x) liên tục trên (a, b) và f(a).f(b) < 0 thì phương trình f(x) = 0 có ít nhat m®t nghi m thu®c (a, b). Định lj 2.3. Giả sả hàm so y = f(x) đơn đi u và liên tục trên khoảng (a; b). Khi đó ta có f(u) = f(v) ⇔ u = v((u; v) ∈ (a; b)) 2.3.1. N i dung phương pháp Bước 1: Thực hi n các phéo bien đői tương đương trên các phương trình của h đe được m®t phương trình có dạng f(u) = f(v) ho c f(u) = 0. Bước 2: Đ t f(t) = f(u) ho c f(v), cháng minh sự đơn đi u và liên tục của hàm so f(t) trên t p xác định của f(t). Bước 3: Sả dụng các tính chat của hàm đơn đi u nêu trên đoi với f(t) đe tìm được moi liên h giǎa x và y đe giải h phương trình ban đau. • Chú j: phương pháp này được sả dụng trong các trường hợp sau: - Loại I: M®t phương trình của h có dạng f(x) = f(y). M®t phương trình cho ta biet t p giá trị của x ho c y. Tà đó ta suy ra hàm f(x) đơn đi u, suy ra x = y. - Loại II: H đoi xáng mà sau khi bien đői thường đưa ve dạng f(x) = f(y) ho c f(x) = 0. Trong đó f là hàm đơn đi u. 2.3.2. Bài t p sfi dnng tính đơn đi u của hàm so Bài toán 2.17 (xem [5]). Giải h phương trình: 36x2y − 60x2 + 25y = 0 36xy2 − 60y2 + 25x = 0 Bài giai. H phương trình đã cho tương đương với x = 60y2 36y2 + 25 60x2 Suy ra x, y không âm. y = 36x2 + 25 Neu x = 0 thì y = 0, suy ra (0; 0) là m®t nghi m của h phương trình. 60t2 Neu x > 0 thì y > 0. Xét hàm so f(t) = 36t2 + 25 , t > 0. Ta có f( J t) 3000t = (36t2 + 25)2 > 0, ∀t > 0.
  • 36. 33 Viết đề tài giá sinh viên – ZALO:0973.287.149-TEAMLUANVAN.COM 6 y √ − 3 ex + √ x − 2007 = 0 − Do đó f(t) đong bien trên (0; +∞). H phương trình được viet lại x = f(y) y = f(x) Tà tính đong bien của f(t) suy ra x = y. Thay vào h phương trình, ta được x(36x2 − 60x + 25) = 0 ⇔ x(6x − 5)2 = 0 ⇔ x = 0 ho c x = 5 . 5 5 Với x = 6 ⇒ y = x = 6 5 5 V y h phương trình có hai nghi m là (x; y) = (0; 0), ( ; ). 6 6 Bài toán 2.18 (xem [5]). Cháng minh rang h phương trình ex = 2007 − √ y y2 1 x e = 2007 x2 − 1 có đúng hai nghi m thỏa mãn đieu ki n x > 0; y > 0 (Đe thi dự bị 1 - Khoi B - Năm 2007) Bài giai. Đ t f(t) = et, g(t) t = t2 − 1 −1 Suy ra g( J t) = < 0, ∀ | t |> 1. (t2 − 1)2 Ta có hàm so f tăng nghiêm ng t trên tàng khoảng xác định, và hàm so g giảm nghiêm ng t trên tàng khoảng xác định. H đã cho tương đương với f(x) + g(y) = 2007 f(y) + g(x) = 2007 ⇒ f(x) + g(y) = f(y) + g(x)(∗) Neu x > y thì f(x) > f(y) g(x) < g(y) Do (∗) suy ra y > x (do g giảm nghiêm ng t) (Vô lí). Tương tự y > x cũng dan đen vô lí. Do v y h đã cho tương đương với x = y x2 − 1 (2) √
  • 37. 34 Viết đề tài giá sinh viên – ZALO:0973.287.149-TEAMLUANVAN.COM ∞ x − Xét h(x) = ex + √ x x2 − 1 — 2007, (| x |> 1) Neu x < −1 thì h(x) < e−1 − 2007 < 0 suy ra h vô nghi m. Neu x > 1 thì h(x) = ex + √ −3 x có x2 − 1 hJ (x) = ex − (x2 − 1) 2 3 −5 3x h”(x) = ex + (x2 1) 2 2 .2x = ex + 5 > 0 (x2 − 1)2 Và lim x→1+ h(x) = + ; lim →+∞ h(x) = +∞ V y h(x) liên tục và có đo thị là đường cong lõm trên (1; +∞). Do đó đe cháng minh (2) có hai nghi m dương, ta chỉ can cháng minh ∃x0 > 1 mà h(x) < 0. Chon x0 = 2 suy ra h(2) = e2 x1 > 1, x2 > 1. 2 + √ 3 − 2007 < 0. Suy ra h(x) = 0 có đúng hai nghi m Bài toán 2.19 (xem [5]). Giải h phương trình x2 − 2x + 1 = 2y y2 − 2y + 1 = 2x Bài giai. Ta có 2y = x2 − 2x + 1 = (x + 1)2 ≥ 0. Tương tự x ≥ 0. Đ t f(t) = t2 − 2t + 1, t ≥ 0, thì f( J t) = 2t − 2 = 2(t − 1) nên fđong bien trên (1; +∞) và nghịch bien trên (0; 1). Đ t g(t) = 2t, (t ≥ 0) thì g( J t) = 2 thì g đong bien trên (1; +∞). Ta có h phương trình f(x) = g(y) g(x) = f(y) Giả sả x = min {x; y}. Xét x ≤ y. - Neu x > 1 thì 1 < x ≤ y ⇒ f(x)≤f(y) ⇒ g(y) ≤ g(x) ⇒ y ≤ x. Do đó y = x Ta có phương trình t2 − 4t + 1 = 0 nên chon nghi m x = y = 2 + √ 3. - Neu 0 ≤ x ≤ 1 thì f(0) ≥ f(x) ≥ f(1) ⇒ 0 ≤ f(x) ≤ 1 Nên 0 ≤ g(x) ≤ 1 ⇒ 0 ≤ y ≤ 1 ⇒ f(0) ≥ f(y) ≥ f(1) ⇒ 0 ≤ f(y) ≤ 1 ⇒ 0 ≤ g(x) ≤ 1 ⇒ 0 ≤ x ≤ 1. Do đó, xét x ≤ y ⇒ f(x) ≥ f(y) ⇒ g(y) ≥ g(x) ⇒ y ≥ x. Suy ra x = y. Ta có phương trình t2 − 4t + 1 = 0 nên chon nghi m x = y = 2 − √ 3.
  • 38. 35 Viết đề tài giá sinh viên – ZALO:0973.287.149-TEAMLUANVAN.COM √ + √ (1 − y) √ x − y + x = 2 + (x − y − 1) √ y (1) (4x2 + 1)x + (y − 3) √ 5 − 2y = 0 V y h có hai nghi m x = y = 2 + √ 3; x = y = 2 − √ 3. Bài toán 2.20 (xem [5]). Giải h phương trình Bài giai. 2y2 − 3x + 6y + 1 = 2 √ x − 2y − √ 4x − 5y − 3 (2) Đieu ki n x − y ≥ 0; y ≥ 0; x − 2y ≥ 0; 4x − 5y − 3 ≥ 0. (1)(1 − y) √ x − y + (x − y − 1) + (y − 1) − (x − y − 1) √ y = 0 ⇔ (1 − y)( √ x − y − 1) + (x − y − 1)(1 − √ y) = 0 (1 − y)(x − y − 1) x − y + 1 (x − y − 1)(1 − y) = 0 1 + y 1 1 ⇔ (1 − y)(x − y − 1)(√ x − y + 1 + 1 + √ y ) = 0 ⇔ (1 − y)(x − y − 1) = 0 ⇔ y = 1 ho c x = y + 1 ∗ Khi y = 1, (2) ⇔ 9 − 3x = 2 ⇔ 9 − 3x = 0 ⇔ x = 3 x − 2 − 4x − 8 ∗ Khi x = y + 1 (2) ⇔ 2y2 + 3y − 2 = 2 √ 1 − y − √ 1 − 2y ⇔ 2y2 + 3y − 2 = √ 1 − y(∗) ⇔ 2y2 + y = 2( √ 1 − y)2 + √ 1 − y(∗∗) Xét f(t) = 2t2 + t, t ≥ 0, f( J t) = 4t + 1 > 0 nên f(t) đong bien trên [0; +∞). Do đó (∗∗) ⇔ y = √ 1 − y ⇔ y2 + y − 1 = 0 ⇔ y = −1 + √ 5 ho c y = −1 − √ 5 (loại) Neu y = −1 + √ 5 2 thì x = 1 + √ 5 . 2 1 + √ 5 2 2 −1 + √ 5 V y h đã cho có hai nghi m (x; y) = (3; 1), ( ; ) 2 2 Bài toán 2.21 (xem [5]). Giải h phương trình Bài giai. 3 4x2 + y2 + 2 √ 3 − 4x = 7 5 Đieu ki n x ≤ 4 ; y ≤ 2 . Ta có (4x2 + 1)x + (y − 3) √ 5 − 2y = 0 ⇔ (4x2 + 1).2x = (5 − 2y − 1) √ 5 − 2y √ √ ⇔
  • 39. 36 Viết đề tài giá sinh viên – ZALO:0973.287.149-TEAMLUANVAN.COM 2 4 √ ∞ ⇔ 2x = √ 5 − 2y ⇔ x ≥ 0 x2(1 + y2) + y2(1 + x2) = 4 √ xy Vì x − √ 1 + x2 < 0 và 1 − √ 1 + y2 < r 0 nên suy ra y > 0. Do đó x > 0 √ x x x2 Ta có f(t) = 1 − √ 1 + t2 − 1 + x2 x2 x2 Xét hàm so f(t) = (t2 + 1)t với ∀t ∈ R thì f( J t) = 3t2 + 1 > 0 nên f đông bien trên R, do đó (4x2 + 1).2x = (5 − 2y − 1) √ 5 − 2y ⇔ f(2x) = f( √ 5−2y) y = 5 − 4x2 5 − 4x2 2 The y = 2 vào phương trình sau ta được 4x2 + ( 5 − 2x2)2 + 2 √ 3 − 4x = 7 3 Với x = 0; x = 4 3 thì không thỏa mãn nên ta chỉ xét khi 0 < x < 4 Thì g( J x) 5 = 8x − 8x( 2 ) 4 — √ 3 − 3 4x = 4x(4x2 − 3) − √ 4 3 − 4x < 0. Nên g(x) nghịch bien trên (0; ), mà g 1 ( ) 2 = 7 nên phương trình sau có nghi m duy nhat 1 x = , suy ra y = 2 : Chon 2 1 V y h có nghi m (x; y) = ( ; 2). 2 Bài toán 2.22 (xem [5]). Giải h phương trình Bài giai. Đieu ki n xy ≥ 0. x2y √ 1 + y2 − √ 1 + x2 = x2y − x Phương trình thá hai của h tương đương với x − √ 1 + x2 = x2y(1 − 1 + y2) Ta có x = 0 không thỏa mãn phương trình 1 1 1 √ Phương trình tương đương với − 1 + = y − y 1 + y2 Xét hàm so f(t) = t − t 1 + t2 trên (0; +∞) J t2 √ √ Suy ra hàm so nghịch bien trên (0; + ). 1 Phương trình f 1 ( ) x = f(y) ⇔ x = y ⇔ xy = 1. Thay vào phương trình thá nhat của h , ta có: x2(1 + 1 ) + 1 (1 + x2) = 4 ⇔ x2 + 1 = 2 t < 0, ∀t ∈ (0; +∞).
  • 40. 37 Viết đề tài giá sinh viên – ZALO:0973.287.149-TEAMLUANVAN.COM √ − − 2 √ x + 1 + √ 4 x − 1 − √ y4 + 2 = 0 3. |a| + |b| ≥ |a + b|. Dau ” = ” xảy ra khi a b ≥ 0. ⇔ (x2 − 1)2 = 0 ⇔ x2 = 1 ⇔ x = ±1. Ket hợp kieu ki n, suy ra x = y = 1. V y h đã cho có nghi m (x; y) = (1; 1). Bài toán 2.23 (xem [5]). Giải h phương trình Bài giai. Đieu ki n x ≥ 1. x2 + 2x(y − 1) + y2 − 6y + 1 = 0 Ta có x2 + 2(y − 1)x + y2 − 6y + 1 = 0 ⇔ (x + y − 1)2 − 4y = 0 ⇔ 4y = (x + y − 1)2(∗) nên y ≥ 0. Và √ x + 1 + √ 4 x 1 y4 + 2 = y ⇔ √ x + 1 + √ 4 x − 1 = √ (y4 + 1) + 1 + √ 4 (y4 + 1) − 1(∗∗) Đ t f(t) = √ t + 1 + √ 4 t − 1 thì f(t) đong bien trên [1; +∞) Nên (∗∗) ⇔ f(x) = f(y4)+1 ⇔ x = y4 + 1 The vào (∗) ta có 4y = (y4 + y)2 = y8 + 2y5 + y2 ⇔ y = 0 ho c y7 + 2y4 + y = 4 ⇔ y = 0, x = 1 ho c y = 1, x = 2. Vì g(y) = y7 + 2y4 + y đong bien trên [0; +∞) và g(1) = 4 V y nghi m của h phương trình là (x; y) = (1; 0) ho c (2; 1). 2.4 Phương pháp sfi dnng bat đang thfíc 2.4.1. Các bat đang thfíc cơ bản thư ng g p 1. |A| ≥ A. Dau ” = ” xảy ra khi A = 0. 2. a2 ≥ 0. Dau ” = ” xảy ra khi a = 0. 4. |a| − |b| ≤ |a − b|. Dau ” = ” xảy ra khi ab ≥ 0 . |a| ≥ |b| 5. a2 + b2 ≥ 2ab. Dau ” = ” xảy ra khi a = b. 6. (a + b)2 ≥ 4ab ⇔ ab ≤ ( a + b )2. Dau ” = ” xảy ra khi a = b.
  • 41. 38 Viết đề tài giá sinh viên – ZALO:0973.287.149-TEAMLUANVAN.COM ≥ bn √ √ √ √ bn 1 2 n 1 2 n Dạng 2: |a1b1 + a2b2 + · · · + anbn| ≤ √ (a2 + a2 + · · · + a2 )(b2 + b2 + · · · + b2 ) Dạng 3: a1b1 + a2b2 + · · · + anbn ≤ √ (a2 + a2 + · · · + a2 )(b2 + b2 + · · · + b2 ) 1 1 2 2 n n 1 1 4 7. a + b ≥ a + b (a; b ≥ 0). Dau ” = ” xảy ra khi a = b. a b 8. b + a ≥ 2(ab > 0). Dau ” = ” xảy ra khi a = b. 9. Bat đȁng thác Caushy (AM - GM) Với n so thực dương a1; a2; · · · ; an. Dạng 1: a1 + a2 + · · · + an n ≥ √ n a1 .a2. · · · .an Dạng 2: a1 + a2 + · · · + an ≥ n. √ n a1.a2. · · · .an Dạng 3: ( a1 + a2 + · · · + an )n a n .a2 . · · · .an Dau ” = ” xảy ra khi a1 = a2 = · · · = an 11. Bat đȁng thác BCS (Bunhiakovsky). Với hai b® so thực bat kì: (a1; a2; · · · ; an); (b1; b2; · · · ; bn) Dạng 1: (a1b1 + a2b2 + · · · + anbn)2 ≤ (a2 + a2 + · · · + a2 )(b2 + b2 + · · · + b2 ) Dau ” = ” xảy ra khi a1 = a2 1 2 = · · · = an n 1 2 n b1 b2 bn Dau ” = ” xảy ra khi a1 b1 = a2 b2 1 = · · · = an 2 > 0. n 1 2 n 12. Bat đȁng thác BCS dạng c®ng mau (Caushy - Swarchz) Với ∀xi > 0, i = 1, 2, · · · , n, ta có a2 a2 a2 (a1 + a2 + · · · + an)2 1 + 2 + · · · + n ≥ x1 x2 xn x1 + x2 + · · · + xn 13. Bat đȁng thác Minkopsky Cho hai dãy so thực dương (a1; a2; · · · ; an); (b1; b2; · · · ; bn), ta có a2 + b2 + a2 + b2 + · · · + a2 + b2 ≥ (a1 + a2 + · · · + an)2 + (b1 + b2 + · · · + bn)2 Dau ” = ” xảy ra khi a1 b1 = a2 b2 = · · · = an 2.4.2. N i dung phương pháp Phương pháp sả dụng bat đȁng thác đe giải h phương trình là m®t phương pháp đánh giá mới, thực hi n qua các bước sau: Bước 1: Tà các phương trình đã cho trong h , ta sả dụng các phép bien đői tương đương đe làm xuat hi n các bat đȁng thác đã được cháng minh. Bước 2: Đoi với moi bat đȁng thác, đe cho dau đȁng thác xảy ra tương áng với dau đȁng thác trong h phương trình, ta tìm được moi liên h của các ȁn. Bieu dien các ȁn trong h thông qua m®t ȁn. 1
  • 42. 39 Viết đề tài giá sinh viên – ZALO:0973.287.149-TEAMLUANVAN.COM x1 + x2 + · · · + x2000 = a 2 2000 Bước 3: Thay the các ȁn vàa được bieu dien vào m®t phương trình trong h , giải phương trình m®t ȁn roi tìm các ȁn còn lại qua ȁn vàa tìm được. 2.4.3. Bài t p sfi dnng tính chat bat đang thfíc giải h phương trình phi tuyen Bài toán 2.24 (xem [2]). Giải h phương trình: x6 + y8 + z10 = 1 (1) x2013 + y2015 + z2017 = 1 (2) Bài giai. Tà (1), ta suy ra −1 ≤ x, y, z ≤ 1. Tà đó, ta có: x6 − x2013 = x6(1 − x2007) ≥ 0 ⇔ x6 ≥ x2013. Dau ” = ” xảy ra ⇔ x = 0 ho c | x |= 1. y8 − y2015 = y8(1 − y2007) ≥ 0 ⇔ y8 ≥ y2015 Dau ” = ” xảy ra ⇔ y = 0 ho c | y |= 1. z10 − z2017 = z10(1 − z2007) ≥ 0 ⇔ z10 ≥ z2015. Dau ” = ” xảy ra ⇔ z = 0 ho c | z |= 1. C®ng tàng ve của các bat đȁng thác trên, ta suy ra 1 = x6 + y8 + z10 ≥ x2013 + y2015 + z2017 = 1 Do đó dau đȁng thác xảy ra, tác là: x6(1 − x2007) = 0 y8(1 − y2007) = 0 z10(1 − z2007) = 0 Ket hợp với (1) và (2), ta suy ra (x; y; z) = (1; 0; 0), (0; 1; 0), (0; 0; 1). Bài toán 2.25 (xem [5]). Giải h phương trình: x2 + x2 + · · · + x2 = a2 1 2 . . . 2000 x2000 + x2000 + · · · + x2000 = a2000 1
  • 43. 40 Viết đề tài giá sinh viên – ZALO:0973.287.149-TEAMLUANVAN.COM √ √ − − √ a a a a a a a a a a a a √ Bài giai. ∗ Neu a = 0, tà phương trình thá hai, ta suy ra x1 = x2 = . . . = x2000 = 0. ∗ Neu a /= 0, tà phương trình thá hai, ta suy ra ( x1 )2 + ( x2 )2 + ( x3 )2 + · · · + ( x2000 )2 = 1 a x1 a x2 a x2000 a ⇒| a |≤ 1; | a |≤ 1; . . . ; | a |≤ 1 ⇒ ( x1 )3 ≤ ( x1 )2; ( x2 )3 ≤ ( x2 )2; . . . ; ( x2000 )3 ≤ ( x2000 )2. C®ng tàng ve của các bat phương trình trên, ta có: 1 = ( x1 )3 + ( x2 )3 + · · · + ( x2000 )3 ≤ ( x1 )2 + ( x2 )2 + · · · + ( x2000 )2 = 1. Dau đȁng thác xảy ra khi ( x1 )2 = ( x1 )3, . . . , ( x2000 )2 = ( x2000 )3. a a a a H có các nghi m: (x1; x2; . . . ; x2000) = (a; 0; 0; . . . ; 0), (0; a; 0; . . . ; 0), . . . , (0; 0; 0; . . . ; a). Bài toán 2.26 (xem [5]). Giải h phương trình √ 1 1 + 2x2 1 + 1 + 2y2 2 = 1 + 2xy x(1 2x) + y(1 2y) = 2 9 (Đe thi HSG Quoc Gia năm 2009) Bài giai. 1 Đieu ki n x, y ∈ [0; 2 ]. Ta cháng minh bat đȁng thác 1 1 2 √ 1 + 2x2 + √ 1 + 2y2 ≤ √ 1 + 2xy ⇔ 1 + 2x2 + 1 + 2y2 + √ 1 + 2x2 √ 1 + 2y2 ≤ 1 + 2xy Ta có x, y ∈ [0; 2 ] nên √ 1 + 2x2 √ 1 + 2y2 ≤ 1 + 2xy ⇔ √ 1 + 2x2 1 + 2y2 ≥ 1 + 2xy. ⇔ (1 + 2x2)(1 + 2y2) ≥ (1 + 2xy)2 ⇔ (x − y)2 ≥ 0 (Luôn đúng) 1 1 2 Và 1 + 2x2 + 1 + 2y2 ≤ 1 + 2xy (1 − 4xy)(x − y)2 ⇔ (1 + 2xy)(1 + 2x2)(1 + 2y2) ≥ 0 : Đúng Do dau đȁng thác xảy ra nên x = y, ta có phương trình 9 √ x(1 − 2x) = 1 ⇔ 162x2 − 81x + 1 = 0 1 1 2 4 1 2 2 √
  • 44. 41 Viết đề tài giá sinh viên – ZALO:0973.287.149-TEAMLUANVAN.COM ⇔ 1 1 1 x y z x + y2 + z3 = 14 (1) x = 81 ± √ 5913 324 V y h phương√trình đã cho√có hai nghi m√ √ 81 + (x; y) = ( 5913 81 + ; 5913 ), ( 81 − 5913 ; 81 − 5913 ). 324 324 324 324 Bài toán 2.27 (xem [5]). Giải h phương trình x + y + z = 1 x4 + y4 + z4 = xyz . Bài giai. Ta có x4 + y4 ≥ 2x2y2 z4 + y4 ≥ 2z2y2 x4 + z4 ≥ 2x2z2 ⇒ x4 + y4 + z4 ≥ x2y2 + y2z2 + x2z2 M t khác x2y2 + x2z2 ≥ 2x2yz x2y2 + y2z2 ≥ 2y2xz z2y2 + x2z2 ≥ 2z2yx ⇔ x2y2 + x2z2 + y2z2 ≥ xyz(x + y + z) = xyz Suy ra x4 + y4 + z4 ≥ x2y2 + y2z2 + x2z2 ≥ xyz Dau đȁng thác xảy ra x = y = z và x + y + z = 1 1 Suy ra x = y = z = 3 1 1 1 V y h phương trình có nghi m (x; y; z) = ( ; ; ). 3 3 3 Bài toán 2.28 (xem [5]). Giải h phương trình ( 2x + 3y + 6z )( 2 + 3 + 6 ) = 1 (2) . (x, y, z là các so dương) Bài giai. 3 2 1 Phương trình (2) ⇔ ( x + y + z )(3x + 2y + z) = 36 x y x z z y ⇔ 6( y + x ) + 3( z + x ) + 2( y + z ) = 22 Vì x, y, z > 0 ⇔
  • 45. 42 Viết đề tài giá sinh viên – ZALO:0973.287.149-TEAMLUANVAN.COM 4 4 4 x + y = 2a 1 + 4 ≤ 3 x y x y x z z y ⇒ 6( y + x ) ≥ 12; 3( z + x ) ≥ 6; 2( y + z ) ≥ 4 3 2 1 ⇒ ( x + y + z )(3x + 2y + z) = 36 x y x z z y ⇔ 6( y + x ) + 3( z + x ) + 2( y + z ) ≥ 22 Dau đȁng thác xảy ra ⇔ x = y = z Thay ⇔ x = y = z vào (1), ta có x3 + x2 + x − 14 = 0 ⇔ (x − 2)(x2 + 3x + 7) = 0 ⇔ x − 2 = 0 ( Vì x2 + 3x + 7 = (x + 3 )2 + 19 ≥ 19 > 0) ⇔ x = 2. 2 4 4 V y h có nghi m duy nhat x = y = z = 2. Bài toán 2.29 (xem [5]). Tìm tat cả các so dương x, y thỏa mãn x + y = 3 (Thi vào 10 Năng khieu Toán - Tin, ĐHQG TP.HCM 1997 - 1998 Vòng 2) Bài giai. 1 4 Tà giả thiet suy ra ( x + y )(x + y) ≤ 9 Áp dụng bat đȁng thác Schwartz, ta có 1 4 √ 2 √ 2 √ 2 √ 2 1 √ 2 √ 2 ( + )(x + y) = [( x y x) + ( y) ][( x) + ( y) ] ≥ (√ x x + √ y y) = 9 1 4 ⇒ 9 ≤ ( x + y )(x + y) ≤ 9 x y Dau đȁng thác xảy ra ⇔ 1 = 2 và x + y = 3 ⇔ x = 1; y = 2 V y h đã cho có nghi m (x; y) = (1; 2). Bài toán 2.30 (xem [5]). Giải h phương trình với tham so a ≥ 0 x + y + xy = a2 + 2a (Thi chon đ®i tuyen toán 9 Thanh Hóa 1995 - 1996) Bài giai. Ta có x4 + y4 ≥ 2x2y2 ⇔ 2a4 ≥ 2x2y2 ⇔ (xy)2 ≤ a4 ⇔ xy ≤ a2
  • 46. 43 Viết đề tài giá sinh viên – ZALO:0973.287.149-TEAMLUANVAN.COM 1 1 x1 + x2 + · · · + xn = 9 (1) ⇒ n2 ≤ 9 ⇒ n ≤ 3 Màn ∈ N∗ ⇒ n = 1, 2, 3 ∗ Neu n = 2, ta có h x1 + x2 = 9 ⇔ x1 + x2 = 9 1 2 2 2 2 2 2 (x + y)2 ≤ 2(x2 + y2) mà (x2 + y2)2 ≤ 2(x4 + y4) = 4a4 ⇒ (x + y)2 ≤ 4a2 ⇒ x + y ≤ 2a Suy ra x + y + xy ≤ a2 + 2a Dau đȁng thác xảy ra ⇔ x = y = a V y h có nghi m duy nhat (x; y) = (a; a). Bài toán 2.31 (xem [5]). Tìm tat cả các so dương x1, x2, . . . , xn thỏa mãn h phương trình (n là so nguyên dương) Bài giai. x1 1 1 + x + · · · + x = 1 (2) 1 Tà (1) và (2) suy ra (x1 + x2 + · · · + xn)( x 1 1 + x + · · · + x ) = 9 M t khác n2 = (1 + 1 + . . . + 1)2 = ( √ x 1 1 1 √ x1 1 + √ x 1 2 √ x2 1 + . . . + √ x 1 n √ xn ≤ ((x1 + x2 + · · · + xn)( x + x + · · · + x ) = 9 ∗ Neu n = 3, ta có h x1 + x2 + x3 = 9 ⇒ (x + x 1 + x )( 1 1 + + ) = 9 1 1 1 + + = 1 x1 x2 x3 1 2 3 x1 x2 x3 1 Mà (x1 + x2 + x3)( 1 1 1 + + ) x2 x3 = [(x )2 + ( √ x )2 + ( √ x )2][( 1 )2 + ( 1 )2 + ( 1 )2] ≥ (1 + 1 + 1)2 = 9 1 2 3 √ x1 √ x2 √ x3 D √ au đȁng √thác xả √ y ra khi và chỉ khi x1 1 √ x1 x1 = 1 √ x1 x1 = 1 √ x1 ⇔ x1 = x2 = x3 Mà x1 + x2 + x3 = 9 nên x1 = x2 = x3 = 3. 1 1 + = 1 x1 x2 x1x2 = 9 Suy ra x1, x2 là các √nghi m c √ ủa phương√ trình t2 − √ 9t + 9 = 0 ⇔ (x ; x ) = ( 9 − 45 ; 9 + 45 ), ( 9 + 45 ; 9 − 45 ) ∗ Neu n = 1, ta có h x1 = 9 Vô nghi m = 1 x1 V y h đã cho có nghi m x1 = x2 = x3 = 3 n 1 n x 2 2 )2 1 n
  • 47. 44 Viết đề tài giá sinh viên – ZALO:0973.287.149-TEAMLUANVAN.COM 2x2000 = x1 + x 2 2 1 Ho c (x ; x ) = ( 9 − √ 45 ; 9 + √ 45 ), ( 9 + √ 45 ; 9 − √ 45 ). 1 2 2 2 2 2 Bài toán 2.32 (xem [5]). Giải h phương trình 2000 ȁn so: 2x1 1 = x2 + 2 1 (1) 2x2 = x3 + x3 (2) . . . 1 2x1999 = x2000 + (1999) 1 x2000 Bài giai. Đieu ki n : xi 0, i = 1, . . . , 2000 Tà (1) ⇒ x1, x2 cùng dau. Tà (2) ⇒ x2, x3 cùng dau. . . . Tà (1999) ⇒ x1999, x2000 cùng dau. Tà (2000) ⇒ x1, x2000 cùng dau. Như v y các ȁn so xi cùng dau. M t khác, neu (x1; x2; . . . ; x2000) là m®t nghi m thì (−x1; −x2; . . . ; −x2000) cũng là nghi m, do đó ta chỉ can xét x1; x2; . . . ; x2000 > 0. 1 Khi đó: 2x1 = x2 + 2 1 1 ≥ 2 ⇒ x1 ≥ 1 ⇒ x 1 ≤ 1. 2x2 = x3 + 3 ≥ 2 ⇒ x2 ≥ 1 ⇒ x 1 ≤ 1. Tương tự ta có 1 x2000 1 ⇒ x1 1 + x + · · · + x 1 2000 ≤ x1 + x2 + . . . + x2000. M t khác neu c®ng tàng ve 2000 phương trình của h , ta có 1 x1 + x2 + . . . + x2000 = 1 1 + x + · · · + x 1 2000 Dau đȁng thác xảy ra khi và chỉ khi x1 = x2 = . . . = x2000 = 1. Tóm lại h có hai nghi m (x1; x2; . . . ; x2000) = (1; 1; . . . ; 1), (−1; −1; . . . ; −1) x 1 x (2000) x x 2 ≤
  • 48. 45 Viết đề tài giá sinh viên – ZALO:0973.287.149-TEAMLUANVAN.COM x + 1 2 ⇒ ≥ 2 = y y2 Bài toán 2.33 (xem [5]). Giải h phương trình 2x2 y2 + 1 = z 4z4 Bài giai. Xét hai trường hợp sau: ∗ Trường hợp 1 z6 + z4 + z2 + 1 = x Trong ba so x, y, z có ít nhat m®t so bang 0 Giả sả x = 0 ⇒ y = 0 ⇒ z = 0. V y (x; y; z) = (0; 0; 0) là m®t nghi m. ∗ Trường hợp 2 Không có so nào bang 0 2x2 Vì y = x2 + 1 > 0 ⇒ y > 0 ⇒ z > 0 ⇒ x > 0 V y x, y, z > 0 2x 2x2 Dě thay ≤ 1 ⇒ ≤ x ⇒ y ≤ x x2 + 1 x2 + 1 Ta lại có y4 − 2y2 + 1 ≥ 0 ⇒ y4 + y2 + 1 ≥ 3y2 3y2 3y3 ⇒ y4 + y2 + 1 ≤ 1 ⇒ y4 + y2 + 1 ≤ y ⇒ z ≤ y Ta có z6 + z4 + z2 + 1 − 4z3 = (z6 − 2z3 + 1) + z2(z2 − 2z + 1) = (z3 − 1)2 + z2(z − 1)2 ≥ 0 z6 + z4 + z2 + 1 4z3 4z3 ⇒ z6 + z4 + z2 + 1 ≤ 1 4z4 ⇒ z6 + z4 + z2 + 1 ≤ z hay x ≤ z Suy ra x ≤ z ≤ y ≤ x ⇒ x = y = z Thay vào phương trình (1) ta có x(x − 1)2 = 0 ⇒ x = y = z = 0( Không thỏa mãn đk) ho c x = y = z = 1 (Thỏa mãn) The x = y = z = 1 vào h phương trình thay nghi m đúng h phương trình. V y h đã cho có nghi m (x; y; z) = (0; 0; 0), (1; 1; 1). Bài toán 2.34 (xem [5]). Cho x, y, z là các so thực thỏa mãn đieu ki n x + y + z = 5 x2 + y2 + z2 = 9
  • 49. 46 Viết đề tài giá sinh viên – ZALO:0973.287.149-TEAMLUANVAN.COM 3 3 2 x + 2xy + 1 2 y = 0 ⇒ 7 Cháng minh rang 1 ≤ x, y, z ≤ 3 (Thi vào 10 chuyên Toán - Tin ĐHTH TP. HCM 23/ 06/ 1996) Bài giai. Tà x + y + z = 5 ⇒ y + z = 5 − x ⇒ (y + z)2 = (5 − x)2 Ta có 2(y2 + z2) ≥ (y + z)2 ⇒ 2(9 − x2) ≥ (5 − x)2 ⇒ 3x2 − 10x + 7 ≤ 0 ⇔ (x − 1)(3x − 7) ≤ 0 ⇔ 1 ≤ x ≤ 7 . 7 Vì x, y, z có vai trò như nhau, cháng minh tương tự, ta có 1 ≤ x, y, z ≤ 3 . 2.5 Phoi h p nhieu phương pháp giải h phương trình 2.5.1. N i dung phương pháp Đoi với nhieu h phương trình phác tạp mà chỉ dùng m®t trong bon phương pháp nêu trên không đưa ra được ket quả, ta can phoi hợp nhieu phương pháp lại với nhau đe đưa ra lời giải cho bài toán đó. 2.5.2. Bài t p áp dnng nhieu phương pháp giải h phương trình Bài toán 2.35 (xem [2]). Giải h phương trình: x2 + 8y2 = 12 (Thi lớp 10 chuyên toán ĐHTH Hà N®i 2008 - 2009) Bài giai. Nh n thay neu h có nghi m (x; y) thì y /= 0 vì neu y = 0 thì x2 = 12 x3 = 0 (Vô lý). Khi y = / 0, thay 12 = x2 + 8y2 vào phương trình thá hai, ta có: x3 + 2xy2 + (x2 + 8y2)y = 0 ⇔ x3 + 2xy2 + x2y + 8y3 x 3 ⇔ ( y ) x x + ( )2 + 2 y y + 8 = 0. Đ t x = t t3 + t2 + 2t + 8 = 0 y ⇔ (t + 2)(t2 − t − 4) = 0
  • 50. 47 Viết đề tài giá sinh viên – ZALO:0973.287.149-TEAMLUANVAN.COM 5 2 4 16 2 r r 16 4 ⇔ u ⇔ (t + 2)[(t − 1 )2 + 15 ] = 0 ⇔ t = −2 ho c (t − 1 )2 + 15 = 0 (Vô lý). x 2 4 Với t = −2 ⇒ y = −2 ⇒ x = −2y. Thay vào phương trình thá nhat, ta có 4y2 + 8y2 = 12 ⇔ y2 = 1 ⇔ y = ±1. Khi y = 1 ⇒ x = −2 Khi y = −1 ⇒ x = 2 H phương trình (2.9) có hai nghi m (x; y) = (2; −1), (−2; 1). Bài toán 2.36 (xem [2]). Giải h phương trình: x2 + y + x3y + xy2 + xy = 5 4 5 x4 + y2 + xy(1 + 2x) = 4 (Đe TS Đại hoc, Cao đȁng 2008 - Khoi A) Bài giai. H đã cho tương đương với x2 + y + xy + xy(x2 + y) = 5 4 (∗) Đ t u = x2 + y (x2 + y)2 + xy = 4 . H (∗) trở thành v = xy u + v + uv = −5 v = −5 − u2 4 u2 + v = −5 4 4 u3 + u2 + = 0 4 ⇔ u = 0; v = −5 ho c u = −1 , v = −3 . ٨ Với u = 0; v = −5 , ta có h phương trình: 4 x2 + y = 0 xy = −5 4 x = 3 −5 ⇔ 25 y = − 3 ٨ Với u = −1 , v = −3 , ta có h phương trình: 3 2 1 2 x2 − + = 0 2x3 + x − 3 = 0 x = 1 2x 2 y = −3 2x y = −3 2x y = −3 2 V y h phương trình đã cho có hai nghi m : (x; y) = ( r 3 −5 ; − r 3 25 ), (1; −3 ). ⇔ ⇔ 4 2 2 4
  • 51. 48 Viết đề tài giá sinh viên – ZALO:0973.287.149-TEAMLUANVAN.COM / 1 1 7 xyzt = 648 1 + 1 = 5 Suy ra z t 18 126 1 1 1 1 x + y + z = 3 1 1 7 7 ( Vô nghi m) Bài toán 2.37 (xem [5]). Giải h phương trình x + y + z + t = 22 + = x y 12 Bài giai. Đieu khi n x, y, z, t = 0 7 z t 18 5 Ta có x + y = xy; z + t = zt 12 18 7 5 7 5 ⇒ (x + y)(z + t) = 12 xy. 18 zt = Và (x + y) + (z + t) = 22 . 12 18 .648 = 105 Suy ra (x + y), (z + t) là hai nghi m của phương trình X2 − 22X + 105 = 0 ⇔ X1 = 15, X2 = 7 Xét hai trường hợp: ∗ Trường hợp 1 x + y = 7 z + t = 15 x + y = 7 xy = 12 (x; y) = (3; 4), (4; 3) + = x y 12 1 + 1 = 5 z + t = 15 zt = 54 (z; t) = (6; 9), (9; 6) ∗ Trường hợp 2 x + y = 15 x + y = 15 z + t = 7 xy = 180 + = x y 12 1 + 1 = 5 z + t = 7 zt = 5 V y h đã cho có bon nghi m (x; y; z; t) = (3; 4; 6; 9), (3; 4; 9; 6), (4; 3; 6; 9), (4; 3; 9; 6). Bài toán 2.38 (xem [5]). a, Cho x, y, z là các so thực thỏa mãn x + y + z = 3 7 1 1 ⇔ ⇔ z t 18
  • 52. 49 Cháng minh rang có ít nhat 1 trong 3 so x, y, z = 3 Viết đề tài giá sinh viên – ZALO:0973.287.149-TEAMLUANVAN.COM x + y + z = 3 ∗Trường hợp 1 y = −z y + 2z 1 1 ∗Trường hợp 3 x = −y y + 2.3 = 1 y + 2z2 = 1 b, Áp dụng câu a, giải h phương trình 1 1 1 1 + + = x y z 3 Bài giai. a, Tà giả thiet, suy ra 1 1 1 + + x y z 1 = x + y + y 1 1 1 ⇔ ( x + y )( z 1 — x + y + z ) ⇔ (x + y)[z(x + y + z) + xy] = 0 ⇔ (x + y)(y + z)(x + x) = 0 ⇔ x + y = 0 ⇒ z = 3 ho c y + z = 0 ⇒ x = 3 ho c x + z = 0 ⇒ y = 3 b, Tà hai phương trình đau suy ra x = 3 ho c y = 3 ho c z = 3, the vào h phương trình, ta có x = 3 y + z = 0 ⇔ x = 3 ⇔ x = 3 + = 0 y z 2 2y 2 − y − 1 = 0 1 y = 1 ho c y = 2 ⇒ (x; y; z) = (3; 1; −1), (3; −1 ; 1 ) ∗ Trường hợp 2 y = 3 x + z = 0 y = 3 ⇔ x = −z + = 0 x z 3 + 2z2 = 1 + = 0 x z 2z2 = −2 ( Vô lí) z = 3 x + y = 0 ⇔ z = 3 ⇔ z = 3 + = 0 x y 2 ⇒ (x; y; z) = (17; −17; 3) 1 1 Suy ra h vô nghi m. 1 1 1 1 2 y + 2z2 = 1 y = −z = 0 x = −y y = −17 2
  • 53. 50 Viết đề tài giá sinh viên – ZALO:0973.287.149-TEAMLUANVAN.COM − V y h đã cho có 3 nghi m (x; y; z) = (3; 1; 1), (3; −1 ; 2 1 2 ), (17; −17; 3). Bài toán 2.39 (Xem [5]). Giải h phương trình ux3 + vy3 = 14 (1) ux2 + vy2 = 5 (2) ux + vy = 2 (3) u + v = 1 (4) (Thi HSG TP. HCM Ngày 17/ 12/ 1994 Vòng 2) Bài giai. Tà (4) suy ra v = 1 − u, the vào (1) xu3 + (1 − u)y3 = 14 ⇒ u(x3 − y3) = 14 − y3 ⇒ u(x − y)(x2 + xy + y2) = 14 − y3(5) Tương tự Neu the v = 1 − u vào (2), ta có u(x − y)(x + y) = 5 − y2(6) The v = 1 − u vào (3), ta có u(x − y) = 2 − y(7) Tà (5) và (7) suy ra (2 − y)(x2 + xy + y2) = 14 − y3 Tà (6) và (7) suy ra (2 − y)(x + y) = 5 − y2 Khai trien phương trình cuoi cùng, ta có 2(x + y)2 − xy(x + y) − 2xy − 14 = 0 2(x + y) − xy − 5 = 0 Đ t x + y = a, xy = b 2a2 − ab − 2b − 14 = 0 2a − b = 5 ⇔ a = 4 b = 3 ⇔ x + y = 4 xy = 3 ⇒ (x; y) = (3; 1), (1; 3) The x và y ta tìm được các nghi m của h 1 (x; y; u; v) = (3; 1; ; 2 1 1 1 ), (1; 3; ; ) 2 2 2 Bài toán 2.40 (xem [5]). Giải h phương trình x4 + x2y2 + y4 = 482 x2 + xy + y2 = 37 ⇒
  • 54. 51 Viết đề tài giá sinh viên – ZALO:0973.287.149-TEAMLUANVAN.COM ∗ xy = 12 Bài giai. Đ t S = x + y; P = xy Ta có h phương trình vào (1) S4 − 4PS2 + 3P 2 = 481 (1) S2 − P = 37 (2) Tà (2) suy ra S2 = P + 37 the Suy ra (P + 37)2 − 4P(P + 37) + 3P 2 = 481 ⇔ P 2 + 74P + 1369 − (4P 2+ ⇔ 148P) + 3P 2 = 481 ⇔ −74P = −888 ⇔ P = 12 ⇔ S = ±7 Xét hai trường hợp Trường hợp 1 x + y = 7 xy = 12 Nên x, y là nghi m của phương trình X2 − 7X + 12 = 0 ⇔ X1 = 3 ho c X2 = 4 ∗ Trường hợp 2 x + y = −7 Nên x, y là nghi m của phương trình X2 + 7X + 12 = 0 ⇔ X1 = −3 ho c X2 = −4 V y h phương trình có 4 nghi m (x; y) = (3; 4), (4; 3), (−3; −4), (−4, −3). Bài toán 2.41 (xem [5]). Giải h phương trình x − y2 − yz − z = 0 (1) x − y − y2 − z2 = 0 (2) x + y − y3 − z = 0 (3) (Thi HSG Qu n I 1995 - 1996 Vòng 1) Bài giai. Lay (2) − (1), ta được : yz − y + z − z2 = 0 ⇔ (z − 1)(y − z) = 0 Do đó h đã cho tương đương với : (z − 1)(y − z) = 0 x − y − y2 − z2 = 0 ⇔ z − 1 = 0 x − y − y2 − z2 = 0 (4) ho c y − z = 0 x − y − y2 − z2 = 0 (5) x + y − y3 − z = 0 x + y − y3 − z = 0 x + y − y3 − z = 0
  • 55. 52 Viết đề tài giá sinh viên – ZALO:0973.287.149-TEAMLUANVAN.COM − − z = 1 y = 0; y = −1; y = 2 1 2 3 Giải h (4): z − 1 = 0 x − y − y2 − z2 = 0 ⇔ x + y y3 z = 0 z = 1 z = 1 x − y − y2 − 1 = 0 x + y − y3 − 1 = 0 ⇔ y3 − y2 − 2y = 0 x = y3 − y + 1 ⇔ y(y2 − y − 2) = 0 x = y3 − y + 1 z = 1 ⇔ y1 = 0; y2 = −1; y3 = 2 x = y3 − y + 1 z = 1 x1 = 1; x2 = 1; x3 = 7 Giải h (5): z = y x − y − 2y2 = 0 ⇔ x − y3 = 0 z = y y3 − 2y2 − y = 0 x = y3 z = y ⇔ y(y2 − 2y − 1) = 0 x = y3 z = y ⇔ y1 = 0; y2 = 1 − √ 2; 1 + √ 2 x = y3 ⇔ (x; y; z) = (0; 0; 0), ((1 + √ 2)3; 1 + √ 2; 1 + √ 2), ((1 − √ 2)3; 1 − √ 2; 1 − √ 2) V y h phương trình có sáu nghi m (x; y; z) = (1; 0; 1), (1; −1; 1), (7; 2; 1), (0; 0; 0), ((1 + √ 2)3; 1 + √ 2; 1 + √ 2), ((1 − √ 2)3; 1 − √ 2; 1 − √ 2) Bài toán 2.42 (xem [5]). Giải h phương trình x + y + z = 6 (1) x2 + y2 + z2 = 18 (2) √ x + √ y + √ z = 4 (3) (Thi HSG Toàn Quoc 1996 - 1997 Bảng A) Bài giai. Tà (1) ⇒ 36 = (x + y + z)2 ⇔ 36 = x2 + y2 + z2 + 2(xy + xz + yz) ⇔ 36 = 18 + 2(xy + xz + yz) ⇔ xy + xz + yz = 9 Tà (3) ⇒ 16 = ( √ x + √ y + √ z)2 ⇔ x + y + z + 2( √ xy + √ xz + √ yz) = 16 ⇔
  • 56. 53 Viết đề tài giá sinh viên – ZALO:0973.287.149-TEAMLUANVAN.COM ⇔ ⇔ √ xy + √ xz + √ yz = 5 ⇔ ( √ xy + √ xz + √ yz)2 = 25 ⇔ xy + yz + xz + 2( √ xy2z + √ xyz2 + √ x2yz) = 25 ⇔ √ xyz( √ x + √ y + √ z) = 8 ⇔ xyz = 4 V y h phương trình đã cho tương đương với: 4 Tà (5) ⇒ yz = x (De thay x, y, z > 0). (4) ⇔ xy + yz + xz + x2 = 9 + x2 ⇔ x(x + y + z) + yz = 9 + x2 6x + 4 = 9 + x2 x ⇔ x3 − 6x2 + 9x − 4 = 0 ⇔ (x − 1)2(x − 4) = 0 ⇔ x = 1 ho c x = 4 The vào, ta suy ra h có các nghi m (x; y; z) = (1; 1; 4), (1; 4; 1), (4; 1; 1). x + y + z = 6 (1) xy + yz + xz = 9 (4) xyz = 4 (5) Bài toán 2.43 (xem [5]). Giải h phương trình x2 + y2 + xy = 37 (1) x2 + z2 + xz = 28 (2) y2 + z2 + yz = 19 (3) (Thi HSG Toàn Quoc 1994 - 1995) Bài giai. Lay phương trình (1) − (2), ta được : y2 − z2 + xy − xz = 9 ⇔ (y − z)(x + y + z) = 9 Lay phương trình (2) − (3), ta được : x2 − y2 + xz − yz = 9 ⇔ (x − y)(x + y + z) = 9 Suy ra : (y − z)(x + y + z) = (x − y)(x + y + z). Mà x + y + z /= 0 ⇒ y − z = x − y x + z = 2y x + y + z = 3y 3 Do đó : (x − y)(x + y + z) = 9 ⇔ (x − y)3y = 9 ⇔ (x − y)y = 3 ⇔ x = y + y . Thay vào phương trình (1), ta có: ⇔
  • 57. 54 Viết đề tài giá sinh viên – ZALO:0973.287.149-TEAMLUANVAN.COM ⇒ y y 3 3 3 3 3 3 (y + 1 )2 + y2 + (y + 1 )y = 37 ⇔ 3y4 − 28y2 + 9 = 0 y2 = 9 ho c y2 = 1 3 ⇔ y1 = 3; y2 = −3; y3 = √ 3 3 ; y4 = − √ 3 3 - Với y1 = 3 ⇒ x1 = 4, z1 = 2 - Với y2 = − √3 ⇒ x2 = −4,√ z2 = −2 √ - Với y3 3 = 3√ ⇒ x3 10 3 = ,z3 3 = −8 3 3 - Với y4 = − 3 x 3 = −10 √ 3 , z 3 8 √ 3 = 3 V y h phương trình đã cho có bon √ nghi√ m √ √ √ √ (x; y; z) = (4; 2; 3), (−4; −3; −2), ( 10 3 ; 3 , −8 3 ), ( −10 3 ; − 3 ; 8 3 ) ⇔ 4 3
  • 58. 55 Viết đề tài giá sinh viên – ZALO:0973.287.149-TEAMLUANVAN.COM Chương 3 M t so fíng dnng của h phương trình phi tuyen 3.1 Ứng dnng của h phương trình đa thfíc trong giải các bài toán cfic trị và chfíng minh bat đang thfíc. Bài toán 3.1 (xem [5]). Cho phương trình x3 − 2002x2 + 2001bx − 2000a = 0 Tìm giá trị lớn nhat của a sao cho ∃b đe phương trình trên có ba nghi m trên [−2002; 2002]. Bài giai. Giả sả x1, x2, x3 là ba nghi m của phương trình. Khi đó theo định lí Viete cho phương trình b c ba, ta có: x1 + x2 + x3 = 2002 x1x2 + x2x3 + x1x3 = 2001b x1x2x3 = 2000a Xảy ra các trường hợp sau : + Neu x2 < 0 ho c x3 < 0, khi đó x1 + x2 + x3 < x3 ≤ 2002 (Mâu thuan (3.1)) + Neu x1 < 0 ≤ x2 ≤ x3, theo bat đȁng thác Caushy, ta có √ √ 20023 2002 = x1 + x2 + x3 ≥ 3 3 x1x2x3 = 3 3 2000a ⇒ a ≤ 2002 54000 20023 Dau ” = ” xảy ra khi x1 = x2 = x3 = Khi đó phương trình đã cho trở thành và khi ay b = 3 . 6003
  • 59. 56 Viết đề tài giá sinh viên – ZALO:0973.287.149-TEAMLUANVAN.COM 2 2 2 x + 3y + 4z =101 (2) − ⇔ x2 − y2 = 21 (1) x3 − 2002x2 + 20022 2 x − 20022 33 = 0 ⇔ (x − 2002 )3 = 0 3 Phương trình trên có ba nghi m x1 = x2 = x3 = 20023 V y amax = 54000 là giá trị lớn nhat can tìm. 2002 3 ∈ [−2002; 2002]. Bài toán 3.2 (xem [6]). Cho bieu thác M = x2 + y2 + 2z2 + t2 Hãy tìm giá trị nhỏ nhat của M và các giá trị nguyên không âm tương áng của x, y, z, t biet chúng thỏa mãn đong thời: x2 − y2 + t2 = 21 (1) (Thi HSG Quoc Gia 1985 - 1986 Bảng A) Bài giai. C®ng (1) và (2), ta có: 2x2 + 2y2 + 4z2 + t2 = 122 ⇔ 2(x2 + y2 + 2z2 + t2) − t2 = 122 ⇒ 2M = 122 + t2 ≥ 122 ⇒ M ≥ 61 ⇒ min M = 61 Khi t = 0 ⇒ x2 + 3y2 + 4z2 = 101 (2) Vì x, y nguyên, không âm nên (x y)(x + y) = 21 x − y = 1 x + y = 21 ⇔ (x; y) = (11; 0), (5; 2). ho c x − y = 3 x + y = 7 ∗ Trường hợp 1: Thay x = 11, y = 0 vào (2), ta có 112 + 0 + 4z2 = 101 ⇒ 4z2 = −20( Vô lí) ∗ Trường hợp 2: Thay x = 5, y = 2 vào (2), ta có 52 + 3.22 + 4z2 = 101 ⇒ 4z2 = 64 ⇒ z = ±4 V y giá trị nhỏ nhat của M là 61, khi (x, y, z, t) = (5; 2; 4; 0), (5; 2; −4; 0). Bài toán 3.3 (xem [6]). Cho x2 + y2 + z2 ≤ 27. Tìm giá trị lớn nhat và nhỏ nhat của bieu thác: P = x + y + z + xy + yz + xz Bài giai. Ta có (x + y + z)2 = x2 + y2 + z2 + 2(xy + yz + xz) ≤ 3(x2 + y2 + z2) ≤ 81
  • 60. 57 Viết đề tài giá sinh viên – ZALO:0973.287.149-TEAMLUANVAN.COM − 2 2 2 4 ≤ r 4 4 1 4 4 4 ⇒ x + y + z ≤ 9 Mà xy + yz + xz ≤ x2 + y2 + z2 ≤ 27 ⇒ x + y + z + xy + yz + xz ≤ 36 V y Max(P ) = 36 khi x = y = z = 3 Đ t A = x + y + z; B = x2 + y2 + z2 ⇒ P = A + A2 B = 2 B + 1 (A + 1)2 2 − B + 1 2 ≥ − B + 1 . 2 Vì B ≤ 27 ⇒ − 2 ≥ −14 ⇒ P ≥ −14. Min(P) = 14 Khi x + y + z = −1 x + y + z = 27 Hay x = − √ 13, y = √ 13, z = −1. Bài toán 3.4 (xem [6]). Cháng minh rang neu 0 ≤ y ≤ x ≤ 1 thì x √ y − y √ x ≤ 1 . Dau đȁng thác xảy ra khi nào? Bài giai. Ta có 0 ≤ x ≤ 1 ⇒ √ x ≥ x2 Giả sả x √ y − y √ x 1 ≤ 4 ⇔ x √ y 1 + y √ x(1). 4 Theo Bat đȁng thác Caushy ta vó : y √ x + 1 ≥ yx2 + 1 ≥ 2 yx2. 1 = x √ y ⇒ x √ y − y √ x ≤ 1 . Dau đȁng thác xảy ra khi 0 ≤ y ≤ x ≤ 1 √ x = x2 yx2 = 1 ⇔ x = 1 y = 4 3.2 M t vài fíng dnng thfic te trong khoa hoc và đ i song Tình huong 3.1. Bà Lam có hai tài khoản ngân hàng gải tiet ki m, tại ngân hàng A bà gải 150 tri u đong, tại ngân hàng B bà gải 250 tri u đong. Bạn của bà Lam hỏi mác lãi suat của moi ngân hàng, nhưng bà Lam không nhớ chính xác, chỉ biet là ngân hàng B có lãi suat cao hơn ngân hàng A là 0, 5% trong m®t năm, và sau hai năm so tien trong cả hai tài khoản của bà là 459, 783 tri u. Hỏi lãi suat của moi ngân hàng là bao nhiêu? Bài giai. Goi mác lãi suat của moi ngân hàng tính trong m®t năm là xA% và xB%,(xA, xB > 0)
  • 61. 58 Viết đề tài giá sinh viên – ZALO:0973.287.149-TEAMLUANVAN.COM x = 0, 5 + x B A ⇔ Vì tài khoản ban đau trong ngân hàng A là 150 tri u, nên sau hai năm, tőng so tien cả goc và lãi tại ngân hàng A là 150.(1 + xA%)2. Vì tài khoản ban đau trong ngân hàng B là 250 tri u, nên sau hai năm, tőng so tien cả goc và lãi tại ngân hàng A là 250.(1 + xB%)2. Theo bài ra, ta có h phương trình xB − xA = 0, 5 150.(1 + xA%)2 + 250.(1 + xB%)2 = 459, 783 150.(1 + xA%)2 + 250.[1 + (xA + 0, 5)%]2 = 459, 783 (1) (1) ⇔ 400.(xA%)2 + 802, 5.xA% − 57, 27675 = 0 ⇔ xA = 6, 9 ho c xA = −2, 075 (loại). Với xA = 6, 9% thì xB = 7, 4%. Như v y lãi suat trong m®t năm của ngân hàng A là 6, 9%, ngân hàng B là 7, 4%. Tình huong 3.2. Giám đoc m®t công ty X vàa khánh thành ngôi nhà của mình có di n tích 4800m2, và phải xây 275 mét tường rào bao quanh. Bây giờ ông muon trong cây xanh và hoa cho căn nhà thêm đep. Doc theo ngôi nhà ông định trong cây tùng, phía trước và sau nhà ông định trong cây vạn tue, sao cho các cây trong đảm bảo ky thu t. Biet cőng ra vào dài 5 mét và khu vườn có dạng hình chǎ nh t. Hãy tính xem can phải mua bao nhiêu cây moi loại? Bài giai. Tình huong đ t ra là phải tính chính xác so cây can mua moi loại đe tiet ki m chi phí thu mua và v n chuyen. Đe làm được đieu đó, can phải biet chieu dài và chieu r®ng của khu vườn. Goi chieu dài và chieu r®ng của khu vườn là x, y (m).x, y > 0 Vì di n tích của mảnh vườn là 4800m2 nên x.y = 4800 Vì tőng chieu dài tường rào là 275m và cőng dài 5m nên chu vi là: (x + y).2 = 280 ⇔ x + y = 140 Ta có h phương trình :
  • 62. 59 Viết đề tài giá sinh viên – ZALO:0973.287.149-TEAMLUANVAN.COM x.y = 4800 2 x + y = 140 Đây là m®t h phương trình phi tuyen đoi xáng loại 1 có nghi m (x; y) = (60; 80) Coi khoảng cách giǎa các cây đúng kĩ thu t là cách nhau 2m. 80 Với chieu dài 80m thì so cây tùng trong hai bên theo chieu doc tường rào là 2. 2 (cây) M t sau của khu vườn r®ng 60m thì trong được 30 cây vạn tue. = 80 M t trước trà đi 5 m cőng, còn 55 m, như v y trong được nhieu nhat 28 cây vạn tue. Tőng so cây vạn tue là 58, trong đó trà 4 góc trong cây tùng, v y can mua 54 cây vạn tue. V y so cây can mua là : 80 cây tùng và 54 cây vạn tue. Tình huong 3.3. Nói ve sự chuyen đ®ng của các hành tinh trong thiên hà, moi m®t hành tinh đeu có quy đạo chuyen đ®ng riêng của nó, quay xung quanh m t trời và quanh các hành tinh khác. Quy đạo chuyen đ®ng của Trái đat và sao chői Halley’s Comet là hai hình elip không giao nhau. Ta đã biet phương trình của elip là m®t phương trình phi tuyen, v y thì h phương trình phi tuyen tạo bởi phương trình chuyen đ®ng của trái đat và sao chői Halley’s Comet là m®t h vô nghi m. Tình huong 3.4. M®t bác nông dân dự tính đào m®t ao nuôi thả cá hình chǎ nh t, với m®t so von xây dựng bờ kè không đői. Hãy tính toán phương án xây dựng đ® dài các cạnh sao cho di n tích sả dụng là lớn nhat. Bài giai. Tình huong đ t ra là với cùng so von xây dựng bờ kè (coi chieu sâu của ao không đői), can xác định chieu dài và chieu r®ng của ao đe di n tích sả dụng lớn nhat. Giả sả với so von dự tính, xây được a(m) dài kè bao. Như v y a là chu vi ao cá và a không đői. Goi x, y là đ® dài các cạnh của hình chǎ nh t, ta can giải quyet bài toán: Cho h phương trình: x + y = a (1) x.y = m (2) Tìm moi liên h giǎa x và y đe m đạt giá trị lớn nhat? Tà phương trình (2) suy ra a a y = 2 − x, suy ra xy = x( 2 − x)
  • 63. 60 Viết đề tài giá sinh viên – ZALO:0973.287.149-TEAMLUANVAN.COM − 2 4x + 5xy = m (2) ≤ xy = x2 + a x 2 a 2 a2 a2 xy = −(x − 4 ) + 16 16 . a a Dau ” = ” xảy ra khi x = 4 ⇒ x = y = 4 V y đe có được di n tích sả dụng lớn nhat thì nên xây ao cá hình vuông. Tình huong 3.5. M®t bác nông dân can xây dựng m®t ho ga hình h®p chǎ nh t không nap, có the tích 3200cm3, tỉ so giǎa chieu cao của ho và chieu r®ng của đáy bang 2. Hãy xác định di n tích của đáy ho ga sao cho khi xây tiet ki m nguyên v t li u nhat? Bài giai. Goi x, y lan lượt là chieu r®ng và chieu dài của đáy ho ga.(x, y > 0) h Goi h là chieu cao của đáy ho ga (h > 0). Ta có Suy ra: = x, suy ra h = 2x. 2 - The tích ho ga là xyh = 3200 ⇔ 2x2y = 3200 ⇔ x2y = 1600. - Di n tích tích toàn phan của ho ga (không nap) là 2(x + y)h + xy = 4x2 + 5xy = m Ta có h phương trình: x2y = 1600 (1) Yêu cau của bài toán là tìm x, y sao cho m nhỏ nhat? 1600 Tà phương trình (1) suy ra y = , ta có x m = 4x2 + 5x. 1600 = 4x2 + 8000 . x2 Khảo sát hàm so f(x) x = 4x2 + 8000 , x > 0 suy ra di n tích toàn phan của ho ga nhỏ x nhat bang 1200cm3, khi x = 10cm thì y = 6cm. Suy ra di n tích đáy ho ga là 10.16 = 160(cm2).
  • 64. 61 Viết đề tài giá sinh viên – ZALO:0973.287.149-TEAMLUANVAN.COM Ket lu n Lu n văn Ve h phương trình phi tuyen và úng dựng trình bày nhǎng van đe sau: 1. Lu n văn đã trình bày chi tiet m®t so kien thác cơ bản của h phương trình nói chung và h phương trình phi tuyen nói riêng. 2. Trình bày các dạng toán cụ the của h phương trình phi tuyen. 3. Trình bày m®t so phương pháp giải h phương trình phi tuyen thường g p. 4. Trình bày các đe toán thi hoc sinh giỏi, tuyen sinh các trường chuyên toán và thi đại hoc, cao đȁng liên quan đen h phương trình phi tuyen. 5. Nêu m®t vài áng dụng của h phương trình phi tuyen trong khoa hoc và đời song.
  • 65. 62 Viết đề tài giá sinh viên – ZALO:0973.287.149-TEAMLUANVAN.COM Tài li u tham khảo Tieng Vi t [1] Tran Thị Vân Anh (2009), Hướng dȁn giải các dạng bài t¾p tù các đe thi Quoc Gia môn Toán của B® giáo dực và đào tạo, NXB ĐHQG Hà N®i. [2] Hoàng Thị Dịu (2014), Lu n văn thạc sĩ M®t so phương pháp giải h phương trình và h bat phương trình đại so, Trường ĐHKHTN - ĐHQG Hà N®i. [3] Tran Trong Hu (2015), Đại so tuyen tính và hình hoc giải tích, NXB ĐHQG Hà N®i. [4] Nguyen Văn M u, Lê Ngoc Lăng, Phạm The Long, Nguyen Minh Tuan (2006), Các đe thi Olympic Sinh viên toàn quoc, NXB Giáo dục. [5] Lê Hoành Phò (2015), Phương pháp giải ba chuyên đe toán khó, NXB ĐHQG Hà N®i. [6] Nguyen Văn Vĩnh, Nguyen Đác Đong và m®t so đong nghi p (2011), 23 Chuyên đe giải 1001 Bài toán sơ cap, NXB Giáo dục Vi t Nam. ————————————————————————————————–